You are on page 1of 132

, , '4 , ,

' '
• • • • • • • ...
, , \ , ,
' \ ,. \ ' \ '
• • .. ' • • • ' • •
\ , , \ I \
- - - ' - - - - -
,
, , I I
' ' \ ' \ ' '
- - - - - - ' ' - -
, , , , , ,
\ ' \ ' I
- ' -' - - - - - ' -
I
, \
, \ , \ ,
' '
- - ' - - ' - -
, , , ,
- ' ,' -
,
' ' ' \
- ' ' - - ' - - ' - -
, , -
' ' ' '
- - \ , - -
, ,
- ' - ' -
, - , I
,
' '
- - ' - -
, ,
\ ' I \ '
- , - \
, , - , -
' , , \ ' , \ ' ,
- - ' - - - - -
, \ , , , , \ , - , \
\ \ ' \
- \
- \
- - \

' -
, - \ , -
, -,
\ , \ , ,
- - - - - - ' - -
, , \ , I \
' ; \ I \ ' , \
- ' - - - - - - -
, , , , , \ , ,
' ' ' \ ' \
, ' , , ,
• ... ' • •
' • •
' • •
\ , \ , \ \
I \ \ ' I \
• ' ' • • • • ' • • •
, \ I \
' I ' ' I \ , \ - I
*****
Merhaba Hocam.
''
Ad ı m Çağ r ı Tıp Fakültesi 5. sınıf öğrencisiyim.
Öncelikle ben, eğitim paketinden önce kendi
başıma konu anlat ıml ı kitaplardan konuyu okuyup
sonrasında sorularını çözmekten çok yorulmuştum.
Bu işin böyle çok yavaş ile rleyeceğ i ni anlayıp
*****
''
e Hocam merhaba sizlere uzun bir teşekkür etmek
istiyorum. Mikrobiyoloji benim ilk tekrarına
başlamaya korktuğum, hazırlanan arkadaşlarımın
ezberinin hep unutulduğunu söylediği korkutucu
b ir dersti. Ben hiç dershaneye gitmedim, yeni
mezun da değilim sizin kitap ve videolarınız
bile
*****
''
e Hocam öncelikle şunun için çok teşekkü r
etmeliyim. Kitabın yazarı olan hocadan dilediğim
zaman en iyi şekilde soru ları ma yanıt alıyor
olabilmek çok büyük bir şanş. Bu kadar iyi
patofizyoloji anlatarak karş ı tarafın verim alamaması
mümkün değil zaten. Youtubedan olsun instagram
acaba TUS hayallerini erteleyeyim mi diye sayesinde mikrobiyoloji en sevdiğim ders oldu kanalınızdan olsun ilk çıktığınız yıllarda da sizi
düşünüyordum. Ama sizinle birlikte tekrardan resmen temel bilimlerde so ruları çözebiliyor takip ediyor ve videoların ı zı kısa da olsa
bir umudum oluştu. Fakültenin birinci, ikinci, olmak en büyük motivemdi, sın ava yaklaşan din liyordum. Ve hep hayran kalıyordum. Şimdi
üçüncü sınıfında b iyokimya ders lerinde çok denemelerde genelde hepsi doğru ya da 1 boş tüm dahiliyeyi sizden dinliyor olmak gerçekten
zorlanmıştım. Hatta komitelerden yarı yediğim çıkıyordu sınavda 2 boştu malesef bu da sınav harika. İşte şimdi oldu diyorum her dersten sonra.
bile olmuştur. O zamanlarda bir türlü anlayama- kaynaklı herkesin bildiği gibi garip bir sınavdı bu Dahiliye demek böyle bir dersm iş . Tusta. hiç
dığım konuları, s izin mekanizmasıyla, mantığıyla sene. İmmunolojide anlattıklarınızla başka ders- çalışmak istemediğim korkulu rüyam diyebi leceğim
ve görse l animasyonlarla anlatmanız sayesinde lerden de soru çözdürdünüz hep bana. bir dersken şümdi sizin sayenizde as l ında ne kadar
minimum ezber ile kavrayabildim. Ge rçekten her Sizin kıymetli kaynaklarınız mikrobiyoloji zevkl i b ir dersmiş onu görüyorum. Hepsi sizin
bir animasyon ve konu anlatım metodu dersinden çekinen tüm hekim arka da şlarıma sayenizde. Benim için önemli olan başka bir konu
" Öğrenci daha kolay nasıl anlayabilir?" tavsiyemdir. da iyi niyettir her zaman. Sizin derslerin
amacıyla düşünülmüş ve üzerine çok eme k verilmiş. sonundaki iyi ki varsın ız sözünüz bile onu
TUS 'ta yakaladığınız sorularla da bu titiz gösteriyor. Uzaktan hiç tanımadan bile bunu
çalışmanızın ka r şılığını alıyorsunuz. Hocam çok • Hocam muhteşemsiniz. Kodlamaların ız sayesinde hissett.irebildiniz bana hocam.
fazla vaktinizi almadan birkaç şey söyleyip bile mikrobiyoloji ve klinikten bir çok soruyu Asıl siz iyi ki varsın ı z:) Umarım bir gün yüz yüze
sözlerime son vermek istiyorum. çözebildim. İyi ki varsınız. tanıyabilme imkanı bulabilirim sizi. Her şey için
Te krardan TUS umutlarımı yeşe rt meniz dışında; teşekkür etmek isterim.
bana "Aa .. Aslında anlayabiliyormuşum."
dedirttiğiniz ve yeni şeyleri öğrenmeye olan e İyi akşamlar hocam. Allah razı olsun s izden.
motivasyonumu s ağ lad ığın ız için çok teşekkür Geçen sınavda 6 net yaptığım mikrobiyolojiden e Hocam emeğinize sağlık videolarınızı dinliyorum
ederim. İnşallah ben de mezun olduktan şimdi 19 doğru bekliyorum. Hakkınızı helal edin . o ldukça güzel ve her konuyu fizyopatolojisiyle
sonra sizin gibi iyi bir hekim olabilirim. beraber anlatmanız anlaşılır o l masını sağlıyor.

BİYOKiMYA DAHiLiYE

,,
**
Hocam çok şükür bitirdim ama bir daha dinlemeyi
düşünüyorum kasımdan bir ay önce almıştım yen i
eklenen yerleri görmek için dinleyeceğim, çok
verim aldım ,özellikle aynı sayfada çok sorulan
karsilasitirma soruları için çok iyi hocam, barbiturat
benzodiazepin gibi, ders çalışmak zevki oluyor
*****
''
e Görsellerle desteklenmiş olması ,sıkmayan ve
çok uzun olmayan videolarla dinlenme kolaylığı,
vip soru çözümleri ve burda şıkla rın tek tek
irdelenip bir nevi konu tekrarının yapılmış olması
çok hoşuma g itti.
e Tıp
*****
akademisi ile tanıştığım için ke nd imi hep
şanslı hissettim. Fizyoloji dersleri de bu his lerimi
artt ı rdı. Diğer demlerde olduğu gibi şiir gibi geldi
dersler. Yorumum abartılı gelebilir, ancak konu l arı
unutmayacak şekilde öğre·necek olmanın ve sınavın
stresinin kenarda bırakılması ile dersi şii r din l ermiş
hocam sorularida çözdüğümuzden, ilk önce genel gibi dinlememek elde değil.Az soru çıktığ ı için
mekanizmayı şemalarla anlatmanız konu bütünlüğü fizyoloji dersini alıp almamakta kararsız kalmışt ı m.
için çok iyi o luyor ,çok büyük b ir emek var ortada, e Hocam selamlar, ben kitabın sonundaki qr Ancak dersleri dinlemeye başladıktan sonra bir kez
hiç pişman olmadım çok çok tetekkür ederim. kodundan bağlanılan anketten görüşlerimi olsun pişman olmadım. Çünkü bu dersler öylesine
paylaştım. Elinize emeğinize sağlık eğitimi bitirdim temel bir bilgi sağlıyor ki ve bunu ezberle değil,
ve çok faydalı buldum. Her şeyi mekanizmalarıyla adım adım mantık la inşa ediyor olmak eğer düzenl i
e öncelikle farma calisirken butun ilaclari bir sekil çok akılda kalıcı bir şekilde anlatmışsınız patolojiye çalışılırsa diğer tüm derslere fayda sağla yacak diye
uzerinden gormek icin kendim cizerdim. Sizin hak im hissediyorum artık. düşünüyorum. Emeklerin iz için çok teşekkürler
kitabinizda ve anlatim inizda butun ilaçların bir hocam, naçizane sadece tusta değil hekimlerin
gorsel uzerinde toplan mis olma si cok guzel oldu. yetişmesinde de büyük bir rolünüz olduğu
Daha cok usmle tarzinda oluşunuz tus' un da son kanaatindeyim. Dersleriniz dinlemek büyük şans.
zamanlarda buna evrildigi için bayagi bi avantajli e Konuların sayfalardan taşmadan 1 2 sayfada Teşekkürler tıp akademisi.
bir konumda oldu k diyebi lirim. Bütün sinavla rda anlatılması, bütünlüğün bozulması çok güzel.
cıkmış olan sorularin nasil gelebileceğini aktarm is Ezberlemeyi, akılda kalıcılığı arttırıyor. Kitabı
olmasi acisindan da cok başarıl ı buldum. Kitabin tercih etme sebebim de buydu. Düz yazı olan Hocam asla anlamadığ ı m bir noktaydı
kisa ve tekrara musait oluşu cok guzel. Farmakoloji onlarca sayfa yerine konunun özeti tek bir sayfa parathormon ve etkileri teşekkürler çok iyi
de mekanizmanin geçtiği yerlerde cok keyif aldim. ile öğr9nmek, hatırlamak çok daha kolay. anladım sayenizde.
Emeginize yureginize saglik tesekkur ederim

*****
''
e Merhabalar hocam. Hem teşekkür hem de ufak
bir rica için msj atıyorum. Sizin vesilenizle kabusum
olan küçük stajlar artik en zevk aldığım derslerden
biri o ldu . Kolay ve anlaş ılı r kitabın ız ve anlatımınız
için cok teşekkür ederim.
*****
anlama açısından çok iyi bir anlatım. ''
• Mekanizmalarin akılda kalması ve mantığı

• Öncelikle iyi ki dersleri satın almışım. Çok çok


memnun kaldım. Hocamız kısa sürede ka dın
*****
• Resiml i anlatımları çok
''
akılda kalıcıydı.

• Öncelikle çok teşekkürler vi deolarınızı ve


kitabınızı aldım ve çok işime yarıyor öğrendiğimi
doğumu tekrar etmekle kalmay ıp aynı zamanda hissediyorum ve ben sınava sizin kitabın ız ve
çözdüğü sorularla sınavda sorulara nasıl videolarla hazırlanıyorum . Ben açıkçası
e Hocam ağzınıza sağlık ben sizi hiç tanımadığım yaklaşmamız gerektiği ile ilgili bize birçok şey patofizyoloji o larak çok eksiktim ezber üzerine
halde dersinizi ald ı m tip akademisinin hocalarına kattı. Emeği geçen herkese teşekkürler. gidiyordum ama bir yerden sonrası mümkün
güvendiğim için iyi ki de öyle yapmışım iyi ki değil yani ama iyi ki size rastlamışım böyle
varsınız. mantığını anlamak çok daha akılda kalıcı,
bol tekrar etmeniz benim için çok iyi tam aa
• Hocam size binlerce kez teşekkür ederim. bu burda da vardı dediğim yerlerde sizde
Kadın doğum benim için kocaman bir soru bahsediyorsunuz tekrarlarla çok iyi oluyor
e Hocam merhaba şimdi uzun bir teşekkür yazısı işaretiydi, sınava bu kadar az zaman çok teşekkü rler.
yazmak istiyo rum ... Sizin tablolarınız, akıcı kala , harika kod larla kafama kaz ınd ı resmen
anlatımınız. sayesinde öğrend i m konuları ve öğrettiklerin iz. Kadın doğumun 12 sorusu
neticesinde nasip oldu ... İnşalla h sizler g ibi klinik isteyen he rkes için çok kıymetli,
hocalarımızın sayılarının artmasını temenni bize böyle bir imkan sunduğunuz için kendi
ediyorum, emekleriniz için teşekkürler. adıma çok teşek kür ederim. Ben çok verim aldım .
Pediatriye de yardımcı olacak şekild e güzel bir
kamp olmuş. Emeğinize sağl ık.

KADIN DOĞUM
ICEJJ ~~a e is"

TUS SORU KAM'P I


1 •

t>AHILIYE
Dr. H.Sefa Ki LIÇKAP
D gfladem.isi
GASTROENTEROLO:Jİ
1- 56 yaşında erkek hasta dahiliye polikliniğine 6 aydan uzun Akut karın ağrısına yaklaşım Kronik karın ağrısına yaklaşım
ilk değerlendirme:
süredir olan karın ağrısı şikayetiyle başvuruyor. Öyküsünde son Öykü,fizikmuayene
Tam kan sayımı, geniş biyokimya idrar analizi,
ilk değerlendirme:
Öykü,fizikmuayene
Tamkansayımı,genişbiyokimyaidrar
2 ayda 9 kilo kaybettiğini ve ev ölçümlerinde ateşinin genellikle transaminaz,amilaz,lipazdüzeyi
+
analizi,transaminaz,amilaz,lipazdüzeyi

38°( ve üzeri olduğu öğreniliyor. Bakılan laboratuvarında; Peritonitleuyumlu


akutbatın
Cerrahi
Bilinen altta yatan
't Hoyır
Lökosit: 11.000/mm3 Barsak dışı Sebep bulunursamedikalya
hastalık
Tedavi et

nedenler da cerrahi tedavi


Hb: 9,6 gr/dL tYok
Yok

Abdomino-pelvik Sebep bulunursa medikal ya Kilokaybı,ateş,

Trombosit: 126.000/mm3 BT
♦ Normal
da cerrahi tedavi sistemiksernptomlar

AST/ ALT/ ALP ve GGT normal aralıkta izleniyor. Abdomino-pelvik


USG
--+ Akutkolesistittanısıalırsa
cerrahi >4 0yaşveya IBS Etiyoloji bulunursa

Bu hastanın en olası tanısına yönelik bu aşamadan sonra ilk ♦ Normal kriterlerini karşılamıyor tedaviet
ÖGD, Kolonoskopi
--+ Sebepbulunursamedikalya l Normal
yapılması gereken hangisidir?
+/- ince barsak
görüntüleme da cerrahi tedavi
Etiyolojibulunursa
"t Normal BTgörüntüleme tedaviet
A. Kolonoskopi Takip,ağrıyönetimi,angiografi, EEG, porfiri için idrar
ı Normal
testi,jinekolojlk/cerrahi/psikiyatrik konsultasyon

B. Batın tomografisi ÖGD, Kolonoskopi +/- Etiyolojibulunursa


ince barsak tedaviet
C. Hepatobilier ultrason görüntüleme

! Nurmoı
D. Özefag ogastroduodenoskopi Takip, ağrı yönetimi, USG, mide boşalma zamanı,
ERCP, EUS, angiografi, jinekolojik /cerrahi/
E. Endoskopik ultrasonografi pslklyatrlkkonsultasyon, asit varise parasentez

2- Yemeklerden sonra karın ağrısı, hazımsızlık şikayeti ve gaz 3- 34 yaşında erkek hasta 6 aydan beridir zaman zaman bulantı/
sancısıyla gelen 32 yaşındaki kadın hastanın öyküsünden bu _ kusmanın eşlik ettiği yemeklerden sonra şişkinlik, karın ağrısı,
şikayetlerinin 1 yıldan beridir var olduğu öğreniliyor. Bakılan hazımsızlık şikayetiyle başvuruyor. Fizik muayenesi olağan ve
hemogramında laboratuvar verileri normal sınırlarda izleniyor. Dispepsi varlığı
Lökosit: 8.000/mm 3 düşünülen hastanın tedavisi için hangi yaklaşım doğru
Hb: 13,8 gr/dL değildir?
Trombosit: 17 4.000/mm 3 izleniyor. A. Dispepsi hastalarına genelde psikiyatrik problemler de eşlik
Kilo kaybı tariflemeyen hastanın ilaç kullanmadığı ve sağlıklı edebilir ama antidepresan kullanımı önerilmez
beslenme için diyetisyen desteği aldığı öğreniliyor. Bu aşama­ B. H.pylori varlığı zemininde gelişirse eradikasyon tedavisi
dan sonra hasta için yapılması gereken ilk tetkik hangisi- uygulanır
dir? C. Antidepresan tedaviye gereğinde ek olarak SSRI ya da SNRI
A. Endoskopiyle üst GİS'i taramak grubu ilaçlar veril eb ilir
B. PPl+Bizmut+Metranidazol+ Tetrasiklin tedavisi başlamak D. Metoklopramid gibi prokinetik ajanlar yemek sonrası alındı­
C. Üre nefes testi yapmak ğında semptomları hafifletir
D. Hızlı üreaz testiyle h.pylori varlığını araştırmak E. Tedavide önemli basamaklardan biri dispepsi kliniği yapacak
E. Ampirik PPI tedavisi başlamak ilaç varlığını ve diyet uyumsuzluğunu sorguli3maktır

* > 55 yaş

Dispepsi -
* Kilo kaybı
* Kusma
* Anemi
* Ailede kanser öyküsü
-
f+)J
Endoskopi

HP eradikasyonu
İyileşme yoksa 4-8 hf
PPİ ➔ İşe yaramazsa
PPI kesilir 8-12 hfTAD
uygulanır başlanır

İlaç kullanımı veya H H.Pylori için non-


diyet düzensizliği invaziv test yapılır

Uygun tedavi

4-8 hf PPİ ➔ İşe


yaramazsa PPI kesilir İyileşme yoksa 4 hafta
8-12 hfTAD başlanır prokinetik etkili
metoklopramid

..
ba·şıanır
~r.,,r.ı;;u;ı'~ı!!':U:?~,.nn

L....i..~2----2~~____2~2.__~~~__:_____:_______~___.JIBI-
Ş!.~I!~l~· ~ıqmn,.tMI~rı_n lDı ~~ademisi
4· 94 yaşında bilinen alzheimer hastalığı olan, genel durumu Orofaringeal disfajl

kötü bakım evi hastası bir gece önce başlayan öksürük ve ateş
şikayetiyle getiriliyor. Fizik muayenesinde sağ orta lobda vibras- Ya pısal
l
Nörolojik Miyojenik

yon torasik ve matite artmış izleniyor. Çekilen akciğer grafisi de


aşağıdaki gibi geliyor. • svo • Myastenia Graves
• Zenker divertikülü
• Neoplazi • Parkinson • Polimiyozit
• Servikal web * ALS • Mix konnektif doku hast
• Krikofaringeal patolojiler • Guillain Bare se ndromu • Okülofaringeal mus. dis.
• Osteofitler • Huntington's korea • Paraneoplastik sendrom
• Konjenital anomaliler • Post-po lio sendromu • Miyotonik dlstrofi
• Boyun/ kranial cerrahi • Multiple skleroz • Sarkoidoz
• Kemoterapiye bağlı mukozit • Serebral pa lsl
• Beyin sapı tümörleri

Hastanın öyküsünden 1 ay önce geçici iskem ik atak ve akabinde


Özefagial disfaji
subaraknoid kanama geçirdiği, hastayla ilgilenen hemşireden
hastanın son günlerde yutmasının iyi olmadığı ve beslenme 1
Yap ısal
l
Odinofaji
l
Progresif
sırasında sık sık besinlerin ağzından geldiği öğreniliyor. Bu
hastayla ilgili aşağıdaki yorumlardan hangisi doğru
değildir? Aral ıklı
• Schatzki halkas ı
Hap özefajiti
Enfeksiyöz özefajit
Malignite
Skleroderma
• Özefagial web Skleroterapi Diffüz Özefagial spazm
A. Hastada aspirasyon pmömonisi gelişmiştir Değişken Kemoterapiye bağlı mukozit Akalazya
• Eozinofilik.özefajit Crohn /Behçet/ Liken
B. Yutma güçlüğü subaraknoid kanamaya bağlı transvers disfaji •

Peptik striktür
Hiatal herni
Büllöz pemfigoid
Kostik yanık

gelişmesine bağlı olabilir •



Ekstrinsik bası
Cerrahi stenoz
• Radyasyon özefajiti
C. Hastaya yutma fonksiyon testi yapılırsa yutma fonksiyonunun 1
• Ring özefagus
• Konjenital Özefagial stenoz
başlamasında kusur izlenir
D. Hastanın biyokimyasında CRP ve sedim artmış; hemogramda
lökositoz izlenebilir
E. Hastanın pnömonisine yönelik aminog likozid grubu bir
antibiyotik seçilebilir

S· Gastroözefagial reflü hastası olduğu bilinen 34 yaşında erkek Gıı.cfe, C oıeJajit Gırarle liJı ôıefaJjt

hasta son 1 haftadır ağrılı yutma güç lü ğü şikayetiyle başvuru­ Bir-vewS Si'nıeyJı 5 Te:pe.l\:yf

yor. Reflü özefajit düşünülerek yapılan endoskopide Los-Ange- ınm/'de,ı,kıJ,;.ük


f!Uıadet
mrn'iıl•m~ü:~tiill
ikiı adeu
bfrıl.'eş~nancalt"
tamenı;aı,ının,
1!.U.tnenıçapmm.
%,75,'tıın

te.pel!:?111 te.peJe.,1ii 'l'.iiS'te.11 anıru f-a!lasm,. trlıtaıı:

les sınıflamasına göre lümen çap ının %75'ten fazlasını tutan f.ıi,r.e,,meyı,ıın
rnulto-ıaf l'ezy~m
bırl~e<,en,
mukoJaU lezv,<ın
t!U.M mullo,al
l'e'Z')iOnl
n:ıakcızall lezyun

mukozal lezyon var lığı rapor ediliyor. Tarif edilen lezyon reflü
özefajitin hangi evres"ne ait olduğunu gösterir?
A. Grade A B. Grade B
C. Grade C D. Grade D PPII !edınıfsf~le IUirılfc t!aftiwıyeterıli.dT,-

E. Grade E

Barrett Özefagus Yönetimi


Barrett's yoksa klinik takip yeterlidir
~ Reflü nedeniyle takipli 28 yaşında kadın hasta od inofaji
şikayetiyle başvuruyor. Yapılan endoskopik biyopside şeki ld eki
Barrett's var
gibi özefagus epitelinin intestinal epite le değişimi ve goblet ancak displazi yok
Dü şük grade displazi Yüksek grade displazi

hücrelerinin varlığı rapor ediliyor ancak displazi izlenmiyor. Bu


aşamadan sonra en uygun takip şekli nasıl olmalıdır?
A. PPI tedavisiyle klinik takip
B. 3 yılda bir endoskopi
6-12 ayda bir Endoskopik mukoza!
C. Yılda bir endoskopi 3 yılda bir endoskopi ya da rezeksiyon yap ılır; ya nıt
endoskopik takip Radyofrekans a lın a maz sa
D. Endoskopik mukozal diseksiyon ablasyon özefajektomi ya p ılı r

E. Özefajektomi
7- Reflü nedeniyle takipli 28 yaşında kadın hasta odinofaji
şikayetiyle başvuruyor. Yapılan endoskopik biyopside yüksek
derece displazi izleniyor. Bu aşamadan sonra en uygun takip
şekli nasıl olmalıdır?
A. PPI tedavisiyle klinik takip B. 3 yılda bir endoskopi
C. Yılda bir endoskopi D. Özefajektomi
E. Endoskopik mukozal diseksiyon

8- Reflü nedeniyle takipli 28 yaşında kadın hasta odinofaji şika­


yetiyle başvuruyor. Yapılan endoskopik biyopside.yüksek derece
displazi izleniyor. Hastaya endoskopik mukozal diseksiyon uygu-
lanıyor ancak hastada semptomatik iyileşme sağlanmıyor.
Bu aşamadan sonra en uygun takip şekli nasıl olmalıdır?
A. PPI tedavisiyle klinik takip B. 3 yılda bir endoskopi
C. Yılda bir endoskopi D. Özefajektomi U)
E. Endoskopik mukozal diseksiyon

9- Özefagus motilite bozuklukları hakkında yazılanlardan hangi-


si doğru deği dir?
A. Akalazyada temel sorun AÖS gevşemesinde yetersizlik olup
tabloya aperistaltizm hakimdir
B. Diffüz özefagus spazmında koordine olmayan, tekrarlayıcı,
prematür kontraksiyonlar vardır ve göğüs ağrısı akalazyaya göre
çok daha ön plandadır
C. Fındıkkıran özefagusta peristaltizm normal olup mevcut
kontraksiyonların şiddeti artmıştır genelde 180 mmHg'dan
büyüktür
D. Hipertansif özefagusta peristaltizm normaldir ve kontraksi-
yonların şiddeti artmış ancak 180 mmHg'dan küçüktür
E. Akalazya sıklıkla ALS zemininde gelişir

10· 34 yaşında erkek hasta hem katı hem de sıvı gıdalara karşı
yutma güçlüğü şikayetiyle geliyor. Öyküsünde son 6 ayda 5 kilo
verdiği öğreniliyor. Laboratuvar inceleme ve detaylı anamnezde
alarm semptomları olmayan hastaya çekilen baryumlu grafi
akalazya lehine sonuçlanan hasta için aşağıdaki yorum-
lardan hangisi yanlı şt ır?
A. Hastada akalazyanın sekonder maligniteye bağlı gelişip
gelişmediği mutlaka endoskopiyle tetkik edilmelidir
B. Hastalık patogenezinde VİP ve NO azalmasının yeri vardır
C. Submukozal sahadaki sirküler kasların kesilerek AÖS basın ­
cının azaltılma işlemi POEM olarak bilinmekte ve bu yöntem
cerrahiye eşdeğer kabul edilmektedir
D. Akalazya prekanseröz bir hastalıktır ve uzun dönem yassı
hücreli kanser riski artmıştır
E. Akalazya tanılı hastalarda aspirasyon pnömonisi riski artmış
olup hastalar profilaktik antibiyotik tedavisi altında izlenir

..
- DJ ~flademisi
11· 24 yaşındaki genç kadın hasta yaşam stresi, hayat zorluğu
gerekçesiyle temizlik ürünleri satan bir dükkandan rastgele bir
şişe alıyor ve su içer gibi içiyor. İçtikten sonra saniyeler içinde
bulantı-kusması başlıyor ve genel durumu kötüleşiyor. Yolun
karşındaki otobüs bekleyen biri tarafından bu durum farkedi-
lince hasta hemen hastaneye götürülüyor. Fizik muayenede ve
anamnezde sırta vuran şiddetli ağrı tarfileyen hastaya çekilen
akciğer grafisinde şekildeki gibi tüm mediasten boyunca ser-
best hava izleniyl!l.
or.~ . ~~:":-:■ır---ı""!'!J

Bu hasta için aşağıdaki yorumlardan hangisi yanlı şt ır?


A. Artık nötralizan ajanlar tercih edilmemektedir
B. Emetik ajanlar yanık boyutunu_artıracağından dolayı verilmez
C. Rutin kortikosteroid uygulamaya gerek yok
D. Perforasyon boyutunu ölçmek için endoskopi yapılmalı
E. Geniş spekturumlu antibiyotik verilir

Kostik madde teması


12· 56 yaşında erkek hasta acil servise ani başlayan ve 1 saatten
beridir artarak devam eden karın ağrısı şikayetiyle başvuruyor.
Fizik muayenesinde batında rebound/defans pozitif izleniyor.
l
Göğüs ve batın grafisi
Perforasyon düşünülen hastada tanıyı kesinleştirmek için çekmek
yapılması gereken tetkik hangisidir?
A. Direkt batın grafisi B. MR inceleme l
Perforasyon yoksa
C. Bilgisayarlı tomografi D. Fluroskopi endoskopi yap ılır
E. Ultrasonografi

13- 33 yaşındaki kadın hasta ağ rılı yutma g ü çlüğü nedeniyle


başvuruyor. Oral mukoza muayenesinde beyaz plaklar izleniyor
ve yapılan endosko ik örüntü ·· · · r.

Lezyonun direk mikroskobik incelemesind e pseudohif yapan


mayalar görülüyor. Bu hasta için aşağıdaki risk faktörlerin-
den hangisinin etiyolojide yer alması beklenmez?
A. AIDS varlığı
B. Kemoterapi öyküs,ü
C. Topikal kortikosteroid kull a nımı

..
D. Sistemik kortikosteroid kull a nımı
E. Gastroözefageal reflüye yönelik omeprazol kull an ımı
~202J MART.TUS SORÜ :i<AMP·ı ~- '_: .• . .· ' .: . .. ·.
g !l.!!.I t•~d
l
~ aKa ► fi
.emısı 'ô~hıır\t . - . / -_ :.· : ~~~~-- .: . ·:_· ___ . ·.·. -_, ~ ... : ;: . '_'>· ... :..
14· Bilinen astım öyküsü olan 24 yaşındaki kadın hasta
retrosternal yanma ve odinofaji şikayetiyle doktora başvuruyor.
Öyküsünden 12 haftadır yüksek doz rabeprazol kullandığı ancak
fayda görmediği öğreniliyor. Bu aşamada sonra baryumlu grafi
çekiliyor, ardından endoskopi yapılıyor ve aşağıdaki görüntüler
elde ediliyor.

Bu hastanın en olası tanısı aşağıdakilerden hangisidir?


A. Eozinofilik özefajit
B. Plummer-Vinson sendromu
C. Diffüz intramural pseudodivertikül
D. Schatzki halkası
E. Radyasyon Özefajiti

15· 21 haftalık gebe olan 23 yaşındaki kadın hasta hematemez


ve melena şikayetiyle acil servise başvuruyor. Öyküde kanaması­
nın tekrarlayan kusma ve öğürme sonrası başladığı öğreniliyor.
Hastaya yapılan endoskopide şekildeki gibi gastro-özefagial

,,.
bileşkede lineer mukoza yırtığı izleniyor.

YapılanUSG ve fizik muayenede kronik karaciğer hastalığı


düşündürecek bulgusu olmayan hastada aşağıdaki yorum-
lardan hangisi doğru deği!dir?
A. Spesifik bir tedavisi yoktur; temel tedavi destek tedavisidir
B. Masif kanama olsa dahi bu bir arter kanaması olduğu için
Sengstaken-Blakemore tüpü faydasızdır
C. Eritrosit transfüzyon ihtiyacı nadirdir ve bu yüzden hasta için
izlem çoğunlukla yeterlidir
D. Hastanın orali kapatılıp 80 mg PPI bolus PPI sonrasında PPI
infüzyon başlanabilir
E. Kanamaya yönelik cerrahi gereksinimi nadirdir

İrwaziv ve norı-irıvaz i v testle rl e ta nı konur


16· Dispeptik yakınmalarla başvuran 56 yaşındaki erkek hasta- , Non-invaziv testler:
ya yapıla endoskopik biyopside üreaz (+)küçük pilili mikroor- Üre nefes testi: C13 ve 04 işaretli üre solüsyonu içeri lir • Üreaz C01 / Amonyum j
Eradikasyon kontrolü için en uygun testtir
ganizmalar izleniyor ve h.pylori enfeksiyonu tanısı konuluyor. Tüm testle r içinde doğrufok oranı en yüıksek tes t

Bu hasta için kullanılan testlerden hangisi uygun kullanım Seroloji: H.pylori'ye karşı gelişen lg G ve lg A antikorlcırllla bakılır
Bakte ri eradike olsa bile pozitif kalır Erad ikasyonda yeri yo~
alanıyla birlikte verilmemiştir? Toplum taramalarında kll' l lanı lı r
Aktif enfek~iyonU' göstermez
A. Üre nefes testi-> Eradikasyon kontrolü Antijen testi: Gayta ya da idrarda h.pylori antijeni bakılır Erad ikasyonda yeri var

B. Seroloji -> Toplum taraması , İnvaziv testler:

C. Üreaz testi-> Hızlı tanı imkanı KOltOr: H.pylori t anısında en spesifik testt ir ancak sensitif değildi r
Antibiyotik di rencinin gösterilmesinde ku ll aııılır

..
D. Kültür-> Antibiyotik direnci Hızlı Oreaz: Mide biyopsisinden a l ınan parçada ü reaz aık t ivite s i bakılır
Histoloji: H.pylo ri sonucu gelişen kronik gastr it/ atrofik gastriti göstermede
E. Seroloji -> Eradikasyon kontrolü H.pylori tansında gold standart testtir
17• Aşağıdakilerden hangisi özofagus yassı hücreli karsinom Skuamoz hümll lıınser Adenolıanser
foıılıM;toloıkıip squamoz hücreli kanser (Oıta Barr~zeminimfee:nım:gaE;ş,ir (Distal 1/3 öıefagus)
geliş minde rol oynayan faktörlerden biri deği dir? 1/3/
5ıca~ ııe-cekle, lKootilc. ~anıkl'aı I Krnaik iıita1~on K'rooiircffM ~ııel'sö;efagusJ' '.fafra ,e liisü
A. Plummer-Vinson sendromu mtıa:laı l fıın9,a fiıois ıal1cı .f !!P\Jlt~a,~/ Alkol !İ!l,Zt'3l/(f.~ıfte-/lıkel( dıısf~-st
~l'um ~J-~'.i'~somı sıı,nd'rvm.u!Mediasfsn~ra:d~ ~a,a i m:ııali oı efag;~ıııı dispf.ıstlkhıkrm ı.ırepi1e[:!ıfeı,'f geli1irr
B. Human papilloma virus le.r&ec", i ~gıif:eııik d i<ıı,:ıtilıilfü I Ma laıı~aı ili:: laaül epilel@ıı-5ı3mu~y;omııve:a cplijfrli
S.ele .,uw,,Mltoı,k,vi!<,oıimi, mı;ılind,,...ıı ek.sik~ji H-i?J!orı eni'alsi'j©no>p'aı ıf1kili ıf~§:il'dıı
C. Tilosis Tyleeis (ÖJE'2gus R-a,uer m; i e/JJ 1 wek !ıı,,ıal,.k} HER-2/neu genekspıe5yama f
lfar.1cıni a nem·-i
D. Fanconi anemisi
E. Alt özofagus halkası

18· 62 yaşındaki erkek hasta halsizlik, iştahsızlık, dispepsi


ve bacaklarda şişlik şikayetiyle geliyor. Bakılan laboratuvarda
albumin 1.8 mg/dl, total protein 3, 1 mg/dl izleniyor. Disfaji
de olması üzerine yapılan üst GİS endoskopisinde şekildeki
gibi mide pililerinde kalınlaşma ve müsinöz karakterde ciddi
anlamda sekresyon izleniyor. Mide öz suyu analizinde aklorhidri
rapor ediliyor.
1

Kalınlaşmış pililerden alınan biyopside ise faveoler hiperplazi;


glandüler atrofi izleniyor. Bu hastanın tedavisinde ilk tercih
ilaç hangisidir?
A. Setuksimab B. İmatinib mesilat
C. Regorafenib D. Sorafenib
E. Erlotinib

19· Romatoid artrit nedeniyle ağrılarına yönelik uzun süreli an-


tiinflamatuvar ilaç kullanmak zorunda olan ve daha önce peptik
ülser nedeniyle kanama geçirmiş hastada gastrointestinal yan
etkilerden korunmak için en uygun tedavi aşağıdakilerden
hangisidir?
A. Yalnızca düşük doz Cox-2 inhibitörü
B. Cox-2 inhibitörü + proton pompa inhibitörü
C. Cox-2 inhibitörü + düşük doz misoprostol
D. Cox-2 inhibitörü + H2 reseptör antagonistleri
E. Cox-2 inhibitörü + sukralfat

20- 52 yaşındaki erkek hasta son 4 aydır giderek artan şiddette


ve genellikle postprandial dönemde gelişen kusma şikayetiy l e
başvuruyor. Öyküsünden peptik ülser nedeniyle bir yıl önce he-
licobacter pylori eradikasyon tedavisi aldığ ını, kusmuğunun pis
kokulu olduğunu ve son yediği besin içeriklerinden oluştuğunu
söylüyor. Şikayet l erinin günden güne artma gösterdiğini de
ifade eden hastada en olası tanı aşağ ı dakilerden hangisidir?
A. Akut pankreatit B. Pilorik obstrüksiyon
C. Fonksiyonel dispepsi
E. İntrakraniya l basınç artışı
D. Besin zehirlenmesi

..
l
~
a tmJQ d
aKa ıemısı
2023 MART TUS SORU KAMPI
... Dahili e

21- Vakadaki hasta için tedaviye yönelik ilk yapılması


gereken aşağıdakilerden hangisidir?
A. Cerrahi dekomprasyon
B. Balon dilatasyon
C. Orali kapatıp hastayı izlemek
D. Bilgisayarlı tomografi çekmek
E. Orali kapatıp nazo-gastrik sonda takmak

22· 65 yaşındaki kadın hasta üst gastrointestinal sistem kana-


masıyla başvuruyor. Gastroskopide midede submukozal kitlesel
lezyon saptanıyor. Biyopsi C-kit (+) gastrointestinal stromal tü-
mörle uyumlu rapor ediliyor. Karın tomografisinde karaciğerd e
bilobar mu iti pi metastaz tespit ediliyor. Bu hasta için aşağıdaki
tedavi seçeneklerinden hangisi ilk olarak tercih edilmel"dir?
A. Palyatif sitotoksik kemoterapi
B. Metastazektomi
C. İmatinib mesilat tedavisi
D. İnterferon-a tedavisi
E. Radyofrekans ablasyon tedavisi

23- Yukarıdaki hastanın prognozunun esas belirleyicisi


hangisidir?
A. Mitoz
B. Tümör boyutu
C. Yerleşim yeri
D. Lenf nodu tutulumu
E. Uzak organ metastazı

24· 23 yaşındaki kardeşine glutene duyarl ı enteropati tanısı


konulan 34 yaşındaki erkek hastada yapılan ileri incelemede
anti-transglutaminaz ve Anti-endomisyum lg A antikorl aryl a
HLAD02 ve 8 pozitif bulunuyor. Aktif hiçbir şikayeti olmayan
hastaya yapıl an duodenal biyopsi tamamen normal geliyor.
Rutin gastroenteroloji takibine a lın an hastada takibin 3.yılında
yapılan duodenal biyopside şek ild e ki gibi villuslarda atrofi,
kriptlerde hiperplazi ve lenfosit art ı şı rapor ed iliyor ve hasta aktif
bir ş ikayet tanımlamıyor.

Bu hasta için en doğru yorum aşağıdakilerden hangisidir?


A. Hastada ilk olarak tropikal sprue geliştiği düşünülmelidir
B. Aşikar çölyak hastasıyken takipte lenfoma ge lişmişi t ir
C. Sessiz çölyak hastasıyken takipte latent hale gelmiştir

..
D. Latent çö lyak h astas ıyken takipte sessiz çö lyağ a dönüşmüştü r
E. Refrakter çö lyak hastasıyken ta kipte aş ikar çö lyak ge li şm i ştir
25- 21 yaşındaki kadın hasta kronik diyare, kilo kaybı, malabsorp-
siyon, alt ekstremitede ödem şikeyetiyle getiriliyor. Yapılan fizik
muayenesinde derin duyuda kayıp ve Romberg (+)izleniyor. Mu-
koza! biyopsi normal intestinal villusları enterositlerde multiple
yağ partikulleri olarak tanımlanmıştır. Ayrıca hemogramda anemi
izlenmesi üzerine yapılan periferik yaymanın görüntüsü aşağıdaki
gibi geliyor

Serum kolesterel ve trigliserid düzeyi düşük gelen bu hasta için


en olası tanı aşağıdakilerden hangisidir?
A. Giardia lamblia enfeksiyonu
B. Çölyak hastalığı
C. VVhipple hastalığı
D. İntestinal lenfanjiektazi
E. Abetalipoproteinemi

26- 68 yaşında bilinen diyabetes mellitusu olan hasta ishal, hal-


sizlik ve kilo kaybı şikayetiyle başvuruyor. Kan şeker regülasyonu
düzenli olmayan ve kendisine reçete edilen diyabet ilaçlarını
kullanmayan hastanın laboratuvarının B12 eksikliği ve demir
eksikliği anemisiyle uyumlu olduğu görülüyor. Folat değeri ise
yüksek geliyor. Çok uzun yıllar önce peptik ülser nedeniyle cer-
rahi geçirdiği öğrenilen hastanın kesin tanısı nasıl konur?
A. İnce barsak biyopsisinde PAS (+) makrofaj görmek
B. Gaytada kist ve trofozoidlerin gösterilmesi
C. İnce barsak biyopsisinde yağla dolu intestinal hücre görmek
D. Jejunal aspiratta 105 ve üzeri E.coli kolonizasyonu görmek
E. Gaytada a-1 antitripsin klirensi bakmak

27- Crohn hastalığı tanısı alan 34 yaşında qir erkek hasta şika ­
yetlerinde alevlenme nedeniyle başvuruyor. İshal, ateş, sağ alt
kadranda ağrısı olduğunu ve kilo kaybettiğini belirten hastanın
fizik muayenesinde vücut sıcaklığı 39 °C, bağırsak sesleri aktif,
sağ alt kadranda palpasyonla serozit düşündürecek hassasiyet
saptanıyor. Bakılan laboratuvarda lökosit 13.000/mm 3, CRP:34
mg/dl, sedimentasyon:43 mm/h izleniyor. Hastalığının ince
bağırsaklar ve terminal ileumda olduğunu; kolonda tutulum
olmadığı öğreniliyor. Laboratuvar incelemelerinde gaytada gizli
kan pozitif ve makrositik anemi bulunuyor. Bu hasta için en
uygun yaklaşım aşağıdakilerden hangisidir?
A. B12 ve folat replasmanı
B. İnfliksimab tedavisi
C. Mesalamin
DBudesonid
E. Sulfasalazin
l
~
g !!!l aKa
t•~d ... emısı

28- 41 yaşında erkek hasta 8 aydan beri ishal, karın ağrısı ve


kilo kaybı şikayetiyle geliyor. Fizik muayenede sağ alt kadranda
ele gelen kitle ve derin palpasyonda ciddi ağrı ve dispnesi ge-
lişiyor. İshal etiyolojisine yönelik yapılan kolonoskopik biyopsi
görüntüsü aşağıdaki gibi geliyor.

Bu hastanın tedavisinde aşağıdaki ajanlardan hangisinin akut


atak tedavisinde yeri yoktur?
A. 5-aminosalisilik asit
8. Kortikosteroid
C. Azatiopürin
D. Siklosporin
E. Anti-TNF ajanlar

29- 12 yıldır ülseratif kolit nedeniyle takipli hasta acil servise


şiddetli karın ağrısı şikayetiyle geliyor. Fizik muayenede batın
distandü izleniyor ve rebound / defans (+)bulunuyor. Ateşi
39 derece ve kalp atım hı z ı 124 atım /dk geliyor. Çekilen batın
grafide transvers kolon çapı şekildeki gibi 8 cm olarak ölçülüyor.

Bu hastanın tedavisine yönelik ilk yapılması gereken


hangisidir?
A. Kortikosteroid
8. Proktokolektomi
C. Azatiopürin
D. 5-aminosa lisilik asit
E. Vedolizumab

30- 12 yıldır ülseratif kolit nedeniyle takipli hasta acil servise


ş idd et li karın ağrıs ı şikayet iyl e
geliyor. Fizik muayenede batın
distandü izleniyor ve rebound / defans (+)bulunuyor. Ateşi
39 derece ve kalp atım hızı 124 atım /dk geliyo r. Çeki len batın
grafide transver kolon çapı 8 cm olarak ölçülüyor. Hastaya pulse
steroid veril iyor ancak klinik iyileşme sağ l anmıyor. Bu hasta nın
tedavisine yönelik ilk yapılması gereken hangisidir?
A. Kortikosteroid
8. Proktokolektom i
C. Azatiopürin
D. 5-aminosalisi lik asit
E. Vedolizumab
31- 54 yaşında obez hasta kilo kaybı ve yemeklerden sonra baş­
layan karın ağrısı şikayetiyle başvuruyor. Öyküsünden hastanın
kolesterol yüksekliği nedeniyle atorvastatin ve 2 ay önce pre-di-
yabet tanısı nedeniyle metformin kullandığı öğreniliyor. Yapılan
fizik muayenede hastanın ileri derecede obez olduğu dikkat
çekiyor. Bu hasta için en olası tanı aşağıdakilerden hangisidir?
A. Akut mezenter iskemi B. İskemik kolit
C. Kronik mezenter iskemi D. Sigmoid volvulus
E. Çekum tümörü

32· Kronik kalp yetmezliği nedeniyle digoksin kullanan hasta


acil servise generalize karın ağrısı şikayetiyle geliyor. Öyküsün-
den son 1 haftadır oral alımının bozulduğu öğreniliyor. Fizik
muayenede makattan kan geldiği görülüyor. Batın USG normal
sınırlarda izleniyor. Yapılan selektif angiografide süperior
mezenter arter akım normal ancak dallarında ılımlı daralma izle-
niyor. Bu hasta için en olası tanı aşağıdakilerden hangisidir?
A. Akut arteriyel emboli
B. Akut arteriyel tromboz
C. Fokal mezenter iskemi
D. Non-okluziv mezenter iskemi
E. İskemik kolit

33- 71 yaşında kadın hasta hematemez ve melena yakınması


ile yakınları tarafından bitkin ve halsiz bir halde acile getiriliyor.
Hastanın öyküsünde bilinen bir hastalığı olmadığı; antikoagü-
lan, antiagrenagan etkili hiçbir ilaç kullanmadığı ancak 2 yıl
önce de benzer şekilde kanama geçirdiği öğreniliyor. Yapılan
tetkiklerde hemoglobin 8 g/dL, hematokrit %25 izleniyor. Tran-
saminaz ve böbrek fonksiyon testleri normal sınılarda izlenen
hastaya vitalleri stabilleştirildikten sonra üst gis endoskopisi
yapılıyor. Endoskopide midenin küçük kurvaturunda pıhtı dışın ­
da ek bir patolojiye rastlanmıyor. Bu hastada kanama nedeni
aşağıdakilerden hangisidir?
A. Hemobilia B. Cameron lezyonu
C. Dieulafoy lezyon D. Gastrik vasküler ektazi
E. Peptik ülser

34- 72 yaşındaki kadın hasta 9 aydır devam eden halsizlik,


kilo kaybı, çarpıntı ve kabızlık şikayetleriyl e başvuruyor. Fizik
muayenesinde konjunktivaların solukluğu dışında bir patoloji
saptanmıyor. Laboratuvar incelemelerinde hemoglobin 8, 1 g/
dl, MCV 75 fL izleniyor. Yapılan periferik yaymada eritrositlerin
mikrositer karakterde olduğu dikkati çekiyor. Bu hasta için en
olası tanı ve en uygun tanı yöntemi hangisidir?
A. Lösemi - Kemik iliği biyopsisi
B. Sağ kolon tümörü - Kolonoskopi
C. Safra kesesi kanseri - ERCP
D. Lenfoma - Dalak biyopsisi
E. Kronik mezenter iskemi - Konvansiyonel anjiyografi
ıD ~~ademisi
35- Attenue familya! adenomatöz polipozis sendromunda kan-
ser tarama programı aşağıdakilerden hangisi gibi olmalıdır?
36- Herediter non-polipozis kolorektal kanser tarama progra-
mı aşağıdakilerden hangisi gibi olmalıdır?
A. 12 yaşından itibaren yıllık kolonoskopi
B. 30 yaşından itibaren 3 yılda bir kolonoskopi
C. 25 yaşından sonra 2 yılda bir kolonsokopi
D. 12 yaşından itibaren yıllık fleksible sigmoidoskopi
E. 40 yaşından itibaren 5 yılda bir kolonoskopi

37- 34 yaşındaki kadın hasta yemeklerden sonra olan hazımsız­


lık, şişkinlik, erken doyma, karın ağrısı şikayetiyle geliyor. Öykü-
sünden bu şikayetlerinin yıllardan beri olduğunu ve bu sebeple
birçok ilaç kullandığı, hatta hatırlamadığı bir dönemde h.pylori
için test yapıldığı ve sonucun negatif geldiği, karın ağrısının
yemeklerden sonra arttığı ve şikayetlerine zaman zaman diyare u,
zaman zaman da kabızlığın eşlik ettiği öğreniliyor. Laboratuvarı
normal olan hasta için en olası tanı hangisidir?
A. İrritable barsak sendromu B. Ülseratif kolit
C. Çekum tümörü D. Divertikülit atağı
E. Crohn hastalığı

38- Soliter rektal ülser ile ilgili hangisi yanlış bir ifadedir?
A. Her iki cinste eşit oranda gözlenir ve rektal kanama sebebidir
B. En sık distal rektumun ön duvarında izlenir
C. Rektumda tek bir derin ülserle karakterize patolojidir
D. İnternal intussusepsiyonla birliktelik gösterir
E. Kanser ekartasyonu için mutlaka dokudan biyopsi alınmalıdır

39- 54 yaşında erkek hasta yemeklerden sonra yaklaşık bir


saat süren karın şişliği şikayetiyle geliyor. Öyküsünden sık sık
hazımsızlık çe ktiği, karın a ğrısının olduğu ve dışkısının genel-
likle yumuşak olduğu, karın ağrısının yemekle arttığı, defekas-
yonla azaldığı ve ağrının zaman zaman kramp şeklinde olduğu
öğreniliyor. Daha önce de polikliniğe başvurduğunu ifade eden
hastaya h.pylori için 4'1ü tedavi başlanmış şikayetleri kısmen ge-
rilemiş. Hastaya irritabl bağırsak sendromu tanısı konulabilmesi
için mutlaka olması gereken semptom hangisidir?
A. Karın şişliği B. Dışkının yumuşak olması
C. Hazımsızlık D. Karın ağrısı
E. Antimikrobiyal tedaviye yanıtsızlık

40n İrritable barsak sendromu tanısı konulan hasta için hangi


yorum doğru deği dir?
A. Bu hastalar genelde doktor doktor dolaşırlar
B. Her va kaya posalı içerikten zengin diyet ve kepekli ekmek
önerilir
C. Karın ağrısı varlığında en sık tercih edilen ajan
anti-spazmolitik mebeverindir
D. Meberin yanıtsız vakalara muskulotrop etkili trimebutin

..
maleate tercih edilebilir
E. Diyare ağılıklı vakalarda difenoksilat veya loperamid verilir
HC'PATOLO,İ 3- 19 yaşında bir erkek hasta, yaklaşık 1Oaydır karında
şişkinlik, kronik kabızlık, kilo artışı ve halsizlik yakınmala ­
1- Aşağıdakilerden hangisi AST'yi ALT'ye göre daha fazla
rıyla başvuruyor. Laboratuvar incelemelerinde hemoglobin
yükselten durumlardan biri değildir?
düzeyi 10.3 g/dl, MCV 102 fl, serum albümin düzeyi 3.3 g/
A. Toksik hepatit
dl, açlık kan şekeri 109 mg/dl AST:123 IU/L, ALT:142 IU/L
1
B. Alkolik hepatit izleniyor. Hastada olası transaminaz yüksekliğinin sebebi
C. Wilson hastalığı olarak tiroid patolojisi düşünülüyor. Bu aşamadan sonra
D. İskemik hepatit bakılması gereken antikor aşağıdakilerden hangisidir?
E. Akut viral hepatit A. Anti -tissue-transglutaminaz lg A
B. Serum TSH düzeyi
C. Serbest T3 düzeyi
D. Total T3 düzeyi
E. SerbestT4 düzeyi

2- 19 yaşında bir erkek hasta, yaklaşık 10 aydır karında şişkinlik, 4- Sağ üst kadran ağrısıyla gelen 76 yaşındaki obez erkek
kronik ishal, kilo kaybı, halsizlik ve ağızda kronik aftöz ülseras- hastada bakılan AST:219 IU/L, ALT:7 6 IU/L geliyor. Bakılan
yonlar yakınmalarıyla başvuruyor. Laboratuvar incelemelerinde koagülasyon parametrelerinden PT, INR aPTT normal sınır­
hemoglobin düzeyi 10.3 g/dl, MCV 74 fL, MCH 25 pg, serum larda izlenen hastada transaminaz yüksekliğinin en olası
albümin düzeyi 3.3 g/dl, açlık kan şekeri 109 mg/dl, AST:123 ~ sebebi aşağıdakilerden hangisidir?
IU/L, ALT: 142 IU/L izleniyor. Hastada olası tra nsaminaz yüksekli - A. Hepatosteatoz
ğinin sebebi olarak çölyak hastalığı düşünülüyor. Bu aşamadan B. Akut bilier obstrükiyon
sonra bakılması gereken antikor aşağıdakilerden hangisi- C. Akut viral hepatit
dir? D. Akut sağ kalp yetmezliği
A. Anti-tissue-transglutaminaz lg A E. Alkol kullanımı
B. Anti-endomisyum lg A
C. Anti-deamine gliadin lg G
D. Anti-deamine gliadin lg A
E. Anti -endomisyum lg G

CK ve aldolaz Öykü, fizik muayene, ilaç Potansiyel toksinler


bakılarak kas kaynaklı +-- kullanımı, madde suistimali, --. kesilir ve transaminaz
etiyolojiler dışlanır alkol alımı düzeyi takip edilir

{!1ffel\İi
Hbe Ag, Anti Hbe,
HBV ONA
ltllaS; ~~ f+)
1
l ~te.rısi~

HbsAg, Anti-HBs, Anti-


J.>.ıııı_uii ltf(C\;/; M HCV RNA
HCV genotipi

HBc, Anti HCV, HAV lg M


tıı~~f S'Ml½\ı v.e.y,a, Dem ir, TDBK, llIDB!k D:t
1!1(~7~1I f+) lJrıaıııısJerıitııı s--aı .. Dt Ferritin
Transferin saturasyonu
Karaciğer biyopsisi HFE gen analizi
ANA, SMA, LKM-1
Karaciğer biyopsisi
Serum elektroforezi,
Serüloplazmin
Batın USG Kilo verme
Karaciğer biyopsisi Egzersiz

ı
IDi!ğ'e rı­
s.elber>l @rı-

a-1 AT fenotipi
Normalin 2 katı
TSH, T3, T4

..
yükseklik varsa biyopsi
TTG lg A
la
I.!!!! !!!!I
tı~
aKad emısı ıır--..ri"ft:......;_;
. " ...
1
ııiııiıAıl........__..;___ _ _ _ _ _ _ _- - - - ~ _ . . . ; . . ._ _- -_ _ _ _ _..;__.....;;....;.;.__~~;;.;;.

5- Sağ üst kadran ağrısıyla gelen 76 yaşındaki obez erkek hasta-


da bakılan AST:675 IU/L,ALT:756 IU/Lgeliyor. Bakılan koagülas-
yon parametrelerinden PT, INR aPTT normal sınırlarda izlenen
hastada aşağıdakilerden hangisinin transaminaz yüksekliği
sebebi olması beklenmez?
A. Kronik vira! hepatit
B. Akut bilier obstrükiyon
C. Akut vira! hepatit
D. Akut sağ kalp yetmezliği
E. İskemik hepatit

6- Kronik karaciğer hastalığıyla takipli 54 yaşındaki erkek hasta-


da bakılan değerlerinde lg Adüzeyi çok yüksek izleniyor. Sadece 1

bu kadar bilgiyle hastanın kronik hastalığının hangi etiyoloji•


ye bağlı olduğu söylenebilir?
A. Otoimmün hepatit
B. Alkolik hepatit
C. Primer sklerozan kolanjit
D. Primer bilier kolanjit
E. Otoimmün pankreatit

KC Biyopsi Endikasyonlan
7- Aşağıdakilerden hangisi karaciğer biyopsi endikasyonla- Kronik hepatit
rın dan biri değildir? Nedeni bilinmeyen hepatosellüler hasaır
A. Radyolojide karakterize edilemeyen lezyonlar
Açıklaınaımaıyan hepato l splenomegaıli
B. Uzamış hepatit
C. Nedeni bilinmeyen ateş Radyolojide taınımlaınaımayan lezyonlar
D. BT'e hemanjiomla uyumlu görünüm Nledeni bilinmeyen aıteş
E. Açıklanamayan hepatomegali Lenfomaı evrelemesi
Uzamış hepaıtit

8- 41 yaşında kadın hasta sağ üst kadran ağrısı şikayetiyle


başvuruyor. Bakılan laboratuvarında AST:65 IU/L, ALT:43 IU/L,
ALP:213 IU/L, GGT:312 IU/L, total bilirubin:12 mg/dl, direkt
bilirubin :8 mg/dl izleniyor. Bakılan tam idrar analizinde üro-
bilinojen düşük geliyor. Bu hastada ilk yapılması gereken
görüntüleme yöntemi aşa ğıdakilerden hangisidir?
A. Ultrasonografi
B. Bilgisayarlı tomografi
C. Magnetik rezonans görüntüleme
D. Biliyer sintigrafi
E. Retikülosit bakılması
9- 21 yaşında bir erkek hasta sarılık nedeniyle başvuruyor. Fizik Sarılık

muayenede patolojik bir bulguya rastlanmıyor. Hastanın yoğun


lıolelndlrekt hole direkt
iş temposunda olduğu, yakın zamanda halsizlik, eklem ve kas hiperbilirObinemi hi~rbfürUbinemi

ağrıları, ateş ve skleralarda sarılık nedeniyle başka bir doktora Hemo!izbu!gularıvarsa Hemolizbulgularıyoksa KCnveUSGoorma!ise
herediterhiperbilirübineml 1'
daha başvurduğu; bunun dışında bilinen bir hastalığı ve ilaç
hemo1iıikanemi Dub inJohnsonı:eRotor MT/N.T/AlP/ GGT
düjünü!melidir akla gelmeli sendromu akla gelmeli

kullanım öyküsü bulunmadığı öğreniliyor. Hastanın daha önce


Bilirubinl·S 8llirubin6-20 Bitirubin>20 USGnormalise USG'deİIISYveEHSYd ilate,
yaptığı başvuru sırasında yapılan laboratuvar tetkiklerinde total mg/dl ➔ Gilbert mg/dl ➔ CN Hp l mg/dl ➔ CN tip 1 intrahepatikkolesı az
düşünülür
l<D!edokgenişiseektrahepatik
kolestaz d üşünülmeli

bilirübin düzeyi 3.8 mg/dL, direkt bilirübin düzeyi 0.3 mg/dL; l Viralhepalit,alkolik Koledokolitiyazis,
al kalen fosfataz ve GGT düzeyleri, karaciğer hasar testleri ve viral
Direkt+lndirekt
hiperbilirübinemi hepat it,ilaıfar, P8S,PSK, ko!edolctümörü,PSK,
!istemikhasıatı kla r pankreas kanseri

hepatite bağlı serolojik testleri normal olarak bulunuyor. bu


MT/N.T/ALP/GGf 1' IST/ALT/AIY/GGT 1'
aşamadan sonra ilk yapılması gereken hangisidir? USGnorma!separankimal
hastalıkdüşürıülür
USG'desafrayoludilateyse
uzamışkolestazdüşünü!ür

A. Gilbert sendromu düşünülerek açlık testi


B. Fenobarbital verip sarılığın prognozunu izlemek
C. Retikülosit sayısı bakmak
D. Abdominopelvik USG
E. Toksik etiyolojiler için karaciğer biyopsisi

10- 10 yıldan beri her akşam alkol kullanım öyküsü olan 42


yaşındaki kadın hasta acil servise bilinç bulanıklığı şikayetiyle
getiriliyor. Bakılan rutinlerinde AST:254 IU/L, ALT:121 IU/L,
GGT:1286 IU/L, ALP:61 IU/L izleniyor. Koagülasyon testlerinde
INR:2, 1 ölçülüyor.
Hastada ön planda alkolik hepatit düşünülüyor ve steroid teda-
visi planlanıyor. Aşağıdaki durumlardan hangisi steroid verme
için kontrendike durumlardan birdir?
A. Aktif enfeksiyon va r lığı
B. Maddrey skorunun 32 ve üzeri olması
C. MELD skorunun 21 ve üzeri olması
D. Eş lik eden GİS kanama varlığı
E. Hepatik ensefalopati gelişmiş olması

11· 44 yaşında erkek hastanın bakılan seroloj isinde HbsAg İnaktif taşıyıa
(+)ancak HBV ONA ve HbeAg negatif geliyor. Biyokimyada AST Sağlıklı taş ıyıcı
olarak da bilinir
ve ALT normal sınırlarda izlenen hasta için en doğru yorum Virüsvar HbsAg(+)
hangisidir? Aktivasyon bulgusu yok
A. İmmün toleran faz HBV DNA(-)ve HbeAg (-)
B. İmmün aktif faz Kronikleşme riski vardır
C. İnaktif taşıyıcı HCC gel i şebi lir
D. Kronik aktif hepatit
Tedavi end ikasyonu yoktur
E. Doğal bağışık

İmmüntoleran faz
12- 44 yaşında erkek hastanın bakılanserolojisinde HbsAg (+),
HBV ONA 2.000.000 IU/ml olarak ölçülüyor. Biyokimyada AST ve Virüs aktiftir HbsAg (+)
ALT ve karaciğer biyopsisi normal sınırlarda izlenen hasta için en Replikasyon devam eder
doğru yorum hangisidir? HBV ONA(+) ve HbeAg (+)
A. İmmün toleran faz Virüse karşı immün yanıt
B. İmmün aktif faz yoktur
C. İn aktif taşıyıcı Karaciğer bx normal
D. Kronik aktif hepatit AST ve ALT normaldir
E. Doğal bağışıklık Tedavi endikasyonu yoktur
la
I.!!!! !l!!!I
trg d
aKa
. " .,..___. . . .;__·
emısı ....................................._......,..........._ ~ - - - - - -
13- 44 yaşında erkek hastanın bakılan serolojisinde HbsAg (+ ), İmmünaktif faz
HBV DNA 2.000.000 IU/ml olarak ölçülüyor. Biyokimyada AST: Virüs var HbsAg (+)
234 U/L, ALT:312 U/L ve yapılan karaciğer biyopsisinde hepatik Replikasyon devam eder
fibrozisi izleniyor. Bu hasta için en doğru yorum hangisidir?
HBV DNA (+)ve HbeAg (+)
A. İmmün toleran faz Virüse karşı immün yanıt
B. İmmün aktif faz
vardır
C. İnaktif taşıyıcı
Karaciğer hasarıvar
D. Kronik aktif Chepatiti
AST ve ALT yüksektir
E. Doğal bağışık
Tedavi endikasyonu vardır

14- 44 yaşında erkek hastanın serolojisi aşağıdaki gibidir 15- 44 yaşında erkek hastanın serolojisi aşağıdaki gibidir
HbsAg : + HbsAg: +
AntiHbs: AntiHbs:
AntiHbc lg M AntiHbc lg M
AntiHbc lg G + AntiHbc lg G +
HbeAg : + HbeAg:
HBV DNA: +++ HBVDNA: +++
Bu hasta için en doğru yorum hangisidir? Bu hasta için en doğru yorum hangisidir?
A.Akut enfeksiyon A.Akut enfeksiyon
B.Kronik aktif hepatit B.Kronik aktif hepatit
C.Kronik aktif hepatit prekor mutand suş C.Kronik aktif hepatit prekor mutand suş
O.Doğal bağışıklık O.Doğal bağışıklık
E.Aşıyla bağışıklık E.Aş ıyla bağışıklık

HbsAg Anti-Hbs Anti-Hbc lgM Anti-Hbc lgG HbeAg Anti-Hbe ALT HBVDNA
Akut enfeksiyon + + -/+ + tt tt
Pencere dönemi + -/+ + t -it

Sağlıklı taşıyıcı + + + N -it

Kronik aktif hepatit + + + tf ttt


Kronik aktif hepatit + + + ft tft
Doğal bağışıklık + + + N

Aşıyla bağışıklık + N
16· 44 yaşında erkek hastanın serolojisi aşağıdaki gibidir
HbsAg:
AntiHbs: +
AntiHbc lg M
AntiHbc lg G +
HbeAg:
HBVDNA:
Bu hasta için en doğru yorum hangisidir?
A. Akut enfeksiyon
B. Kronik aktif hepatit
C. Kronik aktif hepatit prekor mutand suş
D. Doğal bağışıklık
E. Aşıyla bağışıklık

17· 44 yaşında erkek hastanın serolojisi aşağıdaki gibidir


HbsAg: +
AntiHbs:
AntiHbc lg M
AntiHbc lg G +
HbeAg:
HBV ONA:
Bu hasta için en doğru yorum hangisidir?
A. Akut enfeksiyon B. Kronik aktif hepatit
C. Sağlıklı taşıyıcı D. Doğal bağışıklık
E. Aşıyla bağışıklık

18· 44 yaşında erkek hastanın serolojisi aşağıdaki gibidir


HbsAg:
AntiHbs: +
AntiHbc lg M
AntiHbc lg G
HbeAg:
HBV ONA:
Bu hasta için en doğru yorum hangisidir?
A. Akut enfeksiyon B. Kronik aktif hepatit
C. Aşıyla bağışıklık D. Doğal bağışıklık
E. Kronik aktif hepatit prekor mutand suş

19· 44 yaşında erkek hasta nın serolojisi aşa ğıd a ki gibidir


HbsAg :
AntiHbs:
AntiHbc lg M +
AntiHbc lg G
HbeAg:
HBV ONA:
Bu hasta için en doğru yorum hangisidir?
A. Akut enfeksiyon (pencere dönemi) B. Kronik aktif hepatit
C. Aşıyla bağışıklık D. Doğal bağı ş ıklık
E. Kronik aktif hepatit prekor mutand suş
l
~
a aKa emısı
,~~sı:~~~~~,"'11
t·•J) d · " ...

20- Akut viral hepatit Benfeksiyonu geçirerek, bağ ı şıklık Sarılık ~'- ~
kazanm ı ş bir hastanın serolojik grafiği aşağıdaki gibidir. tALTL-.-_ ____,

Ti tre
,,, Y/
HBeAg 1 11==1 tJ
Anti-HBe
tgG Anti-HBc
----- L_
X
HBsAg
----11--- t_
Arıti-HBs

O 1 2 3 4 5 6 12
Temas sonras ı süre {Ay )
lrftafltıala r
-r-----,--,---.-----.--,----ı----r-r--r---t 1---ıı--ıı ı--ı-
Bu grafikte Zile belirtilen eğri aşağıdakilerden hangisinin O 4 8 12 16 20 24 28 32 36 52 100
düzeyini gösterir?
A. HBsAg
B. Total anti-HBc
C. Anti-HBc lg M
D. Anti-HBs
E. Anti-HBc lg G

21· Bir kişiye hepatit aşısı yapılıp yapılmayacağını an lamak için O


tek bir test isteme hakkınız olsa aşağıdaki belirteçlerden ~
hangisine bakmak isterdin iz?
A. HBs antijeni
B. HBe antijeni
C. Anti-HBc lg Gantikoru
D. Anti-HBe antikoru
E. Anti-HBc lgM antikoru

22· 54 yaşındaki erkek hastada bakılan seroloji


Anti HCV: +
HCV RNA: 1.000.000/kopya
anti Hbs: +
HBV DNA: Negatif
anti HAV lg G +
ALT: 856 IU/L geliyor. En uygun tanım hangisidir?
A. Akut hepatit Cenfeksiyonu
B. Kronik hepatit Cenfeksiyonu
C. Kronik HBV enfeksiyonu
D. Akut hepatit Aenfeksiyonu
E. Akut hepatit A+ Kronik hepatit C

23- HCVenfeksiyonunda anti-HCV pozitifliği ne demektir?


Hepatit C enfeksiyonu
A. Virüs replikasyonunu gösterir
B. Bağ ışıklığın göstergesidir Erken evre akut
Akut C hepatiti Kron ik C hepatiti
Geçiri lmi ş
enfeksiyon enfeksiyon
C. Akut aktif Chepatitin in göstergesidir
D. Kronik aktif Chepatitin in göstergesidir AST/ ALT 1' 1' AST/ ALT 1' 1' AST/ ALT 1' AST/ ALT B

E. Virüsl e karşılaşıldığının göstergesid ir Ant i HCV: (-)


HCV RNA (+)
Anti HCV: (+)
HCV RNA (+)
Anti HCV (+)
HCV RNA (+)
Anti HCV: (+)
HCV RNA (-)
24· 51 yaşında bilinen hepatit Btanılı hastanın serolojisi aşağı­ HbsAg{+)
daki gibi geliyor.
Aııti HBc l gı M(-)
HbsAg: (+)
AntiHbs: (- ) Anti HEk lg 6 (+}ı
Anti Hbc lg M: (- ) Antrı-H DV 1~ M(+)
Anti Hbc lg G: (+)
Anti HDV lg M: (+ )
Bu hasta için en doğru yorum hangisidir?
A. Kronik hepatit Benfeksiyonu
B. Delta ko-enfeksiyonu
C. Delta süper enfeksiyonu
D. Akut hepatit Dvirüsü
E. Akut hepatit Benfeksiyonu

25· 51 yaşında bilinen hepatit Btanılı hastanın serolojisi aşağı ­


daki gibi geliyor. Hbs:Ag·(-t
HbsAg: (+) Arııti Hı Bc lg: Ml {+)
AntiHbs: (- ) Aııt i HBc l gı G{·)
Anti Hbc lg M: (+) Anıti- HDV lgı M (-t)
Anti Hbc lg G: (-)
Anti HDV lg M: . (+)
Bu hasta için en doğru yorum hangisidir?
A. Kronik hepatit Benfeksiyonu
B. Delta ko-enfeksiyonu
C. Delta süper enfeksiyonu
D. Akut hepatit Dvirüsü
E. Akut hepatit Benfeksiyonu

26· 34 yaşındaki kadın hasta acil servise bilinç bulanıklığı şika­


Hepatik - -- -. ~
yetiyle getiriliyor. Öyküsünden bilinen ek bir hastalığı olmadığı ensefalopati '\ _l_ Hiperamonyemiye
bağlı fötor hepaticus
öğrenilen hastanın annesinden al ın an anamneze göre evde bir
kutu ilaç içtiği öğreniliyor. Fizik muayende dorsofleksiyondaki
elin bırakılınca düşüp tekrar ayn ı pozisyona geld i ği görülüyor.
Bakılan laboratuvarda açlık plazma glukozu 40 mg/dlgeliyor
skl eralard~ı::ı~~ - ·!
ve INR 2,8 (yüksek) hesaplanıyor. Bu hastada en olası tanı ,,-ı,r · / : ,"-'. . -:-- eaım,,earem / ""'""lioma
aşağ ıd akilerden hangisidir?
Karaciğerde
Konfluent nekroz
\ ' 1 \
ı ' 1
Hipogonadizm

A. Wilson hastalığı 1 1 \ '


- - + Peteş l / Purpura
B. Galaktokinaz eksik li ği \ !
C. Hemokromatozis )
ı;
/

1 1

D. Akut hepatik yetmezlik


E. Septik şok sendromu

27• Vakadaki hasta için en değerli prognostik gösterge


aşağıdaki lerd en hangisidir?
A. INR uzaması
B. Hipogliseminin varlığı
C. Flapping tremor gelişmiş olması
D. Beyin ödemi varlığı
E. Etiyolojide ilaç kullanımının olması
..
la!!!!.l aKad
~
tı,:? . .
emısı

Prog nozu belirlemekve transpla nt en di kasyon l arı belirlemek için King's College kriterleri
28-Aşağıdakilerden hangisi asetaminofen ilişkili karaciğer
Asetaminofen ilişkili aku t karaciğer yet mezliği
hasarında nakil ihtiyacını belirleyen King's Collage kriterleri Ensefalopatiden bağ ım sız arteriyel pH < 7.3 veya aşağ ıdakilerin va rl ığı
arasında yer almaz? • PTZ > 100 sn (INR > 6,5)
• Kreatinin > 3,4 mg/dl
A.Amonyak • Evre 3 veya 4 ensefalopati
B. Ensefa lopati Asetaminofen ilişkisiz akut ka raciğer yetmezliğ i
C. Protrombin zamanı Ensefalopatiden bağımsız INR> 6,5 (PTZ > 100 sn)veya aşağ ıdakil e rin 3'ünün varlığı
• Yaş< i Oveya> 40
D. Kan gazında asidoz olması • INR>3,S(PTl>50sn)
• Serum bilirubin > 17,5
E. Kreatinin yüksekliği • Sarılık ile ensefalopati a rasrrıda 7 günden fazla süre olma sı
• ~!on·A / B/ C hepatiti İl aç ili şkili ya da kriptojenik nedenler

29- Granülom oluşumuna bağlı hepatotoksisite yapmayan İl aç ili ş kili ka raciğer ha sa rı

ilaç aşağıdakilerden hangisinde doğru olarak verilmiştir?


A. Allopurinol Granü lom yapanlar
Kron ik hepatit
yapanla r
Fibrozisle
seyredenler
Kolestatik hepatit
yapan lar

B. Fenitoin
C. Diltiazem Allopürinol
Kinidin
Metil dopa Kontraseptifler
İ zo ni az i d Metotreksat Erltromisin
D. Karbamazepin Diltiazem
Fenitoin
Fe nitoin
Ketokon azol
Avitaminl
Arsen ik
Statinler
Anabolik steroid ler
Karbamazepin
E. Metil dopa Sülfonamid
Sülfon amid TPN uygulama

30- Hepatik fibröz yaparak uzu n dönemde karaciğer sirozu


riskini artıran ilaç aşağıdakilerden hangisidir?
A. Eritromisin
B. Metil dopa
C. Metotreksat
D. Valproik asit
E. Rosuvastati n

31- 24 yaşındaki kadın hastada 8 aydır halsizlik, iştahsızlık,


kaşıntı ve amenore yakınmasıyla başvuruyor. Fizik muayenede
yüzünde akneleri dikkati çekiyor ve kendisi de son zamanda
,,,, Bakteri.. 'lirüs ve toksinlere Akne

vücüdunda kıllanma arıtşı oldu ğu nu ifade ediyor. Laboratuvar baj(ı otoimınün L etı.ıderıme

in celemelerinde serum ALT: 2100 U/L, AST: 120 U/L, alkalen Spider nevüs

fosfataz : 270 U/L, GGT: 160 U/L, total bilirübin: 1,4 mg/dL, total OtoinvnGn hepatft

protein 11 mg/dl, albumin 3.2 mg/dl ve globulin 7,6 mg/dl


sapta nmı şt ır. Hastada anti-HAV-lgG : (+ )ı anti-HBs: (+ )ı anti-HB- Tip 1
c- lgG: (+ )ı anti-HCV: ( - )ı ANA (anti-nükleer antikor): 1/320 (+ )ı En sık izl enen tiptir Genç kadın
SMA (smoot musculer antikor) 1/32 (+)AMA (anti-mitokondri- anti-S MA /ANA/ p-ANCA (+ )
yal antikor):(-) bulunmuştur. Yapılan abdomen ultrasonografi-
HLA DR 3 / 4 il e ilişkili
sinde şe kild e ki gibi safra kesesinde taş l ar görülmüş ancak safra
Aktine kaşı antikor gelişimi (+)
yollarında genişleme göz l enme miştir.
anti-Ll(M 1 / anti-HCV(-)
Globulin i
Tip2
Genellikl e çocuklarda izlen ir
anti -SMA /ANA/ p-ANCA (-)
HLA DR 81 / 0081 il e ili şk ili
Bu hasta için en uygun tedavi kombinasyonu aşağ ıd a kiler­ anti-LKM 1 / anti-HCV (+ )
den hangisidir? Anti-LC siklodeaminaz (+ )
A. Takrolimus + Prednizon B. Metotreksat Globulin normal
C. Ursadeoksikolik asit D. Azatiopürin
E. Pred nizon + Azatiopürin
32· 23 yaşındaki erkek hasta ilerleyici sarılık, iştahsızlık ve kilo 33· Daha önce bilinen bir hastalığı olmayan 32 yaşındaki
verme şikayetiyle başvuruyor. Fizik muayenede konjonktivaları kadın hasta progresif sarılık 1 ellerde titreme 1 el yazısında
soluk izleniyor. Yapılan göz dibi muayenesinde göz çevresinde bozulma ve bilinç bulanıklığı nedeniyle hastaneye yatırı­
şekildeki gibi bakır birikimiyle uyumlu lezyon izleniyor. Çekilen lıyor. İleri tetkik edilen hastaya Wilson tanısı konuluyor.
kranial MR'da ise şekildeki gibi izleniyor. Bu aşamadan sonra hasta için tercih edilmesi gereken ilk
tercih ilaç aşağıdakilerden hangisidir?
A. Çinko
B. Trientin + Penisilamin
C. Penisilamin
D. Tetramolibden + Çinko
E. Deferasiroks
Bak ı lan hemogramındahemoglobin 9 gr/dl ve biyokimyada
haptoglubulin, hemopeksin azalmış, serum LDH düzeyi yüksek
bulunuyor. Bu hastanın en olası tanısı aşağıdakilerden
hangisidir?
34- 24 yaşındaki erkek hastaya Wilson tanısı konuluyor ve
tedavi başlanıyor. Tedavi sonrasında hastada her iki ayakta
A. Hepatosellüler kanser ve göz altlarında şişlik gelişmesi üzerine başvuruyor.
B. Budd Chiari sendromu Bakılan laboratuvarda albumin 2.1 mg/dl ve 24 saat idrar
C. Otoimmün hepatit incelemede 4 gr/gün protein kaçağı izleniyor. Bu klinik
D. İlaç intoksikasyonu tablaya Wilson için başlanan ilaçlardan hangisi
E. Wilson hastalığı sebep olmuş olabilir?
A. Çinko
B. Trientin
C. Penisilamin
D. Tetramolibden
E. Deferasiroks

x
t ,)
Lun ulae
cerul eae
2023 MARTTUS SORU KAMPI
l g tıpd ► U
I.!!!!! ~ aKa · emısı
1
Dahili e
35- 43 yaşındaki erkek hasta, 1 yıldır giderek artan halsizlik ve 36· Hemokromatozise bağlı flebotomi yapılan vakalar hakkın ­
yorgunluk şikayetleriyle başvuruyor. Laboratuvar incelemelerin- da hangisi doğru bir yorum değildir?
de ALT 84 U/L, AST 82 U/L ve açlık kan şekeri 1650 mg/dL sapta- A. Flebotomi sıklığının temel belirleyici serum ferritin düzeyi-
nıyor. Öyküsünden 2 ay önce diyabetes mellitus tanısı aldığı ve dir
bu amaçla metformin kullandığı öğreniliyor. Hastanın,4 aydır B. Artopati ve hipogonadizm flebotomiden fayda görmez
2. ve 3.metakarpofalangeal eklemlerde ağrı ile şişlik olduğu
C. Siroz gelişen vakalarda HCC riskini azaltmak için flebotomi
ve impotans yakınması bulunduğu öğreniliyor. Çekilen eklem
s ıklığı artırılmalıdır
grafisi aşağıdaki gibi olan hastanın yapılan fizik muayenede
D. Sekonder HK vakalarında da flebotomi yapılabilir
karaciğer midklaviküler hatta kot altında 4-5 cm kadar palpe
ediliyor ve deride yaygın pigmentasyon artışı, E. Hemokromatoziste hedef serum ferritin düzeyi 100 ug/L'dir

37• Hemokromatoziste demirin kalpte birikimi sonucu en sık


görülen kardiyak tutulum şekli hangi seçenekte doğru
verişmiştir?

Bu hastanın en olası tanısı aşağıdakilerden hangisidir? A. Restriktif kardiyomiyopati


A. Hepatosellüler kanser B. Mitral kapak yetmezliği
B. Hemokromatozis C.Dilate kardiyomiyopati
C. Otoimmün hepatit D. Konjestif kalp yetmezl iği
D. Sistmeik lupus eritematosus E. Nan-septik endokardit
E. Wilson hastalığı

Karaciğerde dem ir birikimin e Hipogonadizm


bağ lı siroz -+ HCC

Ciltte dem ir birikimine


bağlı pigmentasyon - -

(
/j
{,.ı__u__) \
J)Y
,......ılı'tl~~;.-iii2-ir!,~.-rz.N.ııtn. ~,J'l,P,,.n11~1u1
L2~~~~~:2_22._2__i_s_jm~g~~~~~~~~~Elimml ıQ ~~ademisi
38- Polisitemia verayla takipli 54 yaşındaki erkek hasta ani
gelişen sağ üst kadran ağrısı ve batında yaygın asit ş ikayet iyl e
geliyor. Bakılan laboratuvarda hemoglobin 19 g/dl izleniyor ve
diğer sonuçları normal sınırlarda geliyor. Hastaya yapılan dopp-
ler USG'de ise şekildeki hepati k vende kan akımının olmadığı
ve kaudat lobda hipertrofi olduğu rapor ed iliyor.

Bu hasta için en olası tanı aşağıdakilerden hangisidir?


A. Budd-Chiari sendromu
B. Sağ kalp yetm ez liği
C. Splenik ven trombozu
D. Alkole bağlı karaciğer sirozu
E. Veno-oklüziy hastalık

ıJ
39- Kronik hepatit Cvirüsüne bağlı karaciğer sirozuyla takip edi- Q_~ Va ri s takibi

len hasta hematemez şikayetiyle acil servise başvuruyor. Öykü-


sünden 5 ay önce yapılan endoskopisinde her kolonda grade 4 l
varis olduğu öğreniliyor. Acil serviste yapılan değerlen dirilmede Va ri s yoksa 3 yıld a bir
endoskopiyle takip
Küçük varis varsa 2
yılda bir endoskopi
Büyük vari ste b- bloke r
ve band ligasyon

tansiyonu 60/40 mmHg, kalp atım sayısı 108 atım/dk, solunum


hızı 32 /dakika izleniyor. Yapılan fizik muayenede dalağın orta
hatt ı geçtiği, inguinal bölgeye kadar uzandığı ve göz a ltın da
şekildeki gibi basmakla solan kırmızı lekeler görülüyor.

Muayenin tüm aşama l arında aktif kanamas ı devam eden bu


hasta için en doğru yorum aşağıdakilerden hangisidir?
A. Hasta takip ed ilmeli ve kanama durarsa taburcu ed ilmeli
B. Olası pıulmoner patolojiler için toraks BT çekilmeli
C. Alkol a lı mı sorgu l anma lı ve ön planda mallory weis düşünü l ­
meli
D. Hematemezin en olası sebebi peptik ülserdir
E. Hastada özefagus varis kanaması vard ır

40- Kronik karaciğer yetmez li ği nedeniyle takip li 56 yaşındaki


erkek hasta nefes darlığı şikayetiyle başvuruyor. Ba kıl an parmak
ucu saturasyonu %87 geliyo r. FM'de siyonoze görünen hastanın
ayağa kalkınca nefes darlığı derinleşiyor. Bu hastada sirozun
hangi komplikasyonu gelişmişt ir?
A. Hepatik ensefa lopati B. Skleroderma pul. kriz
C. Hepatopulmoner send D. Hipoglisemi
E. Usual interstisyel pnömoni
41· Hemokromatozise bağlı karaciğeri sirotik bir hasta genel
durumunun bozukluğu, ateş ve karın ağrısı nedeniyle hasta-
neye yatırılıyor. Asit sıvısında polimorf nüveli lökositlerin sayısı
850/mm 3 olarak bulunuyor. Yapılan fizik muayenede batında
rebound/defans izlenmiyor. Asit sıvısı kültüründe Escherichia
cali ürüyor. Hastaya seftriakson tedavisi başlanıyor ve antibi-
yoterapinin 3.gününde yapılan parasentezde polimorf nüveli
lökositlerin sayısı 45/mm 3 geliyor. Takipte ateşi ve akut faz yanıtı
gerileyen hastanın batın muayenesi de artık rahat izleniyor. He-
kim hastayı taburcu ederken ileri evre karaciğer yetmez liği olan
ve SBP geçiren bu hastada ömür ya da nakle kadar profilaktik
antibiyotik verilmesi gerektiği düşünüyor. Çok doğru düşünen
hekimin profilaktik olarak hangi antibiyotiği reçete etmesi
en uygundur?
A. Daptomisin B. Sefotaksim
C. Meropenem D. Norfloksasin
E. Trimetoprim su lfametaksazo l

42· 30 yaşındaki kadın hasta uzun zamandan beridir devam


eden sağ üst kadran ağrısıyla başvuruyor. Anamnezinde 6 yıldır
oral kontraseptif kullandığı ve yakın zamanda gebelik planla-
dığı öğreniliyor. Hastanın batın tomografisinde 9 cm çaplı ve
düzgün s ınırlı kitle saptanıyor.

Bakı l an değerlerinde serum a-fetoprotein düzeyi normal sınırlar


içinde bulunuyor. Yapılan biyopsisinde ise partal alanların
seçilemed iği ve vaskülarizasyon artışı izleniyor. Bu hasta için en
olası tanı aşağıdakilerden hangisidir?
A. Hepatoselüler karsinom B. Fibrolameller karsinom
C. Hepatoblastom D. Fokal nodüler hiperplazi
E. Hepatoselüler adenom

43· Hepatit Cenfeksiyonuna sekonder karaciğer sirozu ile


izlenen 65 yaşında bir hastada karaciğer sağ lobda kitle ve
kanda yüksek alfa-fetoprotein düzeyi saptanmıştır. Çekil en BT'de
şekildeki gibi arteryel fazda erken kontrast tutu lumu, geç faz la
kontrasttan temizlenme (wash aut) görünümü izleniyor.

Bu hasta için en olası tanı aşağ ıd ak il erden hangisidir?


A. Hepatoselüler adenom B. Hepatoselüler karsinom
C. Metastatik kolon karsinomu D. Kolanjiosellüler kanser
E. Foka! nodüler hiperplazi
44- Hepatosellüler kanser tanısı konulan hastada bakılan
değerlerinde serum bilirubin düzeyi 1.4 mg/dl, albumin 3 mg/
dl, INR 1,2 izleniyor. Hastaya yapılan fizik muayenede batında
asit izleniyor ve daha önce bir kez ensefalopatik atak geçirdiği
öğreniliyor. Bu hastada tedavi için en uygun cerrahi dışı
seçenek aşağıdakilerden hangisidir?
A. Sorafenib
B. Sistemik kemoterapi
C. Radyoembolizasyon
D. Radyofrekans ablasyon
E. Cerrahi+ Kemoterapi

45- Kronik HCV tanılı ve buna yönelik anti-viral tedavi a ltında


izlenen hastaya yapılan ultrasonografi karaciğer sağ lobda 2
-
E
cm çapında nodül izleniyor. Çekilen BT'de arteryel fazda erken
kontrast tutulumu, geç fazda kontrasttan temizlenme (wash
out) görünümü izleniyor. Hepatosellüler kanser tanısı konulan
hastada bakılan değerlerinde AST 34 IU/L, ALT 41 IU/L, serum
bilirubin düzeyi 1.3 mg/dl, albumin 3,9 mg/dl, INR 1, 1 izleni-
yor. Hastaya yapılan fizik muayenede batında asit izlenmiyor
ve daha önce ensefa lopati, hepatorenal sendrom veya varis
kanamas ı da yaşamadığı öğreniliyor. Bu hasta için en uygun
tedavi aşağıdakilerden hangisidir?
A. Karaciğer transplantasyonu
B. Cerrahi rezeksiyon
C. Radyoembolizasyon
D. Radyofrekans ablasyon
E. Sistemik Kemoterapi

46· Vakadaki hastaya cerrah i rezeksiyon yapılıyor. Sonrasında


takipte yapılan kontrol doppler USG'de partal ven basıncı 18
mmHg izleniyor. Ayrıca cerrahi rezeksiyondan bir yıl sonra aci le
hematemez şikayetiyle başvurduğu öğreniliyor. Fizik muayene-
sinde skleraları ikterik izlenen hasta için bu aşamadan sonra en
uygun tedavi aşağıdakilerden hangisidir?
A. Karaciğer transplantasyonu
B. Cerrahi rezeksiyon
C. Radyoembolizasyon
D. Radyofrekans ab lasyon
E. Sistemik Kemoterapi

..
l g tı~d . .
l!!!! ~ aKa emısı

47- Gebeliğinin 34. haftasında olan kadın hasta bilinç bulanık­


lığı şikayetiyle acile getiriliyor. Fizik muayenesinde skleraları
ikterik izlenen hastanın bakılan laboratuvarında transaminaz
yüksekliği, direkt hakimiyetinde hiperbilirubineminin varlığı
ve INR uzaması dikkati çekiliyor. Bilinci bulanık olan hastanın
bakılan parmak ucu glukoz düzeyi 45 mg/dL ölçülüyor. Bu hasta
için en olası tanı aşağıdakilerden hangisidir?
A. Gebeliği akut yağlı karaciğer hastalığı
B. Gebeliğin intrahepatik kolestazı
C. HELLP sendromu
D. Akut vira! hepatit
E. Koledokolitiyazis

48- Vakadaki hasta için bu aşamadan sonra en uygun tedavi


aşağ ıdakilerden hangisidir?
A. Hepatik yetmezliğe yönelik N-asetil sistein başlamak
B. Maternal deksametazon uygulama
C. Fetal maturasyon tamamlanınca doğum
D. Normal doğumun indüklenmesi
E. Doğumun sezaryenle gerçekleştirilmesi

49- Yetmiş altı yaşındaki kadın hasta; sağ üst kadran ağrıs ı,
Du-::ıus~p.ı:linıs
halsizlik ve titremeyle yükselen ateş nedeniyle acil servise
-
skıi'\ter

getiriliyor. Öyküsünde hastanın diabeti olduğu ancak düzen li


kontrollerini yapt ırm ad ığı öğreniliyor. Fizik muayenede "Murp-
hy" bulgusu pozitif saptanıyor. Laboratuvar tetkiklerinde lökosit: Vesicabifi.ıris .
ınrundibofııın
- ~ " , \ ,'\

28.000/mm3, total biluribin: 2, 9 mg/dL, AST: 54 U/L, ALT: 101 - Dmıdcnum.


l'JfSlilllXırior

U/L, ALP: 167 U/L ve GGT: 89 U/L bulunuyor. Ultrasonografide


safra yolları normal, safra kesesi duvar kalınlığı 15 mm ölçülü-
Ouctus
yor. Abdominal bilgisayarlı tomografide safra kesesi duvarında dıolcdodıu,

gaz izleniyor ve görüntüsü aşağıdaki gibidir. \/esic~


bili,3ris.
rondus

Pdpitlo ,,,,.,./
dund r-rıi minor

P,r,,lla -----
duodeni mJjor

Otmderıım.

Bu hasta için en olası tanı aşağıdakilerden hangisidir?


A. Kolelitiazis B. Akut pankreatit
C. Amfizematöz kolesistit D. Koledokolitiazis
E. Safra taşı ileusu
50· 54 yaşında erkek hasta sağ üst kadran ağrısı nedeniyle acil Sistik kanal ~
servise başvuruyor. Öyküsünden 4 ay önce kolelitiyazis tanısı fürer
aldığı öğrenilen hastanın bakılan laboratuvarında AST:87 IU/L, Safra Kesesi ----.......

ALT:98 IU/L, ALP:234 IU/L, GGT:865 IU/L, Total bilirubin 9 mg/dl, - Ductus hepaticus communis

direkt bilirubin 7 mg/dl geliyor. Fizik muayenede ateş izlenme- Duktus koledokus ~ (

yen hastada ağrısına yönelik çekilen USG'de hem intrahepatik


hem de ekstrahepatik safra yollarında dilatasyon olduğu; kole- ~
Akseuar - - - - - - - - -
;
Neci< Body
Tail

dok çapının 9 mm olduğu rapor ediliyor. Bu hastanın en olası " - Pankreas

tanısı aşağıdakilerden hangisidir? :~:::::::,;"'' __,/~~


A. Kolelitiazis B. Akut pa nkreatit Ampulla vateri

Ana pankreatik kanal


'l / /
C. Amfizematöz kolesistit D. Koledokolitiazis Duodenum

E. Safra taşı ileusu

51- 38 yaşında kadın hasta sağ üst kadran ağrısı, 38°( ateş ve
skleralarda ikter nedeniyle acil servise başvuruyor. Öyküsünden
8 ay önce kolelitiyazis tanısı aldığı öğrenilen hastanın bakılan
laboratuvarında AST:87 IU/L, ALT:98 IU/L, ALP:234 IU/L, GGT:865
IU/L, Total bilirubin 9 mg/dl, direkt bilirubin 7 mg/dlgeliyor.
Hastaya yapılan USG'de safra yollarının genişlediği izleniyor. Bu
hastanın en olası tan sı aşağıdakilerden hangisidir?
A. Kronik pankreatit /
B. Kolanjiosellüler kanser
C. Akut taşlı kolesistit
D. Koledokolitiazis
1.uııw
E. Akut kolanjit cyı?dranı;
~~lii-lMI ır,tin

52- 34 yaşında alkol kullanım öyküsü olan erkek hasta karın


ağrısı şikayetiyle başvuruyor. Yapılan fi zik muayenede tüm
batında hassasiyet izleniyor. Laboratuvarda AST:78 IU/L, ALT:34
IU/L, ALP:70 IU/L, GGT:11 OIU/L, Total bilirubin :1.2 mg/dl, direkt
bilirubin:0,5 mg/dl, Amilaz:445 IU/L, Lipaz:674 IU/L izleniyor.
Hastaya yapılan abdominal USG'de şekildeki gibi pankreas
hipoekoik, ödemli ve etrafında sıvı koleksiyonu izleniyor.

Bu hastanın en olası tan sı aşağıdakilerden hangisidir?


A. Akut pankreatit B. Kolanjiosellüler kanser
C. Akut taşlı kolesistit D. Koledokolitiazis

..
E. Akut kolanjit
53- Akut pankreatit tedavisi hakkında hangisi yanlıştır?
A. Tedavinin kilit basamağı i.v hidrasyondur
B. CRP ve sedim yüksekliği olan vakalara antibiyotik başlanır
C. Pankreatite bağlı nekrotik koleksiyon varlığı EUS gerektirir
D. Bilier sebepli pankreatitlerde meropenem verilebilir
E. Pankreasta nekroz derecesi antibiyotik başlama kararını
etkilemez

54- 44 yaşındaki kadın hasta son 1 yılda 3 kez tekrarlayan pank-


reatit atağı ve nedeniyle tetkik edilmek üzere servise yatırılıyor.
Yapılan ultrasonografide pankreas normalden biraz büyük
izleniyor. Pankreasta kitle tarif edilmiyor; safra kesesi ve safra
yolları olağan, koledok çapı da normal sınırlarda rapor ediliyor;
yine pankreasta kalisifikasyon ve atrofi izlenmiyor. Etiyolojiye
yönelik sorgulamada hastanın alkol kullanmadığı ve bilinen ek
bir hastalığının olmadığı öğreniliyor. Yapılan ERCP'de şekildeki
gibi ana pankreatik kanalda düzensiz darlıklar ve genişlemeler
dikkati çekiyor.

Bu hasta için en olası tanı aşağıdakilerden hangisidir?


A. Pankreas kanseri B. Kronik pankreatit
C. İntraduktal musinöz neoplazi D. Otoimmün pankreatit
E. Müsinöz kist adenom
NE.:t=ROLO~İ
1- 34 yaşında erkek hasta dahiliye polikliniğine idrar yaparken
yanma şikayetiyle geliyor. Dipstick yöntemiyle bakılan idrar
analizinde (+++)eritrosit tespit ediliyor. Bu aşamadan sonra
ilk yapılması gereken hangisidir?
A. Abdomino-pelvik USG
B. İdrar mikroskopisi
C. Batın bilgisayarlı tomografisi
D. İdrar sitolojisi
E. Ampirik idrar yolu enfeksiyonu tedavisi başlamak

2- 34 yaşında erkek hasta dahiliye polikliniğine idrar yaparken


yanma şikayetiyle geliyor. Dipstick yöntemiyle bakılan idrar
analizinde (+++)eritrosit tespit ediliyor. Bu aşamadan sonra --
bakılan idrar mikroskopisinde her büyütme alanında 18 eritrosit
görülüyor ve tekrarlanan incelemelerde de benzer sonuçlar elde
ediliyor.
Bu hastada en olası tanı hangisidir?
A. İdrar yolu enfeksiyonu
B. Membranöz glomerülonefrit
C. lg Anefropatisi
D. Nefrolitiyazis
E. Benign prostat hiperplazisi

3- 38 yaşında erkek hasta kanlı idrar şikayetiyle başvuruyor. 4-Vakadaki hastanın aşağıdaki anti-neoplastik ilaçlardan
Detaylı alınan anamnezde günde 2 paket sigara içtiği, idrarını hangisi kullanmış olması bu duruma yol açmış olabilir?
yaparken kanın pıhtı şeklinde geldiği ve sık sık idrara çıkma iste- A. Doksorubisin
ği olduğu öğreniliyor. Bakılan dipstick ve idrar mikroskopisinde B. Dakarbazin
hematüri varlığı doğrulanan hastanın 24 saatlik idrar analizinde C. Bleomisin
0,05 g/gün proteinüri izleniyor. Çekilen batın BT'nin normal
D. Busulfan
sınırlarda olduğu rapor ediliyor.
E. Siklofosfamid
Bu aşamadan sonra ilk yapılması gereken hangisidir?
A- Batın BT çekimini tekrarlamak
B- İdrar mikroskopisi
C- Batın bilgisayarlı tomografisi
D- İdrar sitolojisi
E-Ampirik idrar yolu enfeksiyonu tedavisi başlamak

Non-glomerül er hematüri

İYE(+) İYE( - )

! ! > 40
yüksek
yaş veya
şüphe
- Sitoloji / Sistoskopi
İYE t edavi si BT

Kit le yada Negatif


lezyon

< 40 yaş veya


Tedavi İdrar sitolojisi
düşük şüphe

(+) ➔ Sistoskopi ( -) ➔ Takip


l g tıgd ► ~
l!!!I! ~ aKa 1
emısı

5- Aşağıdakilerden hangisi diabetik nefropatide biyopsi endi- Endikasyonlar Kontrendikasyonlar


kasyonu değildir? Hızl ı ilerleyen GN Multiple kist (basit/ ODPK)
A. İdrar sedimentinde hematüri ve eritrosit silendirleri varlığı Nefrotik sendrom Soliter böbrek
B. Tanıdan sonraki 3. yılda proteinüri izlenmesi > 1 gr/ gün proteinüri Akut pyelonefrit
C. Retinopati ve nöropati olmadan böbrek yetmezliği gelişmesi Açıklanamayan KBY Perinefritik apse
D. Akut başlangıçlı böbrek hastalığı Ailesel böbrek hastalığı Böbrekte solid kitle
E. Tanı aşamasında renal tutulum olup/olmadığını anlamak Rejeksiyon tanısı Kanama diyatezi
Etyolojisi belirsiz ABY Kontrolsüz HT
Sistemik hastalık tutulumu Non-koopere hasta
İzole mikroskopik hematüri Üremi/ Obezite (rölatif k.e)

6- 88 yaşında erkek hasta beslenememe, oral alım bozukluğu, Prerenal ABH RenalABH
ishal, ateş ve şuur bulanıklığı nedeniyle acil servise getiriliyor. İdrar dansiitesi > 10'20 < 1010
Yapılan fizik muayenede kan basıncı 70/65 mmHg, oral mukoza idrar asmofaritesi >50OmOsm/L < 350 müsm/L
kuru olarak saptanıyor ve boyun venleri palpe edilemiyor. Labo- idrar sodyumu < 20 mEq/L > 40 mEq/L
ratuvar incelemelerinde serum sodyum düzeyi 168 mEq/L, kan İdrar / plazma kreatinin > 40, < 20

üre azotu (BUN) 11 Omg/dl ve serum kreatinin düzeyi 2,8 mg/ BUNI t Kreatinin > 20 < 10

dl olarak bulunuyor. Aşağıdaki laboratuvar sonuçlarından han- Fraksiyone sodyum <%1 >%1

gisi bu hastada görülmesi beklenen bir sonuç deği dir? Renal faillJre indeks <%1 > %1
fraksiyone Üre <%35 >%35
A. İdrar kreatinin/plazma kreatinin > 40
P-re-renal / Renal ABH ayrımrncfa en değerH pa,rametre FeNa atılımıdır
B. İdrar osmolalitesi > 500 müsm/L Ancak idrar çıkışı yoksa en değerl i parametre BUN/ Kreatin in
C. Fraksiyone sodyum atılımı > %3
D. İdrar üre/plazma üre> 8
E. İdrar sodyum miktarı< 20 müsm/L

7- Lenfoma tanısı konulan ve 5 gün önce hafta önce ABVD Tümör Lizis Sendromu
tedavisi başlanan 62 yaşında bir erkek hastad a idrar miktarında
azalma ve ödem şikayeti gelişmesi üzerine yapılan laboratuvar ' / ÜrlkAsit
___.. Potasyum
incelemelerinde kan üre azotu 40 mg/dl, kan kreatinin düzeyi
-.... Fosfat
4.9 mg/dL, serum ürik asit düzeyi 24 mg/dl olarak saptanıyor.
Yapılan renal USG'de böbrek boyutları olağan izleniyor. Bu has- Ksantin

ta için bu aşamada en çok dikkat edilmesi gereken eletro lit l Oksidaz


HiperFosfatemf H'iperÜrl eml HfperKalemi
imbalansı hangisidir?
A. Hiperkalemi idrar Fosfat1' İ drar ürik Asit 1'
B. Hipokaelmi
C. Hiperkalsemi Ürik asit Taşı

--
Ca~P Taşı
D. Hiponatremi
E. Hiperfosfatemi tı-ıipo Kalremi Akut Böbrek Yetmezliği

8- Aşağıdakiler
hangisi akut böbrek yetmez liğind e kullanılan
güncel belirteçleren biri değildir?
A. Kidney injury molecule-1
B. Neutrophil gelatinase associated lipocalin
C. İnsulin like growth factor binding protein 7
D. Metalloproteinaz 2
E. Metalloproteinaz 9

..
9. 78 yaşında bir kadın hasta sepsis ve oligürik böbrek yetmez-
liği nedeniyle izleniyor. Fizik muayenede kan basıncı 160/90
mmHg, nabız 113/dakika, solunum 24/dakika, ateş 39, 1 °C, her
iki akciğer bazallerinde ince raller ve üst ekstremitelerde kanat
çırpar tarzda (flapping) tremor saptanıyor. Laboratuvar tetkik-
lerinde serum kreatinin düzeyi 4.1 mg/dl, serum potasyum
düzeyi 7.8 mEq/L ve serum bikarbonat düzeyi 21 mEq/L olarak
bulunuyor. Bu hastada hiperkalemi yönetimi hakkında
hangi yorum doğru deği!dir?
A. İlk yapılması gereken kardiyak stabilizasyon sağlamak adına
kalsiyum glukonat vermektir
B. Potasyumun hücre içine girişini artıran tedaviler insülin, bikar-
bonat ve beta agonist tedavisidir
C. İnsülin saniyeler içinde potasyumu hücre içine soktuğu için en
etkin hiperkalemi tedavisidir
D. Patiromer ve zirkonyum oral kullanılan potasyum bağlayıcı
ajanlardır akut tedavide yeri yoktur
E. Hastaya tamponize mai (dextroz + insülin) başlandıktan sonra
yanıt alınamazsa ya da EKG'de Tsivriliği devam ederse diyaliz
düşünülmelidir

10. Aşağıda verilen akut böbrek yetmezliği kliniklerinden han-


gisinde USG bulgusu postrenal akut yetmezlik lehin dir?
A. Pseudomonas sepsisi sonrası puroura fulminans gelişen 45
yaşındaki kadın hasta
B. Pulmoner emboli kliniğiyle gelen hastaya kontrastlı pulmo-
ner angiografi çekilen 18 yaşındaki kadın hasta
C. Servikal kanser sebebiyle kemoradyoterapi alan 54 yaşındaki
kadın hasta
D. Tiklodipin kullanım an bağlı trombotik trombositopenik
purpura gelişen 53 yaşındaki erkek hasta
E. Naproksen sodyum kullanımına bağlı idrar miktarında azalma
şikayetiyle gelen 45 yaşındaki erkek hasta

11- Kronik böbrek yetmez liğinde ilk bulgusu hangisidir?


A. Anüri
B. Poliüri
C. Oligüri
D. Noktüri
E. Pollaküri

12- Kronik böbrek yetmezliği olmasına rağmen böbrek bo-


yutlarında azalma izlen eyen hastalıklardan biri değ ildir?
A. Sistemik lupus eritematosus
B.Amiloid
C. Otozomal dominant polikistik böbrek
D. Hidronefroz
E. Diyabet
2023 MARTTUS SORU KAMPI
l g tı~d . ,. Dahili e
~ !!!!!I aKa :emısı

13· LDL kolesterolü ve vasküler kalsifikasyonu azaltarak kardi-


yoprotektif etkili fosfor bağlayıcı ajan hangisidir?
A) Sevelamer hidroklorür
B) Magnezyum karbonat
C) Lantanum karbonat
D) Sodyum bikarbonat
E) Kalsiyum karbonat

14- K~onik böbrek yetmezliğinde aşağıdakilerden hangi hormo-


nun düzeyi normalden düşük beklenir?
A. Prolaktin
B. Glukagon
C. Gastrin
D. İnsülin
E. Dvitamini

15- Akciğer kanseri sebebiyle takipli 56 yaşındaki erkek has-


tanın laboratuvar incelemesinde serum sodyumu 124 mEq/L,
idrar osmolaritesi 120 mOsm/kg, serum osmolaritesi 262
mOsm/kg, idrar sodyumu 58 mEq/L izleniyor. Bu hastanın en
olası tanısı hangisidir.
A. Primer hipotiroidi
B. Diabetes insipidus
C. Uygunsuz ADH sendromu
D. Nefrotik sendrom
E. Kardiyak siroz

16- 24 yaşında erkek hasta kilo kaybı, halsizlik ve ateş ş ikayetiy­


le geliyo r. Yapılan in celemelerde son 1 ayda 17 kilo kaybettiği
ve geceleri atletini değiştirecek kada r çok terlediği öğreniliyor.
Bu öykü eşliğinde hekimin aklına maligmite taraması yapma k
geliyor ve çekilen grafi serisinde her iki alt ekstrem ite kemikle-
rinde geni varum görüldüğü rapor edi liyor. Laboratuvarında D
vitamini ve fosfat düzeyinin çok düşük; serum FGF 23 düzeyinin
ise yüksek olduğu izleniyor. Bu hastada en olası tanı aşağıda­
kilerden hangisidir?
A. Familya! tümöral kalsinozis
B. Dvitamini dirençli rikest
C. Dvitamini eksikliğine bağlı osteomalazi
D. Onkojenik hipofosfatemik osteomalazi
E. Hipofosfatemik rikets
17- 24 yaşında ayaklarında şişlik şikayetiyle gelen erkek hasta-
da nefrotik sendrom düzeyin protein kaçağı izlenmesi üzerine
ren la biyopsi yapılıyor ve aşağıdaki görüntü elde ediliyor. Alınan
biyopsi materyali elektrolin mikroskopisinde incelenediğinde
ise yine aşağıdaki gibi görüntü elde ediliyor.

Bu hastanın en olası tamsı aşağıdakilerden hangisidir?


A. Fokal segmental glomerüloskleroz
B. Minimal değişiklik hastalığı
C. Membranöz nefropati
D Membranoproliferatif glomerülonefrit
E. Radi pli progressif glomerülonefrit

18- Nefrotik sendrom tanısı konulan 65 yaşında bir hastada re-


nal biyopsi yapılıyor ve membranöz glomerülonefrit saptanıyor.
Bu hasta için en uygun yaklaşım aşağıdakilerden hangisidir?
A. Yalnızca steroid tedavisi
B. Malign hastalıkların detaylı taranması
C. Steroid + siklofosfamid tedavisi
D. Antikoagülan tedavi
E. Kriyoglobulin tayini

19- 1. Mikroskopik hematüri


il. < 500 mg/gün proteinüri
111. (3 kompleman düşüklüğünün devamı
iV. Nefrotik sendrom kliniği
Yukarıdakilerden hangisi/ hangileri APSGN'de_renal biyopsi
endikasyon dur?
A.1 -- 11 B.11--111
C.111 -- iV D.1 -- 11 -- 111
E.1 --11--111-- iV

20- Aşağıdakilerden hangisi bazal membran tutulumu


yapmaz?
A. Anti-glom erüler bazal membran hastalığı
B. Nail-patella sendromu
C. İnce bazal membran hastalığı
D. Alport sendromu
E. Fanconi sendromu
ımı
l a tı~d . .
l.!!!! ~ aKa emısı

21 - Kronik hepatit Ctanısı ile izlenen bir hastada yaygın


anazarka tarzında ödem gelişiyor. Yapılan incelemelerde günlük
12 g proteinüri, serum albümin düzeyi 1,4 g/dl, total kolesterol
41 Omg/dL, serum kreatinin düzeyi 1.9 mg/dl olarak saptanıyor.
Serum C3 ve C4 düzeyleri düşük bulunuyor. Hastanın glutela,
kalça ve kol bölgesinde şekildeki gibi purpuraları izleniyor ve
yapılan biyopside ise glomerülde protein birikimleri izleniyor.

Hastada en olası tanı hangisidir?


A. Membranöz glomerülonefrit
B. Miks kriyoglobulinemi
C. Subakut bakteriyel endokardit ti)
ıı.

D. Kresentrik glomerülonefrit
E. lg Anefropati

22- Serum C3 ve C4 düşüklüğüyle birlikte nefrotik sendrom


kliniği varlığında seçeneklerdeki hangi hastalık akla gelmelidir?
A. Membranöz glomerülonefrit
B. Miks kriyoglobulinemi
C. Primer amiloidoz
D. Kresentrik glomerülonefrit
E. lg Anefropati

23- 22 yaşında erkek hasta ön kol yüzeyinde deriden ka b arı k


şiş lik ge li şmesi üzerine başvuruyor. Hastanın dermatoloji
konsultasyonunda ilgili şişliğin angiokeratomla uyumlu olduğu
rapor edi liyor. Yapılan id rar analinde polarize ı ş ık altında oval
yağ tanecikleri ve glikolipit içerikleri vakuoller görülüyor. Yapılan
biyopsi materyali eletron mikroskopisinde incelendiğinde "ze bra
cisimcilert rapor edi liyor.

Bu vakada en olası enzim eksikliği hangi seçenekte doğru


verilmiştir?
A. Glukokinaz
B. alfa-ga laktozidaz
C. Tirozinaz
D. Sfingomiyelinaz
E. Topoizomeraz 2
24- 12 yıldır tip 2 diyabetes mellitus nedeniyle takipli 45 yaşın­
daki kadın hasta nefes darlığı şikayetiyle acil servise başvuruyor.
Bakılan laboratuvarında D-dimer yüksek izlenen hastada ön
tanıda pulmoner tromboemboli düşünülüyor ve tanı amaçlı
toraks BT anjiografi çekiliyor. BT çekiminden sonra serum krea-
tinin değerinde %35'Iik artış izleniyor. İdra r sedimenti normal
sınırlarda izleniyor. Bu hastanın en olası tanısı aşağıdakiler­
den hangisidir?
A. Akut tübüler nekroz
B. Re nal arter trombozu
C. Ateroembolik renal arter embolisi
D. Kontrast nefropati
E. İskemik nefropati

25- Vakadaki hasta için en önemli risk faktörü hangisidir?


A. Eşlik eden d-dimer yüksekliği
B. Hastanın yaşı
C. Diyabetes mellitus
D. Hipertansiyon varlığı
E. Hastanın kadın olması

26- Yaygın kemik ağrısı nedeniyle başvuran bir erkek hastada Proksimal tübül Kan Distal tübül Lümen
bakılan laboratuvarda kalsiyum 12 mg/dl, albumin 1,2 mg/
dl ve serum globulin düzeyi ise 9 mg/dl geliyor. Hastada
ön planda multiple myelom düşünülüyor ve yap ılan serum
elektroforezde de monoklonal gama bandı izleniyor. Detaylı
bakılan laboratuvar ana lizinde kan gaz ınd a HC0 3: 11 mEq/L ve
hipokalemi, hiperkloremik anyon GAP normal metabolik asidoz
izleniyor. Tam idrar ana lizinde glukoz, protein pozitif ve idrar
pH'sı 5,2 tespit ed iliyor. Bu hastanın en olası tanısı aşağ ıdaki ­
lerden hangisidir? Proksimal tübül Kan Distal tübül Lümen
A. Proksimal renal tübüler asidoz ?roksimal Renal ıubüler Asidoz Distal Renal Tübüler Asidoz
B. Distal renal tübüler asidoz illıul !Tu!.ll
C. Tip 3 renal tübüler asidoz
D. Tip 4 renal tübü ler asidoz
E. Bartter's sendromu

27- Yukarıdaki hastada proksimal RTA tanısına yönelik aşağıdaki


testlerden hangisi en uygundur?
A. Potasyum fosfat infüzyonu
B. Amonyum klorür uygulama
C. Potasyum klorür infüzyon testi
D. Kemik iliği aspirasyonu
E. HC0 3 yükleme testi

..
a
l
~
tı~ ►
aKad''emısı ..

28- 1. Spironolakton il. Epleronon


111. Amilorid iV. Triamteren
V. Lizinopril VI. l<andesartan
Liddle sendromu tedavisinde uygun olmayan ilaçlar hangi
seçenekte doğru verilmiştir?
A.1--11--111
8.1--11 --111--IV
C.111-- iV
D.1--11--V--VI
E. 11-- 111-- IV -- V

29- 23 yaşında bir kadın hasta ateş, halsizlik, oligüri ve cilt


döküntüsü şikayetleriyle başvuruyor. Öyküsünden bir hafta önce
streptokokkal farenjit nedeniyle ampisilin kullanm aya başladığı
öğreniliyor. Fizik muayenede vü cut sıcaklığı 38,8 °C, nabız 98/
dakika ritmik, kan basıncı 155/95 mmHg ve gövdede morbili-
form döküntü saptanıyor. Laboratuvar incelemelerinde idrarda,
mikroskopik hematüri, lökositüri, eozinofilüri, lökosit silendiri;
kanda, antistreptolizin titresi yüksek ve eozinofili, kan üre azotu
42 mg/dL, kreatinin 2.3 mg/dlolarak bulunuyor. Bu hasta için
en olası tanı aşağıdakilerden hangisidir?
A. Akut piyelonefrit
8. Akut tübüler nekroz
C. Henoch-Schönlein purpurası
D. Akut poststreptokoksik glomerülonefrit
E. Akut interstisyel nefrit

30- Aşağıdaki ilaçlardan hangisi


akut interstisyel nefrit
yoluyla olarak akut böbrek yetmezliğine yol açab ilir?
A. Sisplatin B. Lansoprazol
C. Pentamidin D. Asetaminofen
E. Sidofovir

31- Aşağıdaki hastalıklardan


hangisi tübülo interstisyel
alanda fibrozis ve skar oluşumu ile karakterize ren al tutulum
yapmaz?
A. Sarko idoz
B. Ateroembolik hastalık
C. Aristolojik asit nefropatisi
D. Kurşun nefropatisi
E. Fenasetin kullanımı
32- Baş ağrısı ve şuur bulanıklığı nedeniyle acil servise getirilen
54 yaşında bir erkek hastada intrakraniyal kanama saptanıyor.
Hastanın öyküsünden 8 yıldır hipertansiyonu olduğu ve baba-
sının böbrek yetmezliği nedeniyle hemodiyaliz programında
olduğu öğreniliyor Fizik muayenede kan basıncı 160/95 mmHg
ve karında her iki lateral bölgede kitleler bulunuyor. Laboratuvar
incelemelerinde BUN düzeyi 42 mg/dL, serum kreatinin düzeyi
2,4 mg/dl ve idrarda mikroskopi hematüri gözleniyor. Hastaya
yapılan USG'de aşağıdaki gibi multiple kistler izleniyor.

Bu hastayala ilgili olarak aşağıdaki ifadelerden hangisi


doğrudur?
A. Hastalığın gelişiminden nefrokistin geni sorumludur
B. Genellikle 40 yaşından önce son dönem böbrek yetmez liği
gelişir
C. Hastaların tamamına yakınında ürik asit taşları
hastalığa eşlik eder
D. En sık görülen ekstrarena l bulgusu mitral kapak prolapsusu-
dur
E. Hipertansiyon, çoğunlukla böbrek yetmezliği gelişmeden
önce ortaya ç ıkar ve yaşla birlikte s ıklı ğı artar

33- Vakadaki hastanın hipertansiyonuna yönelik tercih edilmesi


gereken ilk tercih anti-hipe ansif hangisidir?
A. Asetezolamid
B. Ep leronon
C. Kaptopril
D. Amlodipin
E. Karvedilol

34- Aşağıdakilerden hangisi otozomal dominant polikistik


böbrek hastalığına eşlik eden bulgulardan biri deği dir?
A. Serebra l anevrizma
B. İnguinal hemi
C. Mitral kapak prolapsusu
D. Hepatik kistler

..
E. Ren al arter stenozu
taJ gflademisi
35· 81 yaşında bir erkek hastanın öyküsünden 6 yıl önce akut
koroner sendrom ve son bir yılda 3 kez akut akciğer ödemi atağı
geçirdiği öğreniliyor. Bu hastada son üç haftada tekrarlanan
ölçümlerde kan basıncı 180/120 mmHg civarında bulunuyor. İki
hafta önce kaptopril tedavisi başlanan hastanın kan basıncının
130/85 mmHg ye düştüğü gözleniyor. Laboratuvar incelemele-
rinde 2 hafta önceki kreatinin değeri normal olmasına rağmen
güncel serum kreatinin düzeyi 2.4 mg/dlolarak bulunuyor.
Bu hastada bu aşamadan sonra tanıya yönelik ilk istenmesi
gereken tetkik hangisidir?
A. Renal doppler ultrasonografi
B. Kaptoprilli renal sintigrafi
C. Renal anjiografi
D. İntravenöz pyelogram
E. Ayakta direk batın grafisi

36- Vakadaki hastada bu aşamadan sonra kesin tanı koymak


için istenmesi gereken tetkik hangisidir?
A. Renal doppler ultrasonografi
B. Kaptoprilli renal sintigrafi
C. Renal anjiografi
D. İntravenöz pyelogram
E. Ayakta direk batın grafisi

37- 63 yaşında erkek hasta ani başlayan karın ağrısı, ateş,


bulantı-kusma ve idrar renginde koyulaşma şikayetiyle başvu ­
ruyor. Fizik muayenesinde kostovertebral açı hassasiyeti izlenen
ve tansiyonu 180/11 OmmHg ölçü len hastanın çeki len EKG'si
aşağ ıd aki gibi ge liyor.

ili I r VJ l ıı, r~

.J,-__ 1:,_ _____..___J.I ,,-'t,-,-Ji ,_J~UJıdvı\Jr_J· I\_ \~ _,..., _ -- 1 1ı_ll


I ı f ' f "/' -1-~..,-lıi
RrlYTr~ srnır: ıı 1
2:::i ,..,./t>ec.ıl ır/,V J

~--1~. ----J."'--1r_Jr-"'-- _,_ .\--J,.~_J,_,J,_. _ 1. ,,,, '-----\~JL


Hastan ın laboratuvar incelemesinde üre 89 mg/dl, kreatini 3,2
mg/dl ve LDH 2980 IU/L izleniyor. Eş zamanlı idrar LDH düzeyi
de yüksek seyreden bu hasta için olası tanı hangisidir?
A. So l dal blok
B. Akut koroner sendrom
C. Rena l arterin embo lik oklüzyonu
D. Gastrointestinal perforasyon
E. Ateroembolik renal hastalık
38- 34 yaşındaki erkek hasta ateş yüksekliğiyle acil servise baş­
vuroyor. Öyküsünden çocukluk çağından beridir VUR nedeniyle
takipli olduğu öğreniliyor. Yapılan fizik muayenede sol yan
tarafta ağrı ve o bölgede kostovertebral açı hassasiyeti izleniyor.
Bakılan hemogramda lökosit 14.000/mm 3, hemoglobin 13 g/
dl, trombosit 17 6.000/mm 3 izleniyor. Hastaya çekilen BT'de
şekildeki gibi sol böbrekte yer kaplayan oluşum izleniyor.

Bu hasta için en olası tanı aşağıdakilerden hangisidir?


A. Renal apse
B. Amfizematöz pyelonefrit
C. Renal papiller nekroz
D. Ksantogranülomatöz nefrit •
E. Renal malakoplaki E

39- 56 yaşındaki erkek hasta ateş yüksekliğiyle acile başvuru ­


yor. Öyküsünden bilinen tip 3 diyabetes mellitus olduğu ancak
insülinlerini ve oral anti-diyabetikelerini düzenli kullanmadığı
öğreniliyor. Cilde bakılan parmak ucu kabn şekeri de 350 mg/
dl ve hemogramda lökosit 17.000/mm 3, hemog lobin 13 g/dl,
trombosit 245.000/mm 3 geliyor. Hastaya çekilen BT'de renal
paran kim içinde gaz ve hava-sıvı seviyeleri izleniyor. Bu hasta
için en olası tanı aşağıdakilerden hangisidir?
A. Renal apse
B. Amfizematöz pyelonefrit
C. Renal papiller nekroz
D. Ksantogranülomatöz nefrit
E. Renal malakoplaki

40· Rutin kontrole gelen 32 yaşında ve 12 yıldır tip 1 diabetes


mellitus hastalığı olan erkek hastaya yapılan fizik muayenede
kan basıncı 125/80 mmHg olarak bulunuyor. Öyküsünden
hastalığı süresince glisemik kontrolünün iyi seyrettiği , diyetine
uyduğu, insülinlerini zaman ında ve düzenli olarak yaptığı ve
herhangi bir diyabetik komplikasyon geçirmediğ i öğreniliyor.
Yapılan kan incelemesinde kan biyokimyasının normal olduğu
saptanıyor. Fundoskopik muayenede patoloji saptanmıyor. Bu
hastada, diyabetik nefropatiye yönelik aşağıdakilerden hangisi-
nin yapılması en uygundur?
A. Böbrek biyopsisi
B. Abdominal ultrasonografi
C. Serum kreatinin ve GFR tayini
D. Dipstisck ile protein tayini
E. Yıllık mikroalbüminüri taraması
l a tı~d ►"
~ !!!.I aKa ·emısı

ENT>OKRİNOLO~İ
1· 51 yaşında kadın hasta zaman zaman olan hafif ve dinlen-
mekle geçen gerilim tipi baş ağrısı şikayetiyle başvuruyor.
Öyküsünden 6 çocuğu olduğu ve hepsinin de işinde gücünde
olduğu öğreniliyor. Değerlendirmeyi yapan hekim bir organik
patolojiyi atlamamak adına hastaya hipofiz MR çekiyor ve aşağ ı ­
daki görüntüler elde ediliyor.

Yukarıdaki MR raporunda sella içinde hücre topluluğuna ait


sinyal alınamadığı rapor ediliyor. Bu hastada en olası tanı
aşağıdakilerden hangisidir?
A. Empty sel la sendromu
B. Kraniofarengioma
C. Sheehan sendromu
D. Hipofizer apopleksi
E. Lenfositik hipofizit

2- Yukarıdaki hasta için bu aşamadan sonra ilk olarak yapıl­


ması gereken hangisidir?
A. Elektroensefalogram çekmek
B. Ön hipofiz hormon düzeylerine bakmak
C. Hipotalamik hormonların seviyelerine bakmak
D. Beyin omurilik sıvısında pleomorfik hücre aramak
E. Dinamik MR görüntülemesi yapmak

3- 1 ay önce kranial kitle nedeniyle opere edilen hasta çok


su içme ve sık sık idrar çıkma ş ikayetiyle başvuruyor. Hastada
bakılan tam id rar analizinde idrar dansitesi 1000 gr/dL(N :1003-
1035), idrar osmolaritesi ise 11 OmOsm/kg (N :300-1000) geli-
yor. Bu aşamadan sonra tan ıy a yönelik ilk yapılması gereken
aşağ ıd aki l erden hangisidir?
A. Vazopressin uygulama sonrası idrar dansitesi bakmak
B. Su kısıtlama testi yapmak
C. Copeptin düzeyi bakmak
D. Hipofiz MR çekmek
E. Se lla turkikadan biyopsi almak

4- Santra l ve nefrojenik diabetes insipidus ayırımında kulla-


nılan, TUS için soru potansiyeli yüksek glikoprotein yapılı
güncel molekül aşağıdakilerden hangisidir?
A. Co-peptin
B. IGF-1
C. Paraaminohipurik asit
D. Sistatin C
E. C-peptit
5- 54 yaşında kadın hastaya 4 yıl önce makroadenom nede-
niyle tetkik ediliyor ve GH salgılayan adenom tanısı konuluyor.
Ardından cerrahi uygulanıyor ve okretotid tedavisi başlanıyor.
Tedavi altında takipte IG F- 1, GH düzeyi artan hastaya çekilen MR
incelemede nüks kitle izleniyor. İzl emde ani gelişen baş ağrısı
ve bilinç bulanıklığıyla aci l servise getiriliyor. Bakılan vitallerin-
de ateşi 36,8 °(, tansiyon 70/60 mmHg ve kan şekeri 50 mg/
dl izleniyor. Fizik muayenede ise görme alanında azalma tespit
ediliyor. Bu hastada en olası tanı aşağıdakilerden hangisidir?
A. Empty sella sendromu
B. Kraniofarengioma
C. Sheehan sendromu
D. Hipofizer apopleksi
E. Lenfositik hipofizit

6- Hashimoto tiroiditi nedeniyle takipli 32 yaşındaki kadın has-


ta zaman zaman olan baş ağrısı ve görmede aza lma şikayetiyle
başvuruyor. Bu şikayet lerinin 3.çocuğunu doğurduktan sonra
arttığını ifade ediyor ve bakılan laboratuvarınd a prolaktin 98
µg/L ve sedimentasyon 105 mm/h izleniyor. Bu hastada en
olası tanı hangisidir?
A. Empty sella sendromu
B. Kraniofarengioma
C. Sheehan sendromu
D. Hipofizer apopleksi
E. Lenfositik hipofizit

7· Baş ağrıs ı, libido kaybı, impotans, halsizlik, soğuk intoleran-


sı yakınmalarıyla başvuran 28 yaşındaki kadın hastanın MR
incelemesinde 30x15 mm boyutunda adenomla uyumlu kitle
izleniyor. Laboratuvarda prolaktin 250 µg/L(N:15-25 µg/L), TSH
O, 1 mlU/L(N:0,4-4 mlU/L) geliyor. Ayrıca östrojen ve testosteron
seviyeleri de düşük izleniyor. Bu hastada en olası/ en doğru
tanı aşağıdakilerden hangisidir?
A. Hipofizer kaynaklı panhipopituitarizm
B. Hipotalamik kaynaklı panhipopituitarizm
C. Prolaktinoma
D. Primer hipotiroidi
E. Adrenal yetmezlik

8- 42 yaşındaki erkek hasta 4-5 aydır devam eden baş ağrısı,


galaktore, libido aza lm ası şikayetiy l e başvuruyor. Bakılan
serum prolaktin düzeyi 458 ng/ml olan hastaya çekilen MR'da
hipofizde 0,5 cm çapında adenomla uyumlu lezyon izleniyor. Bu
hastada en olası tanı aşağ ıd ak il erden hangisidir?
A. Hook effect (kanca etkisi)
B. Sa ik basısı
C. Makroprolaktinemi

..
D. Primer hipotiroidi
E. Risperidon kullanımı
9- 42 yaşındaki erkek hasta 4-5 aydır devam eden baş ağrısı,
galaktore, libido azalması şikayetiyle başvuruyor. Bakılan serum
1
prolaktin düzeyi 60 ng/ml olan hastaya çekilen MR da hipofizde
3 cm çapında adenomla uyumlu lezyon izleniyor. Bu hastada
prolaktinoma tanısını kesinleştirmek için bu aşamadan
sonra istenmesi gereken ilk tetkik hangisidir?
A. TRH stimülasyon testi yapılması
B. Hasta kanının polietilen glikolle (PEG) muamele edilmesi
C. Prolaktin antikoru bakılması
D. Serumun dilüe edilerek prolaktin düzeyinin ölçülmesi
E. Gonadotropin düzeyinin ölçülmesi

10- 28 yaşındaki kadın hastada tesadüfen hiperprolaktinemi


saptanıyor. Amenore, galaktore veya oligomenore gibi hiperpro-
laktinemi semptomları olmayan hastanın hipofiz görüntüleme-
sinde sella tabanında 3 mm çapında düzensiz sınırlı hiperintens
kitle tespit ediliyor. Hastaya bromokriptin tedavisi başlanıyor
ve 4 hafta sonra kontrol MR da kitlede değişiklik olmadığı ve
1

prolaktin düzeyinin aynı yükseklikte seyrettiği rapor ediliyor. Bu


hastada en olası tanı aşağıdakilerden hangisidir?
A. Hook effect (kanca etkisi)
B. Saik basısı
C. Makroprolaktinemi
D. Primer hipotiroidi
E. Risperidon kullanımı

11· Yukarıdaki hastada makroprolaktinemi tanısını kesin-


leştirmek için bu aşamadan sonra istenmesi gereken ilk tetkik
hangisidir?
A. Bromokriptinin kesilip kabergolin başlanması
B. Hasta kanının polietilen glikolle (PEG) muamele edilmesi
C. Stereotaktik radyocerrahi yapılması
D. Transnazal transsfenoidal cerrahi yapılması
E. Sistemik kemoterapi başlamak

12· Bir önceki sorudaki hastaya tetkikler sonucu makroprolak-


tinemi teşhisi konuluyor. Bu aşamadan sonra hasta izlemine
yönelik en doğru yaklaşım hangisidir?
A. Düşük doz dopamin agonisti başlanarak yıllık prolaktin takibi
B. İzl em yeterlidir
C. Stereotaktik radyocerrahi planlanması
D. Hipofizial partal sistemi görüntü lemek için anjiografi
E. Ön hipofiz hormon düzeylerine bakılması

13· Prolaktinom ada tek malignite kriteri meta sta zd ır. Metastatik
olgularda tercih edilmesi gereken ilaç hangisidir?
A. l<abergolin
B. Bromokriptin
C. Pergolid
D. Ouanagolide
E. Temozolamid
14· 54 yaşındaki erkek hasta geceleri horlama şikayetiyle
başvuruyor. Öyküsünden 1 ay önce diyabet tanısı aldığı ve bu
nedenle metformin, 4 ay önce de hipertansiyon nedeniyle
amlodipin başlandığı öğreniliyor. Fizik muayenede alın çizgile-
rinde belirginleşme, yüz hatlarında kabalaşma dikkati çekiyor.
Hastaya hipofiz MR çekiliyor ve hipofizde adenomla uyumlu
kitle görülüyor. Bu hastada kitlenin nedenini teşhis etmek için
en güvenilir test hangisidir?
A. Growth hormon düzeyi
B. 24 saat idrar kortizolü
C. İnsülin benzeri growth faktör
D. Serum prolaktin düzeyi
E. Growth hormon supresyon testi

15- 44 yaşında kadın hasta son 3 ayda 17 kilo alma, omuz


ve pelvik bölgedeki kaslarda hareketle artan ağrı şikayetiyle
geliyor. Yapılan fizik muayenesinde cildinin son derece incelmiş
-tn
olduğu ve buna bağlı cilt altı kapiler damarların görünür hale .
geldiği izleniyor. Vitallerinde tansiyon 150/130 mmHg, kan
şekeri 131 mg/dlgeliyor. Bakılan hemgramda nötrofil, eritrosit
ve trombosit sayısında artış, bazofil, eozinofil ve lenfosit sayı ­
sında ise azalma gözleniyor. Bu hastada bu aşamadan sonra
aşağıdaki testlerden hangisi yapılmahdır?
A. Plazma ACTH düzeyi
B. Hipofiz MR
C. 1 mg deksametazon supresyon testi
D. İnferior petrosal sinüs örneklemesi
E. Adrenal bilgisayarlı tomografi

'
16· Hastaya yapılan 1 mg deksametazon testine yanıt alına -
mıyor ve 24 saat idrar kortizolü de yüksek geliyor. Bu
aşamadan sonra ayırıcı tanı için yapılması gereken hangisidir?
A. Plazma ACTH düzeyi
B. Hipofiz MR
C. 1 mg deksametazon supresyon testi
D. İnferior petrosal sinüs örneklemesi
E. Adrenal bilgisayarlı tomografi

17· Yukarıdaki bakılan plazma ACTH sonucuna göre aşağıdaki


yorumlardan hangisini yapmak doğru olmaz?
A.Plazma ACTH düzeyi düşük gelirse ACTH bağımsı z Cushing
düşünülür ve adrenal kaynaklı sebepler için adrenal BT çekili r
B. Plazma ACTH düzeyi yüksek gelirse ACTH bağımlı Cushing
düşünülür ve santral patolojiler için hipofiz MR çekilebilir
C. Bilateral adrenal hiperplazi varsa tedavide adrenelektomi
yapılır
D. İnferior petrosal sinüs örneklemesi Cushing hastalığını Cus-
hing sendromundan ayıran altın standart testtir
E. Unilateral adrenal hiperplazi varsa tedavide unilateral adrena-
lektomi çoğu zaman yeterlidir
ıDı gflademisi
18- 44 yaşında erkek hasta halsizlik, soğuk intoleransı ve
saçlarda dökülme şikayetiyle geliyor. Yapılan fizik muayenede
hastanın apatik olduğu, solunum sayısının azaldığı ve tiroid
muayenesinde guatr tespit ediliyor. Çekilen EKG aşağıdaki gibi
geliyor.

Bakılan laboratuvarda TSH: 8,4 mU/L(N:0,4-3.45), sT3:6,7 pg/


ml(N:2,4-5), sT4:5 ng/dl(N:0,9-1,6) izleniyor. Bu hasta için en
olası tanı aşağıdakilerden hangisidir?
A. Graves hastalığı
B. Toksik adenom
C. Tiroid hormon direnci
D. TSHoma
E. Tirotoksik periyodik paralizi
E
19- 51 yaşında erkek hasta halsizlik, soğuk intoleransı, saçlarda
dö külme, unutkanlık ve işitme kaybı şikayetiyle geliyor. Yapılan
fizik muayenede hastanın apatik olduğu, solunum say ısının
aza ldığı izleniyor. Bakılan biyokimyada TSH yüksek; sT3 ve sT4
düzeyi düşük geliyor. Bu hastada en olası tanı ve bu aşama­
dan sonra yapılması gereken hangisidir?
A. Tersiyer hipotiroidi ~ TRH stimülasyon testi
B. Sekonder hipotiroidi ~ Hipofiz MR
C. Sekonder hipotiroidi ~ Düşük doz levotiroksin
D. Primer hipotiroidi ~ Anti-TPO bakılması
E. Subklinik hipotiroidi ~ 3 ay sonra tiroid fonksiyon testi tekrarı

20- Endemik iyot eksikliğinin o l duğu yerde yaşayan 46 yaşında ­


ki erkek hasta bilinç bozukluğuyla acil servise getiriliyor. Yakınla ­
rından öğrenildiği kadarıyla 1 haftadan beridir yoğun öküsürük
ve ateş ş ikayet i olan hastaya aci lde yapılan değerlendirmede
ateşi 35.5°C, kalp atım sayısı 45 atım /dk ve tansiyonu 90/60
mmHg ölçülüyor. Fizkik muayenesinde pretibial +4 ödemi olan
hastanın en olası tanısı hangisidir?
A. Primer hipotiroidi
B. Subklinik hipotiroidi
C. Miks ödem koması
D. Tiroid krizi
E. Akut atrial fibrilasyon

21- Bu hastanın tedavisi için en uygun rejim aşağıdaki l er­


den hangisidir?
A. Oral levotiroksin tedavisi
B. İ.v L-tiroksin + İ . v Glukortikoid
C. Yüksek doz sedatif ajan
D. Sadece intravenöz glukokortikoid
E. Aktif harici ı s ıtm a uygulanması
22· 44 yaşında erkek hasta sağ üst kadran ağrısıyla acil servise
başvuruyor. Yapılan incelemede akut kolesisitit tanısı konulan
hastanın öyksünden hipertiroidi nedeniyle bir gün önce ilaç
kullanmaya başladığı öğreniliyor. Hasta acil operasyona alınıyor
anestezi sonrasında nabzı 144/dk, ateşi de 41 °( olarak ölçülü-
yor. Bu hastada en olası tanı aşağıdakilerden hangisidir?
A. Hashitoksikozis B. Tiroid krizi
C. Aşikar primer hipertiroidi D. Miks ödem koması
E. Graves toksikozu

23· Bu hastanın tedavisinde hangi ilacın yeri yoktur?


A. Glukokortikoid B. Propranolol
C. İopanoik asit D. Propiltiourasil
E. Aspirin

24· 41 yaşındaki Hashimoto tiroiditi nedeniyle levotiroksin kul-


lanan hastada TSH baskılı, sT3 ve sT4 yüksek geliyor. Öyküsünden
ilaçlarını
düzenli kullanmadığı bazı günler unuttuğunu, bazı
günlerde ise fazladan ilaç aldığı öğreniliyor. Hastada bakılan
tiroglobulin < 1 ng/ml geliyor. Bu hastada en olası tanı aşağı­
dakilerden hangisidir?
A. Lenfositik tiroidit
B. Tirotoksikozis faktisia
C. Subakut lenfositit tiroidit
D. Hasmimoto tiroiditinin tirotoksikoz dönemi
E. Postpartum tiroidit

25· 40 yaşındaki erkek hasta boyunda ağrılı şişlik, çarpıntı,


terleme ve günde 8-1Okez olan ishal şikayetiyle başvuruyor. Öy-
küsünden kısa zaman önce solunum yolu enfeksiyonu geçirdiği
ve şikayetlerinin de enfeksiyon sonrası arttığı öğreniliyor. Fizik
muayenede tiroid bezi hafif sertleşmiş şekilde palpe ediliyor.
Tiroid fonksiyon testinde sT3 ve sT4 yüksek; TSH ise baskılı
izleniyor. Yapılan tiroid USG'sinde tiroid heterojenitesinde artış
rapor ediliyor. Bakılan laboratuvarda sedim 106 mm/h geliyor
ve radyoaktif iyot uptake'i düşük izleniyor. Bu hastada en olası
tanı aşağıdakilerden hangisidir?
A. Subakut lenfositik tiroidit
B. Akut tiroidit
C. Graves hastalığı
D. Subakut granülomatöz tiroidit
E. İlaç ilişkili tiroidit

26· Yukarıdaki hastanın tedavisinde aşağıdakilerden hangisinin


yeri yoktur?
A. Metilprednizolon B. Lugol solüsyonu
C. İbuprofen D. Aspirin
E. Propiltiourasil
l a tıQd ►"
aKa emısı
I.!!!!

27- Boyunda şişlik nedeniyle başvuran 56 yaşındaki erkek 29- Yapılan İİAB sonucu foliküler lezyon olarak raporlanırsa
hastanın yapılan tiroid USG'sinde sol iter 2x2 cm botuyunda orta bu aşamadan sonra yapılması gereken hangisidir? (Dahiliye)
sertlikte nodül izleniyor. Laboratuvarda sT3, sT4 ve TSH düzeyleri A. Total tiroidektomi
normal sınırlarda izleniyor. Bu aşamadan sonra yapılması B. BRAF / RET mutasyonu araştırmak
gerken aşağıdakilerden hangisidir? C. İİAB tekrarı
A. Total tiroidektomi D. 6 aylık periyotlarla USG takibi
B. Moleküler inceleme E. Lobektomi
C. İİAB
D. Tiroid sintigrafisi
E. Lobektomi

28- Yapılan İİAB sonucu foliküler neoplazi olarak raporlanırsa 30- 28 yaşındaki kadın hasta çarpıntı, terleme ve sıcak intole-
bu aşamadan sonra yapılması gereken hangisidir? ransı nedeniyle başvuruyor. Bakılan laboratuvarda sT3, sT4 yük-
A. Total tiroidektomi sek; TSH düzeyi ise düşük bulunuyor. Yapılan fizik muayenede
B. BRAF / RET mutasyonu araşt ırmak soliter 2x3 mm boyutunda tiroid nodülü izleniyor. Bu aşama­
C. İİAB dan sonra yapılması gerken aşağıdakilerden hangisidir?
D. 6 ay lık periyotlarla USG takibi A. Total tiroidektomi
E. Lobektomi B. Moleküler inceleme
f. İİAB
D. Tiroid sintigrafisi
· E. Lobektomi

Bethesda Sınıflaması

l 1
Bethesda 1 Bethesda 2 Bethesda 3 Bethesda 4 Bethesda 5 Bethesda 6

Önemi
Tanısal Foliküler lezyon Malignite
Bening bilinmeyen
olmayan veya foliküler düşündüren Malign
bulgular ati pi/ foliküler
sitoloji lezyon şüphesi bulgular
lezyon

· ı .. . ,..

i : ", -~
Sıcak nodül

TSH düşük ise


sintigrafi çeki li r
[

Soğuk nodül

TSH yüksek/ normalse İİAB yapılır

Malignite veya Be nign Foliküler neopl az i Foliküler lezyon


Non-diagnostik
şüphesi bulgul a rın varlı ğ ı / şüphesi Önemi belirsiz atipi

6-18 ayda bir Moleküler Moleküler·


Cerrahi İİAB tekrarlanır
USG ile takip inceleme inceleme

Genel cerrahi Genel cerrahi için Non-diagnostik

..
için lobektomi lobektomi / İİAB tekrarı

Yakın takip Cerrahi


31· Endemik iyot eksikliğinin olduğu yerde yaşayan 56 yaşın­
daki kadın hasta boyun ön bölgesinde şişlik şikayetiyle başvuru­
yor. Yapılan fizik muayende tiroİd bezinin ele geldiği ve diffüz
olarak büyüdüğü izleniyor. Öyküsünden de ilaç kullanmadığı
ve hiçbir ek hastalığı olmadığı öğreniliyor. Bakılan laboratuvar-
da sT3, sT4 ve TSH düzeyi normal, tiroid otoantikorları negatif
bulunuyor. Tiroid USG'sined tiroid dokusunda diffüz büyüme
izleniyor, nodül varlığı rapor edilmiyor. 24 saatlik idrarda iyot
atılımı 35 ug/L gelen bu hastada en olası tanı aşağıdakilerden
hangsidir?
A. İlaç ilişkili hipotiroidi
B. Tirotoksikozis
C. Ötroid hasta sendromu
D. Ötroid diffüz guatr
E. Hashimoto tiroiditi

32· İskemik kalp hastalığı ve romatoid artrit nedeniyle takipli


bir hastanın bakılan tiroid fonksiyon testlerinde sT3 düşük, sT4
ve TSH ise normal geliyor. 1 ay önce romatoid artrite bağlı geli-
şen amiloidoza bağlı KBY tanısı alıyor ve bu amaçla eritropoetin
ve kalsiyum asetat kullanıyor. Yapılan USG'de tiroid boyutları ve
paran kimi normal izleniyor. Hastaya yapılan TRH stimülasyon
testinde TSH yanıtı normal sınırlarda bulunuyor. Bu hastanın
en olası tanısı ve buna yönelik en uygun tedavi yaklaşımı
aşağıdakilerden hangisidir?
A. İlaç ilişkili hipotiroidi Eritropoetin tedavisinin kesilmesi
B. Tirotoksikozis Metimazol başlanması
C. Ötroid hasta sendromu Altta yatan hastalığı tedavisi
D. Ötroid hasta sendromu L-tiroksin tedavisi
E. Hashimoto tiroiditi Glukokortikoid

33· Tüberküloz nedeniyle 3 yıl önce tedavi edilen 41 yaşındaki


kadın hasta 6 aydır devam eden halsizlik, kilo kaybı, bulantı - kus­
ma şikayetiyle başvuruyor. Fizik muayenede aksiller kıllanmada
azalma, ağız mukozasında pigmentasyon artışı dikkati çekiyor.
Laboratuvarda Na+: 130 mEq/L, K+:s, 9 mEq/L geliyor. Bu has-
tada en olası tanıya yönelik ilk istenmesi gereken tetkik
hangisidir?
A. FSH ve östrodiol
B. FGF-23 ve parathormon
C. ACTH ve kortizol
D. Aldosteron ve plazma renin
E. İdrar ve plazma metanefrin

34· Yukarıdaki hastaya adrenal yetmezlik tanısı konuluyor ve


glukokortikoid replasmanı olarak hidrokortizon başlanıyor. Baş­
lanan hidrokortizonun doz ayarında en değerli parametre
hangisidir?
A. Serum K+ düzeyi
B. Serum Na+ düzeyi
C. Hastanın klinik durumu
D. Hiperkalemi
E. Plazma kortizol düzeyi

..
35- 52 yaşındaki kadın hasta hipertansiyon ve poliüri şikaye­
tiyle başvuruyor. Öyküsünden ilaç kullanmadığı zaman zaman
kas krampları olduğu öğrenilen hastanın fizik muayenesinde
kan basıncı 180/120 mmHg ölçülüyor. Bakılan laboratuvarda
Na+: 148 mEq/L, K+:2,1 mEq/L izleniyor. Ayrıca renin düzeyi
de düşük geliyor. Hastaya yapılan renal doppler USG'de renal
arter kan akımları olağan izleniyor. Bu hastada en olası tanı
aşağıdakilerden hangisidir?
A. Liddle sendromu B. Gitelman send romu
C. Primer hipoaldosteronizm D. Sek. hiperaldosteron
E. Feokromasitoma

36- 32 yaşında erkek hastada ata kl ar halinde gelen çarpıntı,


yü zde so lukluk ve hipertansiyon nedeniyle başvuruyor. Ayrıca
s ık sık terlediği ve 2 ay önce diyabet tan ısı nedeniyle metformin
kullandığı öğreniliyor. Sağ lık bakanlığı sistem inden h asta nın
tn
eski epikrizleri ne bakan doktor hastanın 2007 yılınd a kol esis- "'
tektomi ameliyatı esnasında kan bas ıncınd a oynamalar olduğu,
ame liyatın güç bela bitirildiği ve tüm ameliyat boyunca hasta- (U
nın hipertansif seyrettiğini farkediyor. Hastaya çekilen adrenal
BT'de 3,2 cm çap lı heterojen kitl e rapor edi liyor. Bu hastada en
olası tanı aşağıdakilerden hangisidir?
A. Liddle sendromu
B. Conn sendromu
C. Primer hiperaldosteronizm
D. Familya! hiperaldosteronizm
E. Feokromasitoma

37- Vakad a tan ıya yön elik en sensitif idrar testi hangisidir?
A. 24 saat idrarda total metanefrin
B. 24 saat idrarda metanefrin / normetanefrin
C. 24 saat idrarda valin mandelik asit bakılması
D. İdrarda normetanefrin
E. İ drarda serbest metanefrin

38. Aralıklı, şiddetl i hipertansiyonu olan 45 yaşındaki kadın


hasta polikliniğe başvuruyor. Gün içinde atlet değiştirecek kadar
ter l ediğini
ve çarpıntı nedeniyle ö l ecekmiş gibi hissettiğini
ifade eden hastanın hipertansiyon ve anksiyete için sertra lin,
labetolol, ben idipin ve kaptopril kullandığı öğreniliyor. Hastada
ön planda feokromas itoma düşünülüyor ve 24 saatlik idrar
metanefrin düzeyi normalin 2 katı yü kseklikte geliyor. Bu hasta
hakkında aşağ ı daki yorumlardan hangisi doğru değildir?
A. Hastada tanı için yapılan testler 1 hafta süreyle labatolol
kesilerek tekrarlanmalıdır
B. Hastada bakılan idrar metanefrin testi tan ıya yön elik en
sensitif idrar testidir
C. Hipertansif kriz ge li ş irse en iyi tercih fento lam indir
D. Plazma adrena lin ve noradrenalin düzeyi takipte kullanılan
güven ilir bir parametredir
E. Sekonder diyabet görülme riski artm ı ştır
39-Ailesinde diyabet öyküsü olan 45 yaşındaki kadın hasta
rutin diyabet taraması için başvuruyor. Açlık plazma glukozu
98 mg/dL geliyor. Bu aşamadan sonra yapılacak en doğru
yorum hangisidir?
A. Bozulmuş açlık glukozu
B. Bozulmuş glukoz intoleransı
C. Diyabetes mellitus
D. Normal glukoz toleransı
E. Gestasyonel diyabet

4O-Aşağ ıdakil erden yorumlardan hangisi diyabet taraması için


yanlış bir ifad dir?
A. Tip 1 diyabet için rutin tarama endikasyonu yoktur
B. Tip 1 diyabet için klasik diyabet semptom ve bulguları mevcut
ise tanı amaçlı kan glukoz ölçümü yapılmalıdır
C. Her birey 40 yaşında sonra 3 yılda bir tip 2 diyabet açısından
taranmalıdır
D. Daha önce prediyabet saptanan bireylerde yılda bir kez diya-
bet taraması yapılmalıdır
E. Gestasyonel diyabet tanısı a lmış kadınlarda yılda bir diyabet
taraması yapılmalıdır

41- Kalp yetmezliğinde en güvenilir DPP-4 inhibitörü ilaç


hangisidir?
A. Vildagliptin
B. Linagliptin
C. Eksenatid
D. Saksagliptin
E. Sitagliptin

42- Ailesinde diyabet öyküsü olan 45 yaşındaki kadın hasta


rutin diyabet taraması için başvuruyor. Açlık plazma glukozu
120 mg/dl geliyor. Bunun üzerine OGTT yapılıyor ve testin
2.saatinden ölçülen kan şekeri 135 mg/dl geliyor. Bu hasta için
en doğru yorum aşağ ıd aki l erden hangisidir?
A. Bozu lmuş açlık glukozu
B. Bozu lmu ş glukoz intoleransı
C. Diyabetes mellitus
D. Normal glukoz to l eransı
E. Gestasyonel diyabet

43 • 46 yaşında erkek hasta yemek yedikten 2 saat sonra


başlayan soğuk terleme, taşikardi ve o anda evde ölçülen klan
şekjer inin düşük olması üzerine başvuruyor. Açken şikayetin in
olmadığını ifade eden bu hastada tedaviye yönelik en uygun
ilaç hangisidir?
A. Akarboz B. Saksagliptin
C.Liraglutid D. Eksenatid
E. Glukagon
la t•~·d
aKa
I.!!! !l.!!!I 'emısı
►"

44· Kronik yorgunluk, gün içinde brkaç kez olan el ve ayaklarda


kramp yakınmalarıyla başvuran 54 yaşındaki kadaın hastanın
öyküsünden graves nedeniyle tiroidektomi olduğu ve bu sebep-
le düşük doz levotiroksin kullandığı öğreniliyor. Fizik muayene
ve öykü derinleştirildiğinde karın ağrısı şikayetlerinin de oldu-
ğunu ve 3 ay önceki göz muayenesinde subkapsüler katarakt
tanısı aldığı öğreniliyor. Laboratuvarda bakılan serum kalsiyum
düzeyi 7, 1 mg/dL ve serum parathormon düzeyi düşük ve fosfor
yüksek gelen hastada etiyolojiye yönelik ilk bakılması
gereken hasngisidir?
A. Dvitamin
B. Fosfat
C. Serum kreatinin
D. Albumin
E. Kalsitonin

45- 69 yaşında kadın hasta sırt ağrısı şikayetiyle başvuruyor.


Çekilen DEXA'da L1-4 Tskoru -1.8, femur boynu Tskoru -1,4
geliyor. Bu hasta için en uygun tedavi hangisidir?
A. Kalsiyum+ Dvitamini
B. Kalsiyum + Dvitamini + Fosfor
C. Kalsiyum + Dvitamini + Parathormon
D. Bifosfanat + Parathormon
E. Kalsiyum + Dvitamini + Bifosfanat

46- Stabil angina pektoris kliniğiyl e başvuran 21 yaşındaki


erkek hastada aşil tenedonu üzerinde ksantom, göz çevresinde
ksantelezma ve arcus kornea izleniyor. Öyküsünden babasının
ve 2 amcas ının erkek yasşa myokard enfarktüsünden dolayı
öldüğü öğreniliyor. Vakadaki hastaya bir statin olan atorvastatin
başlanıyor ve kontrol kolesterol düzeyi 985 mg/dl geliyor. Bu
aşamadan sonra bu hasta için en uygun medikal tedavi
aşağ ıdakilerden hangisidir?
A. İlacı rosuvastatinle değiştirmek
B. Gemfibrozilin yanına nikotinik asit eklemek
C. LDLafarez planlamak
D. Atorvastatin dozunu maksimum a çıkmak
E. Probukol tedavisi başlamak

47- Hipertrigliseridemide Apo- C3'ü in hibe ederek etki


gösteren ilaç güncel il aç hangisidir?
A. Evolocumab
B. Pradigastat
C. Alipogenetiparvovec
D. Volanesorsen
E. Evinacumab
g ~flademisi
48- 38 yaşında kadın hasta acil servise bilinç bulanıklığı şika­
yetiyle getiriliyor. Hastanın derin soluduğu ve solunuma karın
kaslarının da katıldığı gözleniyor. Öyküsünden bilinen diya-
beti olduğu ancak ilaçlarını düzenli kullanmadığı öğrneiliyor.
Laboratuvar incelemede plazma glukozu 560 mg/dl, potasyum
3 mEq/L geliyor. İdrar analizinde +3 keton izlenen hastanın
bakılan kan gazında pH:7.2, HC03:14 mEq/L izleniyor. Vakadaki
hasta için en uygun tedavi hangısidir?
A. 30 mEq/L potasyum replasmanı yapılması
8. O, 15 IU/kg'dan insülin infüzyonu uygulanması
C. 1000 cc/h'den izotonik NaCI uygulanması
D. İntravenöz HC0 3 infüzyonu başlanması
E. 250 cc/h'den dekstroz infüzyon başlanması

49- Tekrarlayan karın ağrısı şikayetiyle doktora başvuran 56


yaşındaki erkek hastanın fizik muayenesinde ekstremitelerin
extansör yüzlerinde ksantom ve lipemik retionopati izlaniyor.
Hastaya bir fi brik asit türevi olan gemfibrozil başlanıyor ve kont-
rol trigliserid düzeyi 1250 mg/dL geliyor. Bu aşamadan sonra bu
hasta için en uygun tedavi aşağıdakilerden hangisidir?
A. İlacı fenofibratla değiştirmek
8. Gemfibrozilin yanına nikotinik asit eklemek
C. Trigliserid afarez planlamak
D. Tedaviyi kolestiraminle değiştirmek
E. Probukol tedavisi başlamak

50- 34 yaşında tiroid nodülüyle tetkik edilen bir ha sta nın öykü-
südnen babasının tiroid kanseri nedeniyle öldüğü ve kardeş­
lerinde ve kendisinde tekrarlayan böbrek ta şı şikayeti olduğu
öğreniliyor. Laboratuvarda bakılan kalsitonun düzeyi 1800 pg/
mL(N:<100 pg/mL) gelen hastanın kalsiyumu yüksek, fosfatı
düşük, potasyum ve sodyum düzeyleri ise normal bulunuyor.
Hastayı değerlendiren hekim ailede tiroid kanseri olduğu için
tiroidektomi planlıyor. Bu hastada beklenen en ola ı mutas-
yon hangisidir?
A. Menin gen mutasyonu
8. RET mutasyonu
C. CDKN18 mutasyonu
D. NF-1 gen mutasyonu
E. APECED gen mutasyonu
2023 MARTTUS SORU KAMPI
l a tı~, d .,. Dahili e
I!!!! !!!!I aKa · ,emısı
HEMATOLO:d
1- 24 yaşında bir kadın hasta halsizlik şikayetiyle başvuruyor.
Yapılan fizik muayenede skleralarda solukluk dışında bir bulgu
izlenmiyor. Bakı lan laboratuvar incelemede hemoglobin 8, 7 g/
dl, lökosit 5800/mm3, trombosit 657 .000/mm3 ve MCV 69 fl
izleniyor. Bakılan pefirefik yayma aşağıdaki gibi izleniyor.

Vakadaki hastada demir eksikliği anemisinin en olası nedeni


aşağ ıdakilerden hangisidir?
A. GİS kökenli kan kayıpları
B. Menstrüel kanama
C. Alım yetersizliği
D. Gastrektomi
E. Kronik böbrek yetmezliği

2- Demir tedavisine yanıtsızlık durumunda aşağ ıdakil er-


-o
den hangisini düşünmek doğru olmaz?
A. Ülseratif kolit varlığı
B. Crohn varlığı
C. Gastrik antral vaskül er atreziye bağlı minör kanama
D. Kronik hastalık anemisi
E. Divertikül kanaması

3- 51 yaşınd a
erkek hasta yutma güçlüğü şikayetiyle geliyor.
Yapılan fizik muayenede dilinin atrofik olduğu ve anguler
stomatit varlığı dikkati çekiyor. Bakılan laboratuvarda ferritin,
hemoglobin ve MCV düşük izleniyor. Disfajiye yönelik çeki len
baryum grafi ise şeki l deki gibi izleniyor.

Bu hastanın en olası tan ı sına yönelik aşağ ı daki ifadelerden


hangisi doğru bir ifade deği dir?
A. Hastanın en olası tan ı s ı plummer vinson sendromudur
B. Demir eks ikli ği anem isi düze lince grafideki web spontan
düzelir
C. Fa renks kanser riskinde artış izlen ir
D. Periferik yaymada mikrositer değişiklikler izlenir
E. Uzun dönemde özefagus adenokanser riski artmıştır
4- Kronik böbrek yetm ez liği nedeniyle rutin diyalize giren 56
yaşındaki erkek hastada bakılan laboratuvarda serum demiri ve
demir bağlama kapasitesi düşük, ferritin yüksek izleniyor. Bu
hasta için en olası tanı aşağıdakilerden hangisidir?
A. Aplastik anemi
B. Demir eksikliği anemisi
C. Sıcak tip otoimmün hemolitik anemi
D. Kronik hastalık anemisi
E. 812 vitamin eksikliği

5- 51 yaşınd a bir erkek hasta halsizlik ve yorgunluk yakınma ­


larıyla başvuruyor. Öyküsünden 15 yıldır kronik alkol alımı ve
immünsupresan ilaç kullandığı için 8 aydan beridir profilaktik Glisin + Süksinil Coa

izoniazid kullandığı öğreniliyor. Laboratuvar incelemelerinde i


5-aminolevulinik asit • • • • • • ► 5-aminolevulinik asit
hemoglobin: 7,7 g/dL, lökosit: 5800/mm 3, trombosit: 328.000/
mm 3 ve ortalama eritrosit hacmi (MCV): 75 fL olarak saptanıyor. HEM
Yapılan periferik yayma şekildeki gibi geliyor.
Porfobilinojen

Koptoporfirinojen 111 ◄• ••••• Koptoporfirinojen 111

Bu hasta için en olası tanı ve en uygun tedavi aşağ ıd ak il er­


den hangisinde birlikte verilmiştir?
Tanı Tedavi
A. Demir eksikliği anemisi Oral demir
B. 812 vitam in eks ikliği 812 vitamini
C. Kronik hastalık anem isi Eritro poeti n
D. Folat e ksikli ği Folik asit
E. Siderob lastik anemi Piridoksin

6- 56 yaşında bir kadın hasta uzun süredir devam eden halsiz- Antikorlar:
* Anti-intrinsik faktör
lik, stomatit, ataksi ve oryantasyon bozukluğu yakınmalarıyla * Pari yeta l hücre antikoru

başvuruyor. Laboratuvar incelemelerinde hemoglobin düzeyi


6.0 g/dL, lökosit sayıs ı 1800/mm 3, trombosit sayı sı 80 000/mm 3
ve ortalama eritrosit hacmi (MCV) 140 µ3, retikülosit %1 izleni-
yor. Bu hastanın ayırıcı tanısında aşağıdakilerden hangisi
yardımcı olmaz?
A. Serum LDH düzeyi Dorsal
yol:
Latera l yol tutu lu mu :
Spa stik parezi / parestezi
B. Retikülosit sayımı 1
Ataksi \ /

C. İdrard a ürobilinojen tayini


D. Serum folik asit ve 812 vitamin i düzeyi Spinal kordda dorsal ve
lateral yollarda miyelin
E. Serum ferritin düzeyi kaybı, periferik nöropati
I
~
IJ blQ·
aKad.• 1
. "
emısı

7- 19 yaşında bir erkek hasta giderek artan unutkanlık, bacaklar-


da güçsüzlük ve ekstremitelerinde uyuşma şikayetleriyle başvu­
ruyor. Hastanın bakılan laboratuvarında hemoglobin düzeyi 9,8
g/dL, ortalama eritrosit hacmi (MCV) 130 µ3 ve retikülosıt %1,7
ve serum kobalamin düzeyi düşük izleniyor. Bakılan tam idrar
ana lizinde spot idrar protein/kreatinin oranı 2,4 gr/g geliyor. Bu
hastada en olası tanıya yön elik görülmesi beklenen genetik
değişiklik aşağıdakilerden hangisidir?
A. BRAF gen mutasyonu
B. RNA polimeraz ili
C. NOD 2 gen ekspresyonu
DCubilin reseptörü
E. ALA sentazı kodlayan gen mutasyonu

8- 21 yaşında erkek hasta bir aydan beridir var olan halsizlik,


efor dispnesi ve yeni başlayan peteşiler ve burun kanaması
şikayeti ile başvuruyor. Fizik muayenede lenfadenopati veya
organomegali izlenmiyor. Bakılan periferik yayması normokrom
makrositer anemi ile uyumlu izleniyor. Laboratuvarda hemoglo-
bin 6.4 gr/dl, trombosit sayısı 38000/mm 3, lökosit 1500/mm3, Lipit

retikülosit sayısı% 0.2 olarak bulunuyor. Pansitopeniye yönelik


yapılan kemik iliği incelemesinde ilik hipoplazik izleniyor ve ilik
içinde yağ infiltrasyonunun artmış olduğu izleniyor.

..,~:~>,::.:\
'-~ . . . ' .. ' .
••

• ~
ı,,-#

I •
(
.... , ~

{;, . ~ \
1

-~ p

Bu hastada en olası tanı aşağıdakilerden hangisidir?


A. Saçlı hücreli lösemi
B. Aplastik anemi
C. Kronik myelositik lösemi
D. Primer myelofibrozis
E. Paroksisma l noktürnal hemoglobinüri

9- 45 yaşında bir erkek hasta burun kanaması şikayetiyle


başvuruyor. Fizik muayenede peteş i, purpura ve ekimoz izlenen
hastada bakılan laboratuvarda hemoglobin 8,9 g/dL, lökosit
say ı s ı 1800/mm 3,trombosit sayısı 67.000/mm 3 ve trabue alanı
kapalı izleniyor. Aşağıdaki hasta lıkl ardan hangileri bu hastanın
ayırıcı tanıları içinde yer almaz?
1. B12 eksikliği
il. Aplastik anemi
m. Akut lenfoblastik lösemi
iV. Hairy cell lösemi
V. Kronik karaciğer hastalığı
VI. Akut miyeloblastik lösemi
A. 1-- 11--1 11 B. Sadece il
C.1-- 11 D.11-- iV
E. IV --V--VI

..
10· 21 yaşında erkek hasta ateş ve halsizlik şikayetiyle acil ser- Revesv Sendromu Coast Plus Sendromu
vise başvuruyor. Öyküsünden erkek kardeşinin 16 yaşında ateşli Diskeratozis konjenita Diskeratozis konjenita
bir hastalık nedeniyle kaybedildiği öğreniliyor. Fizik muayenede Bilateral eksudatif retinopati Retina! telanjiektazi
retiküler tarzda cilt pigmantasyonu, şekildeki gibi ağız muko- Kemik iliği yetmezliği Serebellar hareket bozukluğu
zasında lökoplakiyle uyumlu cilt lezyonu, tırnaklarda distrofik İntrauterin gelişme geriliği Osteopeni
değişiklikler izleniyor.
Serebellar hipoplazi Lökodistrofi

Bakılan laboratuvarı pansitopeniyle uyumlu olan hastanın


kemik iliği incelemesi hiposellüler olarak rapor ediliyor. Bu
hastanın en olası tanısı aşağı kilerden hangisidir?
A. Diskeratozis konjenita
B. Coast plus sendromu ti)
A
C. Revesz sendromu
D. Fanconi aplastik anemi
E. Diamond Blackfan anemisi

11- 17 yaşında erkek hasta solukluk nedeniyle getiriliyor. Öykü-


sünden hastanın sabah uyandığında kendisini çok iyi hissettiği
ancak ilerleyen saatlerde göz kapaklarında ve bacaklarında
güçsüzlük geliştiği ve günlük aktivitelerini ciddi anlamda kısıtla­
dığı öğreniliyor. Fizik muayenesinde de pitozis izlenen hastanın
çekilen BT'si aşağıdaki gibi izleniyor.

Laboratuvar incelemelerinde hemoglobin:4,2 g/dL, MCV: 103 fL,


retikülosit: %0,6 bulunuyor. Kemik iliği incelemesinde sadece
eritrosit öncüllerinin azaldığı, diğer serilerin normal olduğu ve
şekildeki gibi dev proeritroblastlar görülüyor.

...

Bu hasta için en olası tanı aşağıdakilerden hangisidir?


A. Fanconi aplastik anemisi
B. Diamond-Blackfan anemisi
C. Saf eritroid aplazisi
D. Otoimmün hemolitik anemi

..
E. Bernard-Soulier sendromu
2023 MARTTUS SORU KAMPI
ıa!!!!l tı~
aKad·emısı
. " Dahili e
12- 21 yaşında erkek hasta halsizlik ve skleralarda ikter olması
üzerine başvuruyor. Yapılan fizik muayenede dalağın inguinal
bölgeye kadar palpe edildiği izleniyor. Laboratuvarda hemoglo-
bin 9 g/dL MCHC %41, retikülosit %4 indirekt bilirubin 6 mg/
1 1

dl izleniyor ve bakılan direk coombs testi negatif izleniyor. Yapı­


lan ultrasonografide şekildeki gibi multiple safra taşı izleniyor.

Genç Yaşlanan Membranda Eritrosit/erin Dalak Da/aktaki


eritrosit eritrositte fragmentasyon sferosit halini tarafından makrofajlar
çıkıntılar gelişir kaybı a lm ası tanınma tarafından. yıkılma

Bu hastada tanı için aşağıdaki tetkiklerden hangisi önce-


likle istenmelidir?
A. Serum haptoglobulin düzeyi
B. Eritrosit ozmotik frajilite analizi
C. Hemoglobin elektroforezi
O. Eozin 5-maleimide testi
E. Kemik iliği biyopsisi

Pıt6Ageııt~
13- 51 yaşında bir erkek hasta akut başlayan karın ağrısı ve
sarı lık nedeniyle acil servise başvuruyor. Yapılan fizik muayenede
hepatomegali ve asit saptanıyor. Karın ağrısına yönelik çekilen
\ CD 55 C3 koırwertaz

anjiografide şekildeki gibi superior mezenter arterde tromboz !


izleniyor. C3


CD 59 C3b
!
ı
CS korıvertaz

ı
C5

Hastanın öyküsünden 6 ay önce yapılan kontrolde bu bulguların


I \
C5a C5b
bulunmad ığı öğreniliyor. Laboratuvar incelemelerinde hemog-
lobin: 7.5 g/dl, lökosit: 3500/mm 3, trombosit: 50 000/mm 3 1
ı
.._-t:Jı---ı CSb-6-7-8-9
retikülosit: %5, 0-dimer: 2500 olarak izleniyor ve haptoglobulin (MAC)
ve hemopeksin düzeyi ise düşük geliyor. Bu hasta için en olası
tanı ve bu tanıyı koyabilmek için yapılması gereken
aşağıdaki l erin hangisinde verilmiştir?
A. Pulmoner emboli - Acil toraks anjiografi
B. So lid tümör infiltrasyonu - Acil bilgisayarlı tomografi
C. Paroksismal noktürnal hemoglobinüri - Akım sitometrik
inceleme
O. Otoimmün hemolitik anemi - Direkt Coombs testi
E. Behçet hastalığı - Romatoloji konsültasyonu

14· Vakadaki hastada tedaviye yönelik ilk tercih ilaç aşağ ı de­
kilerden hangisidir?
A. Gemtuzumab B. Ecu lizumab

..
C. Erlotin ib D. Alemtuzumab
E. Midastaurin
15- 25 yaşında erkek hasta acil servise ateş, öksürük ve balgam
şikayetiyle başvuruyor. Laboratuvar incelemede hemoglobin 9
g/dl, retikülosit %5, MCV 103 fl izleniyor. Öyküsünden son bir
yılda 3 kez pnömoni geçirdiği öğreniliyor. Yapılan batın ultraso-
nografide dalağın normal boyutlarının altında olduğu izleniyor.
Bu hastada anemi etiyolojisine yönelik aşağıdaki testlerden
hangisi yapılmalıdır?
A. Oksijen-hemoglobin disosiyasyon eğrisi
B. Hemoglobin elektroforezi
C. Ozmotik frajilite testi
D. Asit-Ham testi
E. Serum ferritin düzeyi tayini

16-11 yaşında orak hücre anemisi olduğunu bilinen bir kız çocu-
ğu acil servise karın sol kadranda son 1 saatte giderek artan karın
ağrısı şikayetiyle getiriliyor. Bakılan hemogramı pansitopeniyle
uyumlu olan hastanın retikülosit değeri% 12 bulunuyor. Yapılan
BT'de dalak şekildeki gibi izleniyor.

Vakada OHNsinin hangi komplikasyonu gelişmişt ir?


A. Ağrılı krizler B. Megaloblastik kriz
C. Aplastik kriz D. Hemolitik kriz
E. Splenik sekestrasyon krizi

17- Orak hücre anemisi olduğu bilinen 23 yaşındaki intern


doktor kışın ayazında Erzurum'a TUS kampı için gidiyor. Soğuk
hava karşısında el parmaklarında ciddi ağrı gelişen hasta TUS
kampını boşverip en yakın acil servise gidiyor. Acil serviste el
ayak sendromu tanısı konulan hastada hangi monoklonal antikor
ağrılı krizleri önlemede profi laktik olarak verilebilir?
A. Crizanlizumab B. Caplacizumab
C. Ravulizumab D. Eculizumab
E. Rilonacept

18- Hodgkin hastalığı nedeniyle takipli 34 yaşındaki erkek hasta


ani gelişen sarılık şikayetiyle geliyor. Bakılan laboratuvarda serum
LDH düzeyi ve retikülosit artmış, indirekt hakimiyetinde hiperbi-
lirubinemi izleniyor. Fizik muayenesinde traube alanının kapalı
olduğu izleniyor. Detaylı incelemede lg Garacılı direkt coombs
testi pozitif gelen hastanın tedavisinde ilk tercih edilecek ilaç
aşağıdakilerden hangisidir?
A. Rituksimab B. Plazmaferez
C. Steroid D. Splenektomi
E. İntravenöz immünglobulin
ıa!!!I tı~ d. . "
OKa .emısı
19· 71 yaşında bir erkek hasta halsizlik nedeniyle başvuruyor.
Fizik muayenede hepatosplenomegali saptanmıyor. Laboratuvar
incelemelerinde lökosit:780/mm 3, trombosit: 67.000/mm3,
hemoglobin: 10.8 g/dL, ortalama eritrosit hacmi (MCV): 122.8
fL olarak bulunuyor. Periferik yaymada blast izlenmiyor ancak
kemik iliğinde% 8 blast, nonlobüle megakaryositler, şekildeki
gibi granülsüz hiposegmente displastik nötrofiller ve nükleosi-
top lazmik disosiasyon belirleniyor.

Bu hastanın tedavisi hakkında hangisi yanlış bir ifadedir?


A. Sq delesyonu varlğında en etkin ilaçdesitabindir
B. Nakil için uygun olmayan hastalara 5-azasitidin ve desitabin
verilir
C. Ykromozom anoma lisi izlenebilir
O. Kötü prognoz kriterlerinin varlığı AKİT endikasyonu doğurur
E. Nötr.openiye yönelik G-CSF preparatları verilebilir

20- 21 yaşındaki erkek hasta bacaklarında morluk şikayetiyle


geliyor. Laboratuvar incelemesinde lökosit sayısı 95.000/mm3
ve trombosit sayısı 9.000/mm 3 bulunuyor. Periferik yaymasında
şekildeki gibi blastlar izleniyor.

Akciğer grafisinde ön mediastinal kitle saptanıyor. Kemik iliği


incelemesi sonucu %24 blast görülmesi üzerine akut lösemi tanısı
konuluyor. Hastaya CD 19 · CD3 etkileşimini bloke eden bir
ilaç başlanıyor. Bu ilaç aşağıdak il erden hangisidir?
A. Midastaurin B. Venetoclax
C. İbrutinib D. Obinutuzumab
E. Blinatumab

21· 41 yaşında erkek hasta 1 haftadan beridir artan halsizlik, yor-


gunluk ve iştahsızlık şikayetiyle geliyor. Fizik muayenesinde ateşi
39°( izleniyor ve bakı l an hemogramda lökosit 1200/mm 3, nötrofil
850/mm 3, trombosit 45.000/mm 3 geliyor. Ateş ve nötropenisi
olan hastada febril nötropeni düşünülerek kültürleri alınıyor ve
ardından ampirik antibiyotik tedavisi başlanıyor. Hastaya yapılan
kemik iliği aspirasyonunda promiyelositlerin sayısında artış izleni-
yor. Bu hastanın en olası tanısı aşağ ıd akilerden hangisidir?
A. Akut miyeloblastik lösemi B. Miyelodisplastik sendrom
C. Kronik lenfositik lösemi O. Akut lenfoblastik lösemi
E. Folliküler lenfoma
22· Vakadaki hastada beklenen en olası genetik değişiklik
aşağıdakilerden hangisidir?
A. BCR-ABL füzyon geni
B. PML-RARa füzyon geni
C. AML 1-ETO füzyon geni
D"MLL geni bozukluğu
E. CBF~ -MYH 11

23· 57 yaşında kadın hasta bilinen AML-M3 tanısı nedeniyle ATRA


kullanan hastada 39 dereceyi bulan ateş gelişiyor. Doktor mua-
yenesi esnasında +3 pretibial ödem tespit edi liyor. Nefes darlığı
çektiği fark edilen hastada pulse oksimetre ile bakılan 02 satu-
rasyonu 80 geliyor. Hastaya çekilen akciğer grafisinde bilateral
pulmoner infiltrasyonu ve kostofrenik sinüsler kapalı görünüyor:
Bu hastada en olası tanı aşağ ıd ak il erden hangisidir?
A. AML'nin doku infiltrasyonu
B. Masif plevra! efüzyon
C. ATRA sendromu
D. Heparinle indüklenebilir trombositopeni
E. Blastik transformasyon

24- Vakadaki hasta için bu aşamadan sonra en uygun tedavi


aşağıdakilerden hangisidir?
A. Antrasiklin + glukokortikoid
B. İmipenem + amikasin
C. Arsenik trioksit
D. Taze donmuş plazma+ heparin
E. Midastaurin

25- 54 yaşında erkek hasta halsizlik, yorgunluk ve erken doyma


şikayetiyle geliyor. Sk l eraları soluk izlenen hastanın fizik muaye-
nesinde dalağın kot altından inguinal bölgeye kadar uzandığı
izleniyor. Bakı l an hemogramda lökosit 890/mm 3, nötrofil 235/
mm3,trombosit 30.000/mm 3 izleniyor. Yapılan kemik iliğinde
şekildeki gibi saçaksı lenfositler izleniyor.

Yapılan immünhistokimyasal incelemede CD 103, CD 123 pozitif


izleniyor. Ayrıca TRAP ve Annexin A1 pozitif boyanıyor. Moleküler
incelemede BRAF mutasyonu pozitif izlenen bu hasta için en
olası tanı aşağıdaki l erden hangisidir?
A. Akut lenfoblastik lösemi
B. Hairy ce ll lösemi
C. Kronik nötrofilik lösemi
D. Erişkin Thücreli lösemi
E. Kronik myeloid lösemi
tDı ~flademisi
26· 72 yaşında erkek hasta inguinal, aksiller ve servi kal bölgede
ele gelen ağrısız, mobil, yumuşak kıvamlı ve hareketli şişlik-
ler nedeniyle başvuruyor. Laboratuar incelemesinde lökosit
124.000 mm 3 ve bunun yaklaşık% 85 inin lenfosit olarak rapor- 1

lanıyor ve periferik yaymayla doğrulanıyor. Hemoglobin 15 gr/


dl, trombosit 414.000/ mm 3 bulunuyor. Ayrıca hastada direkt
coombs testi (+)geliyor. Bakılan periferik yaymada aşağıdaki
gibi bol miktarda hücre izleniyor.

Hastaya yapılan lenfosit akım sitometrisi şekildeki gibi izleniyor.

C IC LYl9't«YTIC I.E\WD!IA

?ı :.~;\ \~., :;,;,{"'·: . ,._;~~-~

_L-..:::;i_::·
FU ·lt \CllJ.O · l' fılCP .... .. . .

Bu hastada en olası tanı aşağıdakilerden hangisidir?


AKronik miyeloid lösemi
B. Kronik lenfositik lösemi.
C. Hodgkin lenfoma
D. Akut lenfoblastik lösemi
E. Akut miyeloblastik lösemi

27- Yukarıdaki hasta için aşağıdaki durumlardan hangisi teda- Tedavi başlama endikasyon/arı:
vi endikasyon dur? • RAİ 3 ve 4 veya Binet C
• Masif veya progreslf sp lenıomegali
A. Lökosit sayısının 124.000 olması
• Uzun aksı 1ı 0 cm'den uzu n lenf nodu varlığ ı
B. Trombosit sayısının 414.000 olması
• 2 ay lı k takipte lenfosit sayıs ı n ı n %50 artması
C. Multiple LAP var lığı • Lenfosit 2'ye kaıtlanım a süresinin 6 aydan kısa, olması
D. Direkt coombs testi pozitifliği • Tedaviye dirençli otoimmün anemi ve/veya trombositopenıi
E. Hemogramda lenfositoz varl ığı • Hastalığ a bağl ı semptomlar

28- KLL tedavisinde bcl-2 inhibisyonu yaparak proliferasyonu


azaltan ilaç aşağıdakilerden hangisidir?
A. Ofatumumab
B. Obinutuzumab
C. Venetoclax
D. İbrutinib
E. İdelalisib

29· Kronik lenfositer lösemili bir hastada p53 mutasyonu


saptanması durumunda tedavide öncelikle hangisi kullanıl­
malıdır?
A. Fludarabin B. Rituksimab
C. Pentostati n
E. Kladribin
D. İbrutibin

..
30· 32 yaşında bir erkek hasta ÜSYE sonrası aşırı halsizlik yakın­
masıyla başvuruyor. Laboratuvar inceleme sonuçları şöyledir:
hemoglobin düzeyi 5,5 g/dL, lökosit 8500/mm 3, trombosit
197.000/mm 3, mutlak retikülosit sayısı 5000/mm 3, Lökositlerin
nötrofil %40, lenfosit% 55, monosit %4, eozinofil %1 olduğu
izleniyor. Periferik kan akım sitometrisinde ise lenfositlerde CD3
negatif, CD 16 ve CD 56 pozitif izleniyor. Bu hasta için en olası
tanı aşağıdakilerden hangisidir?
A. Kronik lenfositer lösemi
8. Parvovirus 819 enfeksiyonu
C. Viral hepatit
D. Natura! killer hücreli lösemi
E. T-LGL (T-large granular leukemia)

31- Aşağıdaki lösemilerden hangisinde CD 3, CD 8 ve CD 57 ·


pozitif olup, sıklıkla bir otoimmün hastalıkla birlikt dir?
A. Kronik miyeloid lösemi
8. Akut lenfoblastik lösemi
w
C. Büyük granüler lenfositik lösemi
D. Thücreli prolenfositik lösemi
E. 8 hücreli prolenfositik lösemi

32 -Aşağıdaki lösemilerden hangisinde klonaliteyi gösteren


sabit bir belirteç yoktur?
A. Erişkin Thücreli lösemi
8. Kronik eozinofilik lösemi
C. Büyük granüler lenfositik lösemi
D. Thücreli prolenfositik lösemi
E. 8 hücreli prolenfositik lösemi

33· 54 yaşında kadın hasta gece terlemesi, ateş ve son 4 ayda 8


kilo kaybetmesi üzerine başvuruyor. Yapılan fizik muayenede sağ
servi kal bölgede birçok lenf nodunun sert bir şekilde ele geldiği
gözleniyor. Hepato/splenomegali izlenmiyor. Bakılan hemogram
normal sınırlarda izleniyor. Lenf nodlarına yönelik yapılan artm ış
izleniyor. Lenf nodu biyopsisi ise şekildeki gibi izleniyor.

Bu hasta için en olası tanı aşağıdakilerden hangisidir?


A. Diffüz büyük 8 hücreli lenfoma B. Hogkin lenfoma
C. MALToma D. Foliküler lenfoma
E. Ana plastik büyük hücreli lenfoma
la tııcy
~ !!!!I aK.ademisı
u
~202.fHARttus~;sôRD~K8MP1
,oaıııı{:e~·~":~~--~~- ttt;~~L-:::~:::. ~<
34· 44 yaşında erkek hasta bilinen foliküler lenfoma ile takip ve
tedavi edilmekte. Hastanın son 4 aydır başlayan epigastrik karın
ağrıları şikayeti ile doktora başvuruyor. Yapılan muayenesinde
hastanın batında ele gelen masif bir kitlesi olduğu dikkati çekiyor.
Bakılan biyokimyasal parametrelerinde serum LDH seviyesi
normalin 1Okatı olarak tespit ediliyor. Çekilen batın BT'sinde me-
zenterik ve retroperitoneal lenfadenopatinin (oklar) oluşturduğu
homojen bir konglomera kitlesi izleniyor.

Bu hasta için aşağıdakilerden hangisi en doğru tan dır?


A. Batın için multiple metastazlarla
B. Mezenterik pannikilit
C. Folliküler lenfomadan transforme diffüz büyük Bhücreli
lenfoma
D. Tümör infiltrasyonuna bağlı akut mezenterik iskemi
E. Diffüz büyük Bhücreli lenfoma progresyonu

35- 51 yaşında kadın hastada, halsizlik, yorgunluk, ateş, sol üst


kadranda ağrı ve erken doyma şikayetiyle geliyor. Fizik muaye-
nede cilt ve mukozalar solukluk ve masif splenomegali sapta-
nıyor. Laboratuvarında hemoglobin 14 gr/dl, MCV 84 fi, lökosit
sayısı 148.000 mm 3, trombosit 440.000 mm 3, retikülosit %1
izleniyor ve bakılan periferik yaymada şekildeki gibi eritrositler
hipokrom normositer, %25 nötrofil, %19 çomak,% 18 metami-
yelosit, %22 miyelosit, %3 miyeloblast, %1Oeozinofil %3 bazofil
ol arak saptanıyor.

Vakadaki hastanın kesin tanısını koymak için hangi test


yapılmalıdır?
A. Kemik iliği biyopsisi B. Sitogenetik inceleme
C. Protein elektroforezi D. LAP skorunun ölçümü
E. Lenf nodu biyopsisi

36- Aşağıdakilerden hangisinde PDGFR a mutasyonu vardır?


A. Kronik miyelomonositik lösemi
B. Kronik eozinofilik lösemi
C. Büyük granüler lenfositik lösemi
D. Thücreli prolenfositik lösemi
E. Bhücreli prolenfositik lösemi
37• 78 yaşında meme kanseri nedeniyle takipli kadın hasta
halsizlik ve sol üst kadran ağrısı nedeniyle başvuruyor. Yapılan
fizik muayenesinde trabue alanı kapalı olduğu ve hepatomegali
varlığı tespit ediliyor. Hemogramında lökosit 24.000/mm 3,
hemoglobin 1Ogidi, trombosit 44.000/mm 3 izleniyor.Periferik
yayma çok sayıda aşağıdaki gibi hücre izleniyor.

Bu hastanın en olası tanısı hangisidir?


A. Primer miyelofibrozis B. Miyelofizitik anemi
C. Akut myeloid lösemi D. Aplastik anemi
E. Kronik lenfositik lösemi

38· 67 yaşında kadın hasta epistaksis şikayetiyle geliyor. Fizik


muayenede splenomegali izlenen hastaya 3 ay önce rutin
kontrollerinde trombositoz izlenmesi üzerine aspirin reçete
edilmiş. Bakılan hemogramında hemoglobin 15 gr/dl, lökosit
12000/mm3, trombosit sayısı 1.200.000/mm 3 olarak saptanıyor.
Hastada ön tanı olarak esansiyel trombositoz düşünülüyor. Bu
hastayla ilgili aşağıdaki yorumlardan hangisi yanlışt ır?
A. Kemik iliğinde megakaryosit proliferasyonu izlenir
B. Trombosit sayısının 450 bin üzeri olması major tan ı kriteridir
C. Demir eksikliği anemisiyle ayırıcı tanıya girer
D. CALR pozitifliği major tanı kriterleri arasında yer alır
E. En karakteristik bulgusu pleatoredir

39- Altmış yedi yaş ın da bir erkek hasta sırt ağrıs ı ve halsizlik
yakınmalarıyla başvuruyor. Fizik muayenede solukluk d ı şında
bulgu saptanmıyor. Laboratuvar incelemeleri sonucunda,
hemoglobin: 8.5 g/dl, eritrosit sedimentasyon hızı : 103 mm/
saat, BUN: 35 mg/dl, kreatinin: 1.9 mg/dl, total protein: 9.2 g/
dl, albümin: 1,8 g/dl, kalsiyum: 11.5 mg/dl olarak bulunuyor;
idrar analizinde ise (+ ) proteinüri saptanıyor. Ayrıca kranial
grafide şekildeki gibi litik lezyon la izleniyor.

Bu hastada kesin tanıya yönelik istenmesi gereken incele-


me aşağ ıdakilerden hang isi olmalıdır?
A. Uzun kemikl erin direkt grafil eri
B. ~2 mikroglobulin düzeyi
C. Serum ve idrar elektroforezi
D. Protein elektroforezi
E. Kemik iliği biyopsisi
40- 67 yaşındaki kadın hasta; 8 ayda 15 kilo kaybetme, halsizlik
ve yaygın ağrı yakınmalarıyla başvuruyor. Öyküsünden son
birkaç gündür devam eden poliüri ve konstipasyon şikayetleri
olduğu da öğreniliyor. Hastanın fizik muayenesinde, oturur-
ken sırt ve belinde ağrı ile hareketlerinde kısıtlanma dışında
özellik tespit edilmiyor. Laboratuvar incelemelerinde; eritrosit
sedimentasyon hızı: 84mm/saat, CRP: 49 g/L, fosfor: 4, 1 g/dl,
albüm in: 3,5 g/dl ve kreatinin: 2,1 mg/dlsaptanıyor. Çekilen
EKG'de şekildeki gibi OT mesafesinden kısalma izl eniyor.

-I--J.~-~~-ır-ır-~,TILU
I • 4'11< ' 1 1 ,V' V<

IJLı._
· ~~~--J .JL-_l
11

- •··· . . ~--··l····
;

~;_~~{~~
' . 4

: .:
, ,

••
aw,

•••
,

' •
vı , ,

r-~-
. , ,

ı _~LL~
\'5 ,

••••

Bu hasta tetkiklerden hangisi öncelikle istenmelidir?


A. Abdominal ultrasonografi
B. Serum lg Gdüzeyi
C. Serum kalsiyum düzeyi
D. Spot idrar analizi
E. Tam idrar analizi

41- 54 yaşında multiple miyelom nedeniyle takipli hastada ani


ge li şen görme kaybı, baş ağrıs ı, baş dönmesi ve kulak çın l aması
ş ikayet iyl e getiriliyor. Yapılan fundoskopik muayene retina! ka-
namalarla uyumlu lezyonlar ve eksuda lar izleniyor. Bu hastada
en olası tanıya yönelik ilk yapılması gereken aşağıdakiler­
den hangisidir?
A. Plazmaferez
B. Diyaliz
C. Kortikosteroid
D. Albumin replasmanı
E. İntravenöz immünglobulin

42- 71 yaşındaki erkek hastanın genel kontrol amaçlı inceleme-


leri sırasında sedimentasyon yüksekliği saptanması nedeniyle
yapılan ileri incelemelerinde; hemoglobin 12,5 g/dL, serum
kreatinin 0,8 mg/dl ve serum kalsiyu-m 8,9 mg/dl ölçülüyor.
Serum protein elektroforezinde gamada monklonal artış (2, 1 g/
dl) gözleniyor. Serum immünofiks~syon elektroforezinde lgG
kappa monoklonal protein saptanıyor ve serbest lambda/kappa
hafif zincir oranı 4 bulunuyor. Bak ıl an kemik iliği inceleme-
sinde plazma hücresi oranı% 4 bulunuyor. Manyetik rezonans
görüntüleme yöntemiyle gerçekleştirilen tüm vücut kemik
değerlendirmesinde lezyon saptanmıyor. Bu hasta için en olası
tanı aşağıdakilerden hangisidir?
A. Multiple miyelom
B. Smoldering miyelom
C. Önemi bilinmeyen monoklonal gamopati
D. So liter plazmositom
E. Waldenstrom makroglobulinemisi
43- 54 yaşındaki kadın hastanın rutin sağlık kontrolü sırasında,
eritrosit sedimentasyon hızı: 66 mm/saat bulunuyor. Sedimen-
tasyon yüksekliğine yönelik yapılan tetkiklerinde sonuçları
şöyledir: Hemoglobin: 13 g/dL, lökosit: 7500/mm\ trombosit:
198.000/mm 3, BUN: 15 mg/dL, kreatinin: 0.6 mg/dl; serum
protein elektroforezinde gama bölgesinde monoklonal band
(3,2 g/dl) izleniyor. Yapılan kemik iliğinde şekildeki gibi plazma
hücre oranı% 12; kemik taraması normal izleniyor

Bu hasta için en olası tanı aşağıdakilerden hangisidir?


A. Smoldering myelom B. Benign monoklonal gamopati
C. Multipl miyelom D. Soliter plazmasitom
E. POEMS sendromu

44- 21 yaşındaki kız çocuk vücudundaki morluklar ve tekrar-


layan burun kanamaları nedeniyle getiriliyor. Öyküsünden ADP Rese tör ADP
~
anne ve babasının akraba olduğu öğreniliyor. Kan sayımında
trombosit sayısı 384.000/mm 3 saptanıyor. Periferik yaymasında
trombosit ve eritrosit normal morfoloji ve sayıda görülüyor. Bu ~
hasta için en olası tanı aşağıdakilerden hangisidir?
A. Bernard-Soulier sendromu
B. İmmün trombositopenik purpura
C. Linezolid kullanımı
D. Glanzman sendromu
E. Trombotik trombositopenik purpura
... ~ ..
vWF

Subendotelyal
Kollajen
Eksiklik:

45- Hemofili Atedavisi için faktör 8 başlanan hastada F8'e karşı


direnç geliştiğinde hangi monoklonal antikor verilebilir?
A. Eculizumab B. Emisuzumab !!l EMİÖZUMAB

C. Ravulizumab D. Andexanat
E. Crizanlizumab

46· Burun kanaması ve peteşiler nedeniyle acil servise getirilen


34 yaşında bir kadın hastanın tam kan sayımında hemoglobin
düzeyi 143.5 g/dL, lökosit sayısı 9800/mm 3, trombosit sayısı
17000/mm 3 olarak saptanmıştır. Yapılan diğer biyokimyasal
testleri normal sınırlar içinde bulunmuştur. Bu hasta için tanıya
yönelik bu aşamadan sonra ilk yapılması gereken aşağı­
dakilerden hangisidir?
A. Periferik yayma
B. Hemogram tekrarı
C. Ristosetin agregasyon testi
D. ADP-kollagen adezyon testi
E. Kemik iliği aspirasyon bozukluğu
47· Akut promiyelobasltik lösemi nedeniyle takip ve tedavisi
yapılan hasta yoğun bakım yatışının 6. gününde bacaklarda ge-
lişen morluk üzerine tetkik ediliyor. Detaylı bakılan laboratuvar
incelemede kanama zamanı, aPTT, PT ve trombin zamanı uzun
izleniyor. Hemogramda trombosit sayısı 27.000/mm 3 izlenen
hastada fibrinojen düşük izleniyor. D-dimer artmış izlenen
hastanın bakılan periferik yayma şekildeki gibi izleniyor.

I\

Fibrin yıkım ürünleri de artmış hastada en olası tanı aşağıda ­


kilerden hangisidir?
A. İmmün trombositopenik purpura
B. Dissemine intravasküler koagülasyon
C. von Willebrand eksikliği
D. Trombotik trombositopenik purpura
E. Hemolitik üremik sendrom

48· Daha önce sağlıklı olan 34 yaşındaki erkek hasta, 1Ogün


önce başlayan kanlı ishal ve ateşin ardından gelişen halsizlik
ve idrar miktarında azalma yakınmalarıyla başvuruyor. Fizik
incelemesinde kan basıncı: 130/90 mmHg ve nabız: 102/
dakika ölçülüyor. Konjunktivalarında solukluk ve spontan diş eti
kanamaları dışında başka bir bu lguya rastlanmıyor. Hasta, son 2
gündür ishalinin olmadığını ifade ediyor. Laboratuvar incele-
melerinde serum kreatinin: 7,8 mg/dL, LDH: 2.800 U/L, total
bilirübin: 3,5 mg/dL, indirekt bilirübin: 3,1 mg/dL,AST: 98 U/L,
ALT: 84 U/L, hemoglobin: 7 g/dL, trombosit: 21.000/mm 3 olan
hastada periferik yaymada şistositler saptanıyor. ADAMTS-13
enzim aktivitesi: %95 ve ANA negatif bulunuyor. Gaita kültürün-
de Escherichia cali O157:H7 üremesi saptanıyor. Bu hasta için
en olası tanı aşağıdakilerden hangisidir?
A. İmmün trombositopenik purpura
B. Dissemine intravasküler koagülasyon
C. von Willebrand eksikliği
D. Trombotik trombositopenik purpura
E. Hemolitik üremik sendrom
49- 42 yaşında erkek hastada ishal gelişiyor ve sonrasında şuur
bulanıklığı, idrar miktarında azalma saptanıyor. Fizik muaye-
nede kan basıncı 140/90 mmHg, vücut sıcaklığı 39.6 °C, nabız
108/dakika olarak bulunuyor. Yapılan incelemelerde hemoglo-
bin düzeyi 6 g/L, lökosit sayısı 10.500/mm3 ve trombosit sayısı
41.000/mm 3 olarak bulunuyor. Serum kreatinin düzeyi 3.3 mg/
dl olan hastada idrar tetkikinde 3 g/gün proteinüri bulunuyor.
Serum LDH düzeyi 1950 U/L, protrombin zamanı ve aktive
parsiyel tromboplastin zamanı normal olan hastanın periferik
yaymasında şekildeki gibi fragmante eritrositler gözleniyor

:::-
Bu hasta için en olası tanı aşağıdakilerden hangisidir?
A. İmmün trombositopenik purpura
B. Dissemine intravasküler koagülasyon
C. von Willebrand eksikliği
D. Trombotik trombositopenik purpura
E. Hemolitik üremik sendrom

50- 45 yaşında erkeke hastanın koroner angiografi yapıldıktan


1 hafta sonraki rutin kontrollerinden trombosit sayısı 13.000
izleniyor. Vakadaki hasta hakkında hangi yoru doğru
değildir?
A. Heparin kesilmeli ve düşük molekül ağırlıklı heparin başlan­
malı
B. Vaka heparinin indüklediği trombositopeni tip 2'dir
C. En özgün tanısal test seratonin salınım testidir
D. Hastalık heparin-platelet faktör 4 antikoruyla ilişkilidir
E. Vakaya antikoagülan olarak dabigatran verilebilir
ONKOLO:)İ
1- Kanser gelişiminde önemli bir rol oynayan siki inler hakkında
aşağıdakilerden hangisi yanlış bir yorumdur?
A. Siki inlerin oluşma sırası D-E-A-B şeklindedir
B. G1 aşamasından Saşamasına geçişi siklin Dsağlar
C. Siklin D nin etkilerine CDK 1 ve 2 aracılık eder
1

D. Hücre bölünmesinde mutasyonlar p53 tarafından durdurulur


E. Mutasyon sonucu Rb defosforile olur ve siki us durdurulur

2- Teyzesini ve annesini meme kanseri nedeniyle kaydeben 30


yaşındaki
1
biri Google dan BRCA-1 mutasyonunun ailesel meme
kanserine yol açabileceği bilgisine ulaşıyor ve kendisinde de bu
mutasyonun olup olmadığına bakmak istiyor. Genetik hekimi
Yavuz hoca hastanın bu isteği için a ldığı örnekte hangi kromo-
za bakmal dır?
A. Kromozom 11
B. Kromozom 13
C. Kromozom 15
D. Kromozom 17
E. Kromozom 19

3-Aşağ ıdakiberlirteçlerden hangisi bir tümörün nöroendok-


rin kökenli olduğunu gösterir?
A. Sitokeratin
B. Sinaptofizin
C. TTF-1
D. Napsin A
E. CEA

4- Aşı ile önlenebilen kanserler hangi seçenekte doğru


olarak veri lmiştir?
A. Hepatosellüler kanser -- Serviks
B. Endometrium --Akciğer
C. Özefagus -- Pankreas
D. Mide -- Kolon
E. Over -- Serviks

5- Aşağıdakilerden hangi seçenekte virüs ve sebep olduğu


kanser yanlış eş le şt irilmi ştir?
A. HCV -- Sp lenik marjinal zon lenfoması
B. HHV-8 -· Anjiofoliküler lenf nodu hiperplazisi
C. Herpes sip leks virüs tip 2 --Serviks
D. HBV -- Hepatosellüler kanser
E. H.pylori -- Mide kanseri
6- Bir patolog elindeki dokuyu mikroskopta incelerken nükleer IEpitr.ell OCe:rıa'fİlını (kaılfSÜ1!oım)
ve hücresel anlamda boyut, şekil değişikliği ve normal doku Bağ) dlokuısuı \fime:ırııtıılill, v/W/f, CD 31, r?ECAM (sarkom}
organizasyonunda kayıp görüyor. Elindeki dokuda malign hücre !Kas [)es;miını, SMA (sarkom)
odakları da gözlemleyen patolog dokunun desmin (+)boyandı­ ~lörıa il dıohıı 6H'(P !Nleuı rNl,. l4180i/Mac-t INIS[
0

ğını tespit ediyor. Patolog un elindeki neoplazik hücre hangi Meıarıııom S.1 O'O, frl MB ~-5
organa ait olabilir? Ge·rım ~üue AfP~ b-frl(G
A. Cilt Hematropoıetrık rn 451,CD 4ll ([)3' (lemıfoma)
B. Kas dokusu K iı 67 Pmiııfarn:s.yoıırıı gıöSıtergıesi

C. Sinir dokusu
D. Bağ dokusu
E. Epitel dokusu

7- Serviks kanser taraması hakkında hangisi doğru değildir?


A. 21 yaşından önce tarama önerilmez?
B. 24 yaşındaki bir kadın 3 yılda bir pap-smear ile taranır
C. 38 yaşında kadın 5 yılda bir pap-smear ve HPV DNA ile taranır
D. Kanser dışı nedenle histerektomi olan 5 yılda smearla taranır
E. 70 yaşında önceki tarama normalse kişi taramadan çıkarılır

8- 51 yaşında kadın hasta karın ağrısı, karında şişkinlik hissi


ve hazımsızlık şikayetiyle başvuruyor. Hastaya çekilen batın BT
sonucunda overde sınırları düzensiz 3x6 cm boyutunda kitle iz-
leniyor. Kitleden yapılan biyopsi sonucu over kanseri tanısı alan
hastaya 6 kür karboplatin + paklitaksel tedavisi uygulanıyor.
Ardından kontrol tomografisinde tümör boyutunda %56 azalma
olduğu izleniyor. Bu hastanın tedavi yanıtı için en doğru
yorum hangisidir?
A. Tam yanıt
B. Parsiyel yanıt
C. Progresif hastalık
D. Stabil hastalık
E. İnkomplet yanıt

9a 45 yaşında erkek hasta acil servise halsizlik ve yüksek ateş CK 7 (+) / CK 20 (·) CK 7 (·)/ CK 20 (+)
şikayetiyle geliyor. Acil serviste bakılan hastanın hemogramında Akciğer adenokarsinomu Kolorektal kanser
pansitopeni izlenmesi üzerine kemik iliği aspirasyon biyopsisi Meme karsinomu Merkel hücreli kanser
yapılıyor. Biyopsi adenokarsinom metasta z ı olarak rapor edili- Tiroid karsinomu
yor. Patoloji raporunu detaylı inceleyen dahiliye hekimi raporda Endometrium karsinom
tümör dokusunun CK 7 (-), CK 20 (+)boyandığını görüyor. Serviks kanseri
Bu aşamadan sonra aşağıdakilerden hangisinin yapılm a sı en Tükrük bezi tümörleri
doğru yorumdur?
A. Hastaya tiroid doppler USG yapılmalı CK7(-)/CK20(·)
B. Serum AFP ve hepatobiliyer USG bakılmalı
CK 7 (+) / CK 20{+) Hepatosellüler kanser
Üretelyal tümör Renal hücreli kanser
C. Kolonoskopik biyopsi uygulanmalı
Over müsinöz adenokars Prostat kanseri
D. Transvaginal USG ve detaylı jinekolojik muayene
Pankreas adenokarsinomu Skuamoz / Küçük Ac ca

..
E. Kontrastlı üst abdomen MR çekilmeli
Kolanjiosellüler kanser Baş-boyun kanserleri
10· Eğer vakanın CK 7 (-}, CK 20 (-) boyandığını rapor edilseydi
hangisinin yapılması en doğru olur?
A. Hastaya tiroid doppler USG yapılmalı
B. Serum AFP ve hepatobiliyer USG bakılmalı
C. Kolonoskopik biyopsi uygulanmalı
O. Transvaginal USG ve detaylı jinekolojik muayene
E. Kontrastlı üt abdomen MR çekilmeli

11 · Burkitt lenfoma nedeniyle takipli 34 yaşındaki erkek hasta-


ya kemoterapi başlanıyor. Kemoterapinin 3.gününde idrar çıkışı
Tümör Lizis Sendromu
azalıyor, ellerde ve ayaklarda paresteziler başlıyor. Hastanın
' / ÜrikAsit
yapılan fizik muayenesinde her iki bacakta gode bırakan +2 __., Potasyum

ödem ve göz altlarında da ödem lehine değerlendirilebilecek --- Fosfat

ş işlikler mevcut. Bakılan l aboratu varında serum kreatini:2,4


ı
Ksantin
mg/dl, üre:87 mg/dl, sodyum128 mEq/1, potasyum 6, 1 mEq/L, Oksidaz

kalsiyum 7,7 mg/dl ve FeNa değeri %3,2 izleniyor. Bu hastanın Hiperfosfatemi Hiperüri mi HiperKalemi

en olası tanısı aşağıdakilerden hangisidir?


İdrar Fosfat'!' İ drar Ürik Asit 1'
A. Perikardiyal tamponad
B. Prerenal böbrek yetmezliği Ca2)•
Ürik asit Taşı

-
Ca-PTaşı
C. Tümör lizis sendromu
D. Kontrast nefropati
Hipo Kal mi Akut Böbrek Yetmezliği
E. Postrenal böbrek yetmezliği

12- Vakanın tedavisi için en uygun olan hangisidir?


A. İdrar alkalizasyonu
B. İntravenöz hidrasyon + Allopürinol
C. Febuksostat
D. Allopurinol
E. Fosfor bağlayıcı reçine

13· 64 yaşındaki kadın hasta yüzünde şişlik nedeniyle baş­


vuruyor. Yapılan fizik muayenede juguler venlerde dolgunluk
izleniyor. Hastanın tansiyonu 100/80 mmHg ve hasta zaman
zaman dinlenmekle geçen göğüs ağrıların ın olduğunu tarif
ediyor. hastaya çekilen akciğer grafisinde akciğer apeksinde yer
kaplayan oluşum göze çarp ıyor. Bunun üzerine ileri tetkik amaç-
lı toraks tomografisi çekiliyor ve sağ juguler ven trasesindeki
lenf nodlarında şişlik ve bu lenf nodlarının juguler vene bası
yaptığı görülüyor. Bu aşamadan sonra tanısal anlamda ilk
yapılması gereken aşağıdakilerden hangisidir?
A. Akciğer grafisi
B. Bilgisayarlı tomografi
C. Lenf nodu biyopsisi
D. Ekokardiyografi
E. PET-CT çekilmesi
14- Yukarıd a ki hastaya biyopsi yapılıyor ve sonuç maligniteyle
uyumlu geliyor. Bu aşamadan sonra yapılması gereken
aşağıdakilerden hangisidir?
A. Akciğer grafisi
B. Bilgisayarlı tomografi
C. Lenf nodu biyopsisi
D. Ekokardiyografi
E. PET-CT çekilmesi

15- Aşağıdaki malignitelerden hangisinin hiperkalsemiye yol


açma mekanizması yanlış verilmiştir?
A. Multiple myelom -- Litik lezyonl ar üzerinden
B. Renal hücre li kanser -- Protaglandin salgısında art ı ş
C. Lenfoma -- Kalsitriol üretimi
D. Sq uamoz hücreli a kciğ er kanseri -- Dvitamini sentezi
E. Feokromasitoma -- PTHrP üretimi

16· Malign hiperka lsemide k u ll an ıl an ilaçlar ve etki mekaniz-


maları eşleştirmelerinden hangisi doğru değildir?
A. Steroid -- Osteoklast arac ılı kemik rezorpsiyonunu aza l t ır
B. Kalsitonin -Osteoblastik aktivasyonu art ırı r
C. Bifosfonatlar -- Osteoklastik aktivasyonu aza l t ı r
D. Denosumab -- Osteoklastik aktivasyonu aza l t ı r
E. ACTH -- Barsaktan Ca+ 2 geri em ilimini aza ltı r

17- Akut lenfob lastik lösemi neden iyle kemoterapi alan 45


yaşındaki kadın hasta tedavid en 4 gün sonra ateş ş ikayet iyl e
acil servise başvuruyor. Hastanın fizik muayenesinde tansiyonu
normal, genel durumu iyi ve 39°( ateş dışında başka hiçbir ek
patoloji tariflem iyo r. Acil serviste bakılan hemogramınd a lökosit
300/mm3, nötrofil 1OO/mm 3, hemglobin 8, 7 g/dl, trombosit
87.000/mm 3 izlen iyor. B iyok imyas ın da böbrek ve ka raciğer
fonksiyon testleri olağan s ını r l arda izlenen bu hastanın en
olası tanısı aşağ ı daki l erden hangisidir?
A. ALL nüksü
B. Santra l sinir sistem i metastaz ı
C. Febril nötropen i
D. Minimal rezidü hastalık
E. İnvaz iv pu lmoner aspergiliozis
18. 45 yaşındaki kadın hasta sağ memesinden ele gelen bir
kitle nedeniyle doktora başvuruyor. Kitleden yapılan biyopsi
sonucu hastaya invaziv duktal karsinom tanısı konuluyor. Takipte
nefes darlığı gelişmesi üzerine çekilen akciğer grafisi aşağıdaki
gibi izleniyor.

Maligniteye bağlı plevral efüzyon olduğu düşünülen bu hasta-


da efüzyona yönelik aşağıdaki diüretiklerden hangisinin
kullanımı önerilmez?
A. Furosemid
B. Hidroklortiyazid
C. Spironolakton
D. Amilorid
E. Triamteren

19. 43 yaşında akut miyeloblastik lösemi nedneiyle takipli


kasdın hastaya allogenik kemik iliği nakli yaplıyor. Nakil sonrası
nötropenik seyreden hastanın takipte 39° Cateşi oluyor ve eş
zaman lı bakılan nötrofil sayısı 100/mm 3 geliyor. Fizik muaye-
nesinin olağan olduğu intravenöz tedavi gerekliliği amacıyla
açılan kateter bölgesinin de kızarık olmadığı görülüyor. Önce-
den alınan kültürleri tarandığında spesifik bir etken üremesine
rastlanmıyor. Bu hasta için aşağıdaki antibiyotiklerden
hangisi ilk tercih olarak verilebilir?
1. Sefepim il. Seftazidim
111. Piperasilin / Tazobaktam iV. İmipenem / Silastatin
V. Meropenem VI. Aztrenonam
VII. Gentamisin vm. Vankomisin
A.1-11-111- IV-V B. 1- 11-111 -VI -VII
C.1- il - VI -VII -VIII D. Sadece VI 11
E. V-VI

20. 65 yaşında kadın hasta memede kitle nedeniyle tetkik edi-


lirken meme kanseri tanısı a lıyor. Tümör dokusunun inceleme-
sinde östrojen ve progesteron reseptörü ve HER2/Neu ekspres-
yonu pozitif izleniyor. Ardından hastaya doksorubisin, sisplatin
ve transtuzumab içeren bir kemoterapi rejimi başlanıyor. Bu
hasta için kullanılan ilaç beklenen yan etki eşleştirmelerin ­
den hangisi doğrudur?
1. Sisplatin İ ş i tme kaybı
il. Sisplatin - Sensörimotor nöropati
111. Doksorubisin Doz bağımlı reversible KMP
iV. Transtuzumab Doz bağımsız reversible KMP
A.1-11 B. 11 - 111
C. Sadece iV D.1 - 11 - iV
E.1 - 11 - 111
21. 66 yaşında bir kadın hasta şuur bulanıklığı nedeniyle acil
servise getiriliyor. Yakını tarafından bilinen ek bir hastalığı ve
ilaç kullanım öyküsü olmadığı öğrenilen hastada. serum kalsi-
yum düzeyi 16,5 gelince malign hiperkalsemi kabul edilerek
yoğun intravenöz hidrasyon ve furosemid başlanıyor. Çekilen
EKG'de şekildeki gibi OT süresinin 260 ms olduğu görülüyor.
' t'"' 1V,'ll
_A,.,..____.Jı,,,.......__-:-v~--~r------~V .\.___}V
/'~ t, ] '~I
j I jl

.ırı. v.ı ';Ş!

~~h-L-1~~~~01A.,,- J j___Jh_J
f I i il

= - w ~

-;Jl--------t1------1-~.L-uh~---~
l :1 1 l

Ardından dahiliye servisine yatırılarak ileri tetkik ediliyor ve


paratormon düzeyi düşük; parathormon ilişkili peptit düzeyi ise
yüksek geliyor. Bu hasta için aşağıdaki malignitelerden hangisi ---
malign hiperkalsemi sebebi olması beklenmez?
W
•·
A. Mesanenin transfüzyonel hücreli kanseri
B. Nazofarenks skuamoz hücreli karsinom
C. Akciğer skuamoz hücreli karsinom
D. Meme adenokarsinomu
E. Diffüz büyük Bhücreli lenfoma

22. Bir öndeki sorudaki hiperkalsemi tedavisi hakkında


hangisi yanlış bir ifad ir?
A. İlk olarak sal in infüzyonu başlanmalı ve ardından furosemid
eklenerek zorlu diürez indüklenmelidir
B. Pamidronat, etidronat ve zolendronat gibi bifosfanatlar verile-
bilir ancak etkileri 1-2 gün içinde ancak başlar
C. Bifosfanatlar serum kalsiyumunu 1-2 hafta boyunca baskıla­
yabilir
D. Prednizon gibi kortikosterodiler osteoklast aracı lı kemik
rezorpsiyonu engellemek amacıyla verilebilir
E. Kalsitonin renal kalsiyum atılımını artırmak ve kandan kemi-
ğe kalsiyum geçişini sağlamak için verilebilir

23. 65 yaşında mide adenokarsinomu sebebiyle takip edilen


hastaya dosetaksel, sisplatin ve 5-fluorourasil tedavisi başlanı­
yor. Ek olarak ateşi olunca parasetamol infüzyonu alan hastada
tedavi esnasında el-ayak sendromu geliştiği tespit ed iliyor. Bu
klinik tabloya aşağıdakilerden hangisinin yol açması en
olas dır?
A. Sisplatin
B. 5 -Fluorourasil
C. Parasetamol
D. Dosetaksel doz aşımı
E. Sisplatinin nefrotoksisitesi
la t,pd ► U
~ !!!I OKa ,emısı
24. Akciğer kanseri etiyolojisi ile ilgili aşağıdaki ifadeler-
den hangisi yanlıştır?
A. Aktif sigara içicilerin akciğer kanseri riski 20 kat
artmıştır

B. KOAH, akciğer kanseri riskini artırır


C. Sigara dumanındaki nitrozamin'in karsinogenezde
rolü olabilir
D. c-erbB-1 protoonkojenin overekspresyonu küçük
hücreli olmayan akciğer kanserlerinde% 65 - 90
arasındadır
E. EGFR geninde olan mutasyonlar kötü prognoz göstergesidir

25. 54 yaşında erkek hasta 3 yıl önce kolon kanseri nedeniyle


opere ediliyor. Ayrıca 5 yıl önce de tiroid kanseri sebebiyle total
tiroidektomi yapılan hastanın takipte tiroglobulin düzeyi artmış ,
izleniyor. Kontrollerinde çekilen akciğer grafisinde şekideki gibi
sol orta lobda yaklaşık 6 cm boyutunda bir kitle tespit ediliyor.

Metastaz olduğu düşünülen akciğer kitlesinden alınan biyop-


side aşağıdaki belirteçlerden hangisi tiroid / kolon kanser
metastazı ayrımında kullanılır?
A. Sitokeratin 6
B. TTF-1 (Tiroid transkripsiyon faktör-1)
C. Serum PSA düzeyi
D. Her-2 / neu
E. PD-L1 gen mutasyonu

26. Akut lenfoblastik lösemi tanısı konulan 34 ya ş ındaki kadın


hastaya tedavinin remisyon indüksiyon aşamasında daunoru-
bisin, vinkristin, prednizolon, L-asparaginaz, ve siklofosfamid
veriliyor. Tedavinin ardından takipte kan şekeri 240 mg/L
gelmesi üzerine tetkik ediliyor ve yapılan OGTT'de 2.saatte kan
şekeri 216 geliyor. Bu sonuca göre hastaya diabetes mellitus
tanısı konuluyor ve metformin başlanıyor. Bu hastada remis-
yon-indüksiyon aşamasında verilen ilaçlardan hangisi
buna sebep olmuş olabilir?
A. Daunorubisin
B. Vinkristin
C. Hiçbiri
D. L-asparaginaz
E. Siklofosfamid
27- Uygunsuz ADH sendromuyla ilişkili olabilen malignite
aşağıdakilerden hangisidir?
A. Timik karsinom
B. Üretelyal karsinom
C. Küçük hücreli akciğer kanseri
D. Plevra! mezotelyoma
E. Germ hücreli tümör

28. Sekiz ay önce renal hücreli kanser sebebiyle nefrektomi


yapılıp kardeşinden böbrek nakli yapılan ve tacrolimus kullan-
makta olan 43 yaşındaki erkek hasta koyu renkte idrar çıkarma
yakınması ile getiriliyor. İdrar tetkikinde bol eritrosit olduğu
tespit ediliyor. Tam kan sayımında hemoglobin 6, 7 g/dl, hema-
tokrit %19, trombosit sayısı 187.000/mm 3 saptanıyor. Yapılan
sistoskopide şekildeki gibi epitelin yerini telenjiekta zi k subepi-
telyal damarların aldığı rapor ediliyor.

Bu tabloya neden olabilecek en olası etken aşağıdakiler­


den hangisidir?
A. Herpes virüs tip 1
B. Parechovirüs
C. Echovirüs
D. BK virüs
E. Coxsackievirüs

29. Vakadaki hasta için hangi anti-viral tedavisinin etkinli-


ği gösterilmişt ir?
A. Vidarabin
B. Asiklovir
C. Gansiklovir
D. Tenofovir
E. Valasiklovir

30- Bir hekim febri l nötropenide MASCC skoru hesaplayarak


düşük/yüksek risk ayrım ını yapmak istiyor. Aşağıdakilerden
hangisi bu skorlama sistemi içinde yer almaz?
A. Nötropeninin derecesi
B. Hastanın yaşı
C. Solid tümör h astas ı olup olmaması
D. Hipotansiyon
E. Kronik obstrüktif akc iğ er hastalığı olup olmaması
a
l~ !!!!!! tı~ d . .
aKa emısı
1
...
~}i'ı.iP""1~.i~~~ıil~~~~".~r'l.z~~~'.'S'.7Jx~~0EGJl}Vi-;;0::2
~~___.
_,........,.-..:,.___________....;___
\}Y!;1\W!f.i\ti~i~d{)1ıittl~itt!

31· Nötropenik ateş aşağıdakilerden hangisi vankomisin


ekleme endikasyonu değildir?
A. Kateter enfeksiyonu veya kateter enfeksiyonu varlığı şüphesi
B. Ağır mukozit varlığı
C. Gram (+)üremesi
D. Gram(-) enfeksiyon varlığı şüphesi
E. Hipotansiyon

32. 23 yaşında erkek hasta sağ testisinde şişlik sebebiyle dok-


tora başvuruyor. Yapılan fizik muayenede sağ testiste ele kitle
gelmesi üzerine yapılan doppler USG'de şekildeki gibi vasküler
ak ımda artış izleniyor ön planda testis kanseri düşünülerek
orşektomi yapılıyor.

Testis kanseri tanısı konulan hastaya bleomisin içeren kemotera,-


pi rejimi başlanıyor. Hastayı takip eden onkolog bleomisindeiı
tam yanıt almak istiyorsa tedavi öncesi aşağıdakiler­
den tetkilerden hangisini istemelidir?
A. Hemoglobin elektroforez
B. 812 vitamini ve folat düzeyi
C. Ferritin, hemogram, periferik yayma
D. Kolonoskopik biyopsi
E. Fekal kalprotektin düzeyi

33. Vakada onkolog demir eksikliği açısında hastayı tetki


ediyor ve gerekli demir replasman ını yaptıktan sonra tedaviye
başl ıyor. İlerleyen süreçte nefes darlığı, egzersiz kapasitesinde
kısıtlılık ve çabuk yorulma izlenirse aşağıdakilerden hangisi-
nin geliştiği düşünülmelidir?
A. Bronşektazi
B. Pulmoner fibrozis
C. Ferritin, hemogram, periferik yayma
D. Kolonoskopik biyopsi
E. Fekal kalprotektin düzeyi

34- Jinekomastiye yol açabilen tümör hangi seçenekte verilmiş ­


tir?
A. Timik karsinom
B. Üretelyal karsinom
C. Küçük hücreli akciğer kanseri
D. Plevra ! mezotelyoma
E. Germ hücreli tümör
35- 64 yaşında kadın hasta sağ memede kitle nedeniyle dokto-
ra başvuruyor. Kitleye yönelik yapılan mamografi BIRADS 4 ola-
rak yorumlanıyor. Bu aşamadan sonra hastaya biyopsi yapılıyor
ve invaziv lobüler karsinom rapor ediliyor. Çekilen tüm vücut BT
ve PET/CT incelemede uzak organ metastazı izlenmeyen hastaya
radikal mastektomi yapılıyor. Mastektomi sonrası tümör çapı 3.2
cm, östrojen, progesteron reseptörü ve HER-2 negatif geliyor.
Aksiller diseksiyonda ise 2 tane lenf nodunda metastazla uyum-
lu bulgular elde ediliyor. Bu hasta için aşağıdaki tedavilerden
hangisi en uygundur?
A. Tamoksifen
B. Pertuzumab
C. Transtuzumab
D. Sistemik kemoterapi
E. Ekzemestan

36- Yukarıdaki vaka östrojen, progesteron reseptörü nega•


tif; HER-2 pozitif gelseydi hangi ajan verilebilirdi?
A. Tamoksifen
B. Pertuzumab
C. Anastrazol
D. Fu lvestra nt
E. Ekzemestan

37• Vakadaki hasta için en önemli prognostik kriter aşağ ı ­


dakilerden hangisidir?
A. Östrojen ve progesteron reseptör negatifliği
B. HER-2 negatifliği
C. Aksillada 2 adet metastatik lenf nodu varlığı
D. Tümör çapının 3,2 cm olması
E. Hastanın yaş ı
38- Sisplatin kullanımı sonucu görülen en sık yan etki
hangisidir?
A. Bulantı-kusma
B. Nefrotoksisite
C. Kardiyotoksisite
D. Nörotoksisite
E. Kemik iliği supresyonu

39- Sisplatinin nefrotoksisite yapıcı etkisini engelle-


mek için hangi ilaç verilmelidir?
A. Lökoverin
B. Amifostim
C. Dekstrazoksan
D. Glukarpidaz
E. Mesna

40- c- KİT mutasyonu izlenen hastaya GİSTtanısı konuluyor ve


imatinib mesilat veriliyor ancak yanıt alınamıyor. Daha sonra
sunitibin tedavisine başlanıyor ancak yine yanıt alınamıyor. So-
nuçta hem imatinib mesilata hem de sunitinibe yanıtsız
bu hastada bu aşamadan sonra hangi ilaç verilebilir?
A. İnterferon - alfa
B. Sistemik kemoterapi
C. Yüksek doz imatinib mesilat
D. Sun itinib
E. Regorafenib

..
KARDİYOLO:>İ
1- 81 yaşındaki kadın hasta nefes darlığı şikayetiyle acil servise Kompansatuvar mekanizmaların olumsuz etkileri:
başvuruyor. Öyküsünde nefes darlığının yatarken arttığını ifade
• RAAıS a lkfüıasyoırm Srıvıı rretansfıyonu
ediyor ve özgeçmişinde iskemik kalp hastalığı olan hastanın
fizik muayenesinde dinlemekle her iki akciğer bazal inde ince • Sempatiık uıyar~ arıt.ıı şıı Ta.şiıkardfı
raller duyuluyor. Ekstremitelerinde periferik ödem tespit edili- • Hiip1 erıtmff~ Rö l atiıf iıslkemiı ve ar~tmiı
yor. Akciğer grafisinde kalp gölgesinde genişleme saptanıyor.
Çekilen akciğer grafisi de şekildeki gibi izleniyor.

Aşağıdakilerden hangisi uzun dönemde hastalığın progresyo-


nunu hızlandıran kompansasyon mekanizmaların dan U)
biri değildir?
A. Kardiyak hipertrofi
B. Renin-angiotensin aldosteron aktivasyonu
C. Pulmoner venöz basınçta artış
D. Renal ve vasküler nörohümoral değişiklikler
E. Sempatik uyarı artışına bağlı taşikardi

ACC / AHA Kalp Yetersizliği Evreleri


2- Maksimum tıbbı tedaviye rağmen yapısal kardiyak anor- Evre A: Yapısal ya da işlevsel anormallik yoktur.
malliği devam eden ve dinlenme halinde bile kalp yetmezliği Semptom veya bulgu yoktur
semptom ve bulguları olan hastanın kalp yetmezliği evresi
Evre B: Kalp yetmezliğine yol açabilecek kardiyak
aşağıdakilerden hangisidir?
patoloji var ancak semptom ya da bulgu yoktur
A. Evre A
B. Evre B Evre C: Semptomatik kalp hastalığı ve altta yatan
yapısalkardiyak patoloji vardır
C. Evre C
D. Evre D Evre D: Maksimum tıbbı tedaviye rağmen kardiyak
E. Bu hastada kalp yetmezliği yoktur patolojisi devam eden ve dinlenme halinde de kalp
yetmezliği semptom ve bulguları vardır

3- 12 ay önce bir anterior miyokard enfarktüsü geçiren 57


yaşındaki bir hasta rutin takip için geliyor. Bakılan vitallerinde
kalp hızı 65 atım/dakika ve kan basıncı 104/82 mmHg izleniyor.
Herhangi bir şikayeti olmayan hastanın aspirin, klopidogrel,
atorvastatin, karvedilol ve valsartan kullandığı öğreniliyor.
EKG si sinüs ritminde izleniyor ve yapılan ekokardiyografide
1

sol apekste anevrizma ile uyumlu kontraksiyona katılmayan bir


lezyon izleniyor ve ejeksiyon fraksiyonu% 25 hesaplanıyor. Bu
hastanın ölüm oranını azaltmak için aşağıdaki tedavilerden
hangisi endikedir?
A. İvabradin
8. Kardiyak açık cerrahiyle restendleme
C. İmplantab'le kardiyoverter defibrilatör (ICD)

..
D. Digoksin başlanması
E. Antikoagülan tedavi
'2023 · MART-rus·s0Rü KAMPı:' : · .- ; -: .- --. · · ·
l a tı~d
l.!!!ı~ OKa
.;,
emısı 'ôah·m·e, , - .\· :>' _::· --/~': .. _ .. _ -~ ~:~_·_· -:, , ·.... -.. _· . _·>· __ -
4- Digoksin intoksikasyonu için aşağıdakilerden hangisi yanlış
bir ifadedir?
A. İlk yapılması gereken ilacın kesilmesidir
B. Geçici kalp pili takılabilir
C. Ventriküler ekstrasistol gelişirse lidokain verilebilir
D. En etkin ted avi Fab (digibind) tedavisidir
E. Digibind yanıtsız vakalarda acil diyaliz uygulanır

5- 62 ya ş ında erkek hasta çok ajite bir şekilde acil servise geti-
riliyor. Bakılan vitallerinde nabzı 101 atım/dk, solunum sayısı
44 soluk/dakika, tansiyon 180/120 mmHg izleniyor. Bakılan
kan gazında pH:7.1, pO 2:64 mmHg, pC0 2: 58 mmHg izleniyor.
Çekilen akciğer grafisinde bilateral plevra! efüzyon ve diffüz
infiltrasyon izleniyor.
.,,?:.,.
,'\

Hastaya akut akciğer ödemi tan ısı konuluyor. 1Olt/dk'dan


oksijen açılıp 1 ampul/h'den furosemid infüzyon tedavisi baş­
lanıyor. Yapılan tüm müdahalelere rağmen hasta exitus oluyor.
Bu hastada aşağıdaki verilen tedavilerden hangisi buna
sebep olmuş olabilir?
A. Klortalidon
B. Levosimendan
C. Tolvaptan
D. Mannitol
E. İntravenöz morfin

6- 71 yaşında bilinen böbrek yetmezliği olan obez kadın hasta


nefes darlığı ile acil servise başvuruyor. Bak ılan laboratuvard a
NT-proBNP düzeyi yüksek olarak bulunuyor. bulunuyor. Hangisi
bu NT-proBNP yüksekliğinin sebebi olamaz?
A. İl eri yaş
B. Obezite
C. Böbrek yetmez liği
D. Sağ ventikü l yetmezliğin e yol aç mış pu lmoner emboli
E. Kadın cinsiyet

7- İl eri evre kalp yetmezliğinde loop diüretiklerine yantsızlık söz


konusuysa aşağ ıdakil erden hangisini yapm ak uygun olmaz?
A. Tuz kısıtlamasını artırmak
B. Diüretiği infüzyon şek lind e vermek
C. Loop diüretiğini kesip hidroklortiyazid baş l amak
D. Hastayı hemod iyalize almak
E. Tedaviye başla bir diüretik eklemek

..
8- 56 yaşında erkek hastada ataklar halinde gelen çarpıntı,
yüzde solukluk ve hipertansiyon nedeniyle başvuruyor. Fizik
muayenede tansiyonu 190/130 mmHg izlen iyor. Ayrıca sık sık
terlediği ve 2 ay önce diyabet tanısı nedeniyle metformin kul-
landığı öğreniliyor. Sağlık bakanlığı sisteminden hastanın eski
epikrizlerine bakan doktor hastanın 2007 yılında kolesistektomi
ameliyatı esnasında kan basıncında oynamalar olduğu, ame-
liyatın güç bela bitirildiği ve tüm ameliyat boyunca hastanın
hipertansif seyrettiğini farkediyor. Hastaya çekilen adrenal BT'de
3,2 cm çaplı heterojen kitle rapor ediliyor. Vakadaki hastanın
tedavisi için en uygun antihipertansif hangisidir?
A. Epleronon
8. Triamteren
C. Metoprolol
D. Labetolol
E. Diltiazem

9- 23 yaşında bir kadın hasta 4 ay önce baş ağrıs ı nedeniyle


başvurmuştur. Tekrarlanan ölçümlerde kan basıncı 190-
100/100-120 mmHg olarak bulunmuştur. Hipertansiyon tanısı
konularak karvedilol tedavisine başlanmış ancak tedaviye yanıt
aiınamamıştır. Öyküsünde başka bir hastalık ya da ilaç kullanımı
olmayan hastanın aile öyküsünde hipertansiyon yoktur. Fizik
muayenede kan basıncı 190/120 mmHg ve evre 3 hipertansif
göz dibi değişiklikleri dışında hasta normal olarak bulunuyor.
Beden kitle indeksi 21,9 kg/m 2 olan hastanın yapılan labora-
tuvar incelemelerinde tam kan ve tam idrar tetkikleri normal
sınırlarda, açlık kan şekeri 85 mg/dl, serum kreatinin düzeyi 0.9
mg/dl, serum potasyum düzeyi 4.4 mmol/Lolarak bulunuyor.
Bu hastada kesin tanıya yönelik istenilmesi gereken tetkik
aşağıdakilerden hangisidir?
A. Ekokardiyografi
8. Tora kal anjiografi
C. Renal arter doppler ultrasonografi
D. Renal anjiyografi
E. Transözofageal ekokardiyografi

10· 56 yaşında daha önce sağl ıklı olduğu bilinen bir erkek
hastada kan basıncı 190/120 mmHg olarak bulunuyor. Göz
dibi incelemesinde belirgin bir patoloji saptanmayan, böbrek
fonksiyonları ve EKG si normal olan hastaya yapılması gereken
en uygun yaklaşım aşağıdakilerden hangisidir?
A. Hastayı yatırarak nitroprussid ile kan basıncının düşürülmesi
8. Dil a ltı nifedipin uygulaması
C. Yaşam tarzı değişik li ği
D. Yaşam tarzı değişikliği yanı sıra antihipertansif tedavi baş l an­

..
ması

E. Hastaya ambulatuvar kan basınc ı izlemi yapılması


g !l!!!I t,~d
l
I.!!!! OKa emısı
► "

11· Dört yıl önce koroner by-pass operasyonu geçiren 78


yaşındaki erkek hasta; son 4 aydır başlayan, 170/120 mmHg
düzeyine ulaşan kan basıncı yüksekliği ve akciğer ödemi
tablosuyla birkaç kez acil servise başvuruyor. Aktif olarak sigara
içen ve diabetes mellitusu olan hastanın hipertansiyon öyküsü
bulunmuyor. Fizik muayenesinde kan basıncı 180/120 mmHg
ve kalp hızı 78 atım/dakika ritmik tespit ediliyor. Karında sol
paraumbilikal bölgede sistolik üfürüm duyuluyor. Serum
potasyum düzeyi 3,2 meq/L bulunuyor. Hastaya çekilen doppler
incelemede şekildeki gibi orta renal arterde aşağı yönlü türbü -
lan akım rapor ediliyor.

Bu hasta için en olası tanı aşağıdakilerden hangis"dir?


A. Renovasküler hipertansiyon
B. Primer hiperaldosteronizm
C. Aort koarktasyonu
D. Steroid cevaplı hiperaldosteronizm
E. Hiperparatiroidizm

12- 4 yıldır sigara kullanımı ve hiperlipidemi öyküsü olan 56


yaşındaki erkek hasta yaklaşık iki aydan beridir eforla artan ne-
fes darlığı şişkayetiyle geliyor. Öyküsünün dinlenmekle geçtiği
ve yoğun egzersizle yeniden başladığı öğreniliyor. Bu hasta için
en olası tanı aşağıdakilerden hangis"dir?
A. Ventriküler taşikardi
B. Stabil angina pektoris
C. Varyant angina
D. Akut perikardit
E. Unstabil angina pektoris

13- Vakadaki hastada tanıya yönelik ilk yapılması gereken


aşağdak il erden hangisidir?
A. Troponin Tve CK-MB bakılması
B. NT pro-BNP bakılması
C. Düz batın grafi
D. Ekokardiyografi
E. Eforlu ekokard iyografi

14- Stabil angina pektoriste kan basıncını düşürmeden


antianginal etkinlik sağlayan ilaç hangisidir?
A. Rana lozin B. Trimetazidin
C. Nikorandil D. Diltiazem
E. İvabradin
15- Aşağıdakilerden hangisi ya da hangileri akut koroner send-
romu ifade eder?
1. ST eleve miyokard enfarktüsü
il. ST eleve olmayan miyokard enfarktüsü
111. Unstabil angina pektoris
iV. Stabil angina pektoris
V. Varyant angina
A.1--11 B.1--11--111
C.1 --11--111--V D. IV -- V
E.1--111--V

16· 67 yaşında bir erkek 45 dakika önce göğsünün orta


yerinden başlayan baskı tarzında, terleten, korkutan, nefesini
kesen bir ağrı ile başvuruyor. Öyküsünden ağrının sabaha karşı
başladığı, uykudan uyandırdığı ve daha önce hiç olmadığı
öğreniliyor. Özgeçmişinde hipertansiyon ve sigara içme öyküsü
vardır. Fizik muayenede kan basıncı 160/85 mmHg, nabız 115/
dakika, kalp sesleri derinden geliyor. Bakılan EKG'si şekildeki
gibi izlenen hastada kardiyak enzim düzeyi normal izleniyor.
,, .,,. <I 1 ' l J
~~Jl Jl-l__r---~Trr-1r~r-r-~y+aj, --~r~- 1 .
! . ı L j ...
p,Jı..,ıvj\..,.,...,J
· .,• · r. ,
-~+.r..Jr-JJ0-Jlr'"\_...,t_...,ıır,~ı_ fr t~~ırl". !,J·~n
1~

1
'" l 1
V' l}.r-)r-----h,

1
l

j
••~··ı 1 1, ~
J
1/

1
.ıf
, /''
ı
ı
I

j ' J
{

l
il ,
ı · r
l ./ .7
J

1
,/\.-·,}ı.A-,,f'./'----J\A-.}~v--,-..ıV'--'J\./'-. '\_1\.4---Jir'ı.,..-,,/W ,A,,/L.!L_r---JL/·.....,\..._r-'..ılr..J
1 I

1 ., !J ,1 ;/ 1

Bu hasta için en olası tanı aşağıdakilerden hangisidir?


A. ST eleve miyokard enfarktüsü
B. Non-ST eleve miyokard enfarktüsü
C. Aort diseksiyonu
D. Desedan aorta rüptürü
. E. Unstabil angina pektoris

17- Vakadaki hasta için seçeneklerde yapıl a n yoruml ard an


hangisi doğru deği dir?
A. Patogenezinde trombüs oluşumu vardır
B. Erken dönemde koroner angiografi yapılarak tıkalı damar
varsa stend konulmalıdır
C. Hastaya 2'1i antiplatelet tedavi başlanıp sık poliklinik kontrolü
önerilmelidir
D. Hastanın miyokard enfarktüsüne ilerleme riskim vardır
E. Egzersiz yada ilaçla indüklenmiş efor testi mutlak kontrendi-
kedir

18- 41 yaşınd a ki kadın


hasta sabaha karşı ani gelişe n göğüs
ağrısı şikayetiyle acil servise getirliyor. Hastaya çekilen EKG'de
ST elevasyonu görülmesi üzerine miyokard enfarktüsü düşünü ­
lerek angiografiye alınıyor ancak koroner arterlerin açık olduğu,
herhangi bir tıkanıklık olmadığı görülüyor. Bu hasta için en
olası tanı aşağıdakilerden hangisidir?
A. ST eleve miyokard enfarktüsü B. Akut koroner sendrom
C. Varyant angina D. Desedan aorta rüptürü
E. Unstabil angina pektoris
·2023 MARTTÜS SORU KAMPİ , .
a
l
~
tıtQ d .
~ aKa · emısı
" ôatılit e -. .·. - · · · ~ _· ... .
.
. . .

·. .: .. · :

19-Varyant anginada kesin tanı hangisiyle konur?


A. Ergotamin provakasyon testi
B. Koroner angiografide koronerlerin açık izlenmesi
C. EKG'de ST elevasyonu görülmesi
D. Troponin yüksek liği
E. EKG'de Tnegatifliği

20- 63 yaşındaki kadın hasta epigastrumdanbaşlayıp göğse Enfarkt Alanı EKG Değişikliği
ve sırta vuran, bulantı ve kusmanın eşlikettiği ağrı tanımlıyor.
Fizik muayenesinde herhangi birpatoloji saptanmayan hasta- İnferior D2, D3,AVF
nın elektrokardiyografisindeileti kusurunun olmadığı tespit Anterior V1-6
edi liyor; sinüs ritmi, kalpatım hızı 90/dakika ve EKG'de şekildeki
gibi V1 -6 derivasyonlarda 2-4 mm STsegment yükselmesi Anteroseptal V1 -4
saptan ıyor. '(aygın anterior D1, AVL, V1-6
Lateral D1, AVL, V5-6
' Yüksek lateral D1, AVL
Posterior V1 'de belirgin R
Sağ ventrikül V4R

Bu hasta için en olası tanı aşağıdakilerden hangisidir?


A. Akut inferior miyokard enfarktüsü
B. Akut posterior miyokard enfarktüsü
C. Akut lateral miyokard enfarktüsü
D. Akut anterior miyokard enfarktüsü
E. Akut septal miyokard enfarktüsü

21 - Vakadaki hasta için en olası tutulum hangi arterdedir?


A.Sağ koroner arter B. Sol anterior desendan arter
C. Birinci diagonal arter D. Sirkumfleks arter
E. Obtuse marginal arter

22· 12 yıldır kontrolsüz ve takipsiz hipertansiyonu olan 56


yaşında erkek hasta son 2 yıldır eforla artış gösteren göğüs
ağrıları şikayeti ile kliniğe başvuruyor. Öyküsünden son üç
gündür ağrısın ın şiddetlendiği dinl enmekle de geri l emediği
öğreniliyor. EKG'de ve ekokardiyografisi normal sınırlarda olan
hastanın laboratuar incelemesinde CK-MB 19,6 ng/ml (normal
<7 ng/ml), Troponin 1: 0,56 ng/ml (norm al <0,01 ng/ml) olarak
tesp it ediliyor. Bu hastanın tedavisinde aşağıdaki ilaçlardan
hangisinin kullanımı kontendikedir?
A. Asetil salisilik asit
B. Enoksaparin
C. Klopidogrel
D. Propranolol
E. Alteplaz

.......
~
::: · ·.•.. , ; . ... ,, :. , ·, 202yMl}RTJ'US,SO~U~~~,l
. .. . .. , , . . . ., . . .' ,., ., ~ ·. . ,, ... Dahılı e IIJ ~fiademisi
23· Acile ani göğüs ağrısı ve genel durum bozukluğuyla gelen
hastanın çekilen EKG sinde yaygın anterior miyokard enfarktüsü
1

olduğu izleniyor. Hastanın izlemde tansiyonunun 50/30 mmHg


olduğu görülüyor. Acil çekilen EK0 sunda kalp debisi 2,3 it/dk
1

(normali 5 it/dk) olarak ölçülen hastanın takılan swan-ganz ka-


teteri ile pulmoner arter kama basıncı 24 mm/hg (artmış) olarak
tespit ediliyor. Bu hastada miyokard enfartüsü komplikas-
yonlarından hangisi gelişimiştir?
A. Serbest duvar rüptürü
B. Kardiyojenik şok
C. İnterverntriküler septum rüptürü
D. Tromboemboli
E. Otoimmün perikardit

24- 3 gün önce akut inferior miyokart enfarktüsü sonras ı primer


koroner anjiyoplasti yapılan 76 yaşındaki erkek hastada, sorun-
suz bir izleme dönemi geçirilirken an iden şiddet li epigastrik
ağrı ve nefes darlığı şikayetleri başlıyor. Fizik muayenede ster-
numun solunda 3° /6° şiddetinde pansistolik üfürüm duyuluyor
ve tril alınıyor. Bu hasta için en olası tanı aşağıdakilerden
hangisidir?
A. Aort diseksiyonu
B. Papi ller kas rüptürü
C. Serbest duvar rüptürü
D. Ventriküler septum rüptürü
E. Sinüs valsalva rüptürü

25- 54 yaşında erkek hastada anterior MI ge li şiyor. Hasta teda-


visi yapılarak yoğun bakım servise alınıyor. Hastada 1 gün sonra
tansiyon ölçümleri 180/120 mmHg civarında geliyor. Doktor
hastaya kardiyoprotektif olduğu için ACE inhibitörü başlıyor.
Hastanın ertesi gün kreatini 3,8 (önceki 1,6) geliyor. Bu hastada
miyokard enfartüsü komplikasyonlarından hangisi
gelişim i ştir?
A. Serbest duvar rü ptü rü
B. Kardiyojenik şok
C. İnterverntriküler septum rüptürü
D.Tromboembo li
E. Otoimmün perikardit
26· 53 yaşında erkek hasta iki saat önce başlayan ve gittikçe
artış gösteren sırt ağrısı ile acil servise başvuruyor. Hastanın öy-
küsünde günde 1 paket sigara içtiği ve hipertansiyonu olduğu
öğreniliyor. Medikal tedavide kandesartan ve furosemid tedavisi
alan hastanın ilaçlarını düzenli kullanmadığı ve öz bakımın da
yetersiz olduğu öğreniliyor. Çekilen EKG'sinde anterito deri-
vasyonlarda 3mm kadar STsegment elevasyonu saptanıyor.
Hastanın fizik muyanesinde dispne, ortopne ve dinlemekle
akciğer bazallerinde ince ral duyuluyor. Kardiyak oskültasyonda
ise erken sistolik üfürüm ve birinci kalp sesi şiddetinde yumu-
şama tespit ediliyor. Yapılan ekokardiyografide şekildeki gibi sol
ventirkül sistolünde kanın sol atriyuma kan kaçağı izleniyor.

Bu hastada miyokard enfartüsü komplikasyonlarından


hangisi gelişim iştir?
A. Dressler sendromu B. Papiller kas rüptürü
C. Post-infark anevrizma D. Sağ ventrikül infarktüsü
E. Otoimmün perikardit

27- 78 yaşında kadın hasta şiddetli göğüs ağrısı ile acil servise
başvuruyor. Hastaya STsegment elevasyonlu miyokard infarktüs
teşhisi koyuluyor. Hastaya acil perkütan koroner girişim uygu-
lanıyor ve yoğun bakımda takibe alınıyor. Hastanın takibinin
üçüncü gününde aniden hipotansiyon, nefes dar lığı ve ortopne
gelişiyor. Fizik muayenesinde akciğerlerde bilateral alt zanl ara
ince raller duyuluyor. Çekilen akciğer grafisi şekildeki gibi
izleniyor.

Hastadaki mevcut kliniği oluşturan en olası neden hangisi-


dir?
A. Sol ön inen arter tıkanıklığı
B. Sirkumfleks arter tıkanıklığı
C. Marginal arter tıkanıklığı
D. Diagonal arter tıkanıklığı
E. Sağ koroner arter tıkanıklığı

..
28· 20 yaşındaki kadın hasta dispne şikayetiyle başvuruyor.
Yapılan fizik mauyenede birinci kalp sesi şiddetinin azalmış
olduğu, sol koltuk altına yayılan pansistolik üfürüm izleniyor
ve üfürümün şideetinin ekspiryumda arttığı farkediliyor. Ayrıca Sol atriyum

akciğerde bazallerde dinlemekle ince raller duyuluyor. Hastaya


çekilen akciğer grafisinde şekildeki gibi kardiyomegali izleniyor. - Mitral kapak

_cc---- Sol ventrikül

: 1
<l

! ~
Bu hasta için en olası tanı aşağıdakilerden hangisidir?
A. Mitral yetmezlik
B. Mitral darlık
C. Aort darlığı
D. Mitral valv prolaksusu
E. Pulmoner yetmezlik

29- Mitral darlık sebebiyle tetkik edilen bir hastada EKO'da Daralmış

mitral kapak çağı 0,9 cm 2 ölçülüyor ve sol atriyumda şekildeki mitral kapak

gibi trombüs izleniyor.

Sol ventrikül
Normal Mitral darlık
Bu hastada tedaviye yönelik bu aşamadan sonra en uygun
tedavi aşağıdakilerden hangisidir?
A. Mitral balon valvüloplasti
B. Varfarin ve düşük molekül ağırlıkl ı heparin başlanması
C. Enoksaparin sodyum başlanması
D. EKG ve ekokard iyografiyle takip
E. Mitral kapak replasmanı

30- 78 yaşında bir erkek hasta s ı k sık bayılma yakınmalarıyla


hastaneye başvuruyor. Fizik muayenede kan basıncı 100/90
mmHg, nabız 67/d akika, sternumun sağında 2. interkostal
aralıkta boyna yayılan sistolik ejeksiyon üfürümü duyuluyor. Bu
hastanın tedavisi için aşağıdak i ilaçlardan hangisi konten-
dik dir?
1. Furosem id
il. Digoksin
111. Fosfodiesteraz inhibitörleri
A.1--11
B.11--111
C.1--111
D. Sadece 111
E. 1-- 11 -- 111
la tıQd ·"
I.!!!! ~ OKO · emısı

31- 34 yaşında kadın hasta baş ağrısı, baş dönmesi ve eforla


çabuk yorulma şikayeti ile başvuruyor. Diğer fizik muayene bul-
gularından tansiyon üst ekstemitede tansiyon180/120 mmHg,
alt ekstremitede ise 90/60 mmHg ölçülüyor. Nabız 86 atım/dk,
oskültasyonda ikinci kalp sesinde sertleşme ve sırtta interskapu-
lar alanda sistolik üfürüm tespit ediliyor. Çekilen telekardiyogra-
fide kostalarda çentiklenme izleniyor.

Hasta olası tanısına yönelik en sık izlenen komplikasyon ve


infant çağda mortalitenin en sık sebebi hangisidir?
A. Klaudikatio intermitant
8. Willis poligonunda anevrizma rüptürü
C. Sol kalp yetmezliği a,
D. Subakut bakteriyel endokardit
E. Asendan aorta rüptürü

32- 19 yaşında bir hasta koşu yaparken ani ve kısa süreli Normal kalp
Hipertrofik
Kardiyomiyopati

sen kop geçiriyor. Daha önce hiçbir yakınması olmayan hastanın


öyküsünden babasını da futbol oynarken aniden öldüğü öğre­
niliyor. Fizik muayenesinde kan basıncı: 180/80 mmHg olarak
bulunuyor. Kardiyak muayenede tüm prekordiyumda apekse
yayılım gösteren sistolik üfürüm duyuluyor ve apekste palpabl
S4 hissediliyor. EKG'sinde şekildeki gibi sol Ventriküler hipert-
rofisi, lateral / anterior derivasyonlarda patolojik Odalgaları, ST
değişiklikleri ve Tdalgası inve rsiyonları izleniyor.

. LL "7 ~· · , r-r
~~-~, , r--r-l,,JU
1 i

Bakılan nabız traseside şekildeki gibi izleniyor.

Bu hastadan aşağıdakilierden hangisinin varlığı ani kardiyak


ölüm riskini artırmaz?
A. Senkop geçirmiş olması
8. Ventriküler ekstrasistol ataklarını geçirmiş olması
C. Ailede ani kardiyak ölüm öyküsü varlığı
D. Sol ventrikül duvar kalınlığının 32 mm olması
E. Egzersiz sonrası tansiyonun 190/170 mmHg olması
33- 42 yaşında kadın hasta batıcı ve keskin vasıflı göğüs
ağrısıyla başvuruyor. Öyküsünden göğüs ağrısının öne eğil­
mekle azaldığı, düze yatmak ve nefes almakla arttığı izleniyor.
Oskültasyonda hem sistolik hem de diyastolik üfrüm duyulan
hastanın ateşi 39°( ve nabzı 104 atım/dk izleniyor. Çekilen
EKG'si şekildeki gibi izleniyor.

Vakadaki hastada tanı için öncelikle yapılması gereken


tetkik aşağıdakilerden hangisidir?
A. Toraks tomografisi
B. Transözefagial ekokardiyografi
C. Kalp kateterizasyonu
D. Spiral tomografisi
E. Teleradyografi

34· 41 yaşında erkek hasta bir yıldır giderek artan, eforla mey-
dana gelen nefes darlığı ve deride renk değişikliği şikayetleriyle
başvuruyor. Fizik muayenede siyanoz, juguler venöz basınçta
artış ve S2 de sabit çiftleşme saptanıyor. Hastanın öyküsünden
8 yıl önce fizik muayene bulguları ve akciğer grafisinin normal
olduğu öğreniliyor. Hastaya yapılan EKO şekildeki gibi izleniyor.

Oskültasyonda middiyastolik üfürüm ve sistolik ejeksiyon


üfürümü duyulan bu hastada en olası tanı aşağıdakilerden
hangisidir?
A. Fallot tetralojisi
B. Trunkus arteriozus
C. Atrial septal defekt
D. Ebstein anomalisi
E. Ventriküler septal defekt
a
l
~ !!!!.I tı~
aKad· ·emısı
2023 MART TUS SORU KAMP!
► " Dahili e

35-Artvin Hopa'da yaşayan 65 yaşındaki erkek hasta yoğun


kar yağışı sebebiyle yollar kapandığı için rutin diyaliz sea nsına
gidemiyor. Takip eden günlerde bilinç değişikliği olunca acil
a ranıyor ve 112 tarafından acile götürülüyor. Çekilen EKG'si
şekildeki gibi izleniyor.

~ı~~JJ~N
,~~L,~JrJY~
~~ıı-f--~l
Bu hastadaki EKG bulgusu hangi elektrolit im balansı sonucu
gelişmiştir?
A. Hiponatremi B. Hiperkalsemi
C. Hipermagnezemi D. Hipokalemi
E. Hiperkalemi

36- 41 yaşındaki kadın hasta çarpıntı şikayeti ile başvuruyor. Öy-


küsünde sen kop veya presenkop tanımlamıyor. Hemodinamisi
stabil izlenen hastaya çekil en EKG aşağıdaki gibi geliyor.

EKG'si supraventriküler taşikardiyle uyumlu izleniyor ve adeno-


zin başlanıyor. Ancak tedaviye yanıt alınamıyor ve kalp atım hızı
170 atım /dk olarak devam ediyor. Bu hasta aşağıdakilerden
hangisi adenozin tedavisine yanıtsızlığı açıklar?
A. Dipiridamol kullanımı
B. H asta nın teofilin kullanıyor olması
D. Hastanın kardiyak nakilli olması
C. Kadın hasta olması
E. Kalp hı z ının 150 at ım/dk'nin üzerinde olması

37- Çarp ıntı yakınmasıyl a polikliniğe başvuran 72 yaşındaki


kadın h asta nın fizik muayenesinde nabız 121 /d akika, kalp tepe
at ım sayısı 164/dakika olarak sapta nıyor ; ek ses ve üfürüm
duyulmuyor. EKG'si şekildeki gibi izlenen h astan ın öyküsünde
hipertansiyon ve bir kez geçici iskem ik atak geç irdi ğ i öğrenili ­
yor.
' '__"'" -ır-ı.-------. . . .r:~~r-71--·~r-J/\_....,r-...J
nı,____t_J_-JL--~.il ı ı ı" r ı ,
ı ı !
ı ·ı
./1....
1

rı,;~L_,,_ '-~ ;:;-.L.-'-1,_::--rrı:LLJ,ı.


n-~-r---,'-~-_J, __~__...Jı_, '\.. . lr~('\,._~/\_J_~JJı.. .......,L .
'" •"· ···7 ı ı "
Vakada ki hastanın tedavisi için aşağıdaki ilaçlardan hang isi en
uygu dur?
A. Metoprolol B. Kinidin

..
C. Atopin D. İ zoprote reno l
E. Amlodi pin
38- 76 yaşında erkek hasta acil servise son 3 gündür giderek
artan kusma şikayeti ile başvuruyor. Özgeçmişinde 1 yıl önce
akut miyokard enfarktüsü geçirdiği öğreniliyor. Hastanın yapı­
lan fiziksel muayenesinde presenkop halinde olduğu, tansiyon
arteryeli: 80/60 mmHg, nabız: 44/dk izleniyor. Hastaya çekilen
EKG ise aşağıdaki gibi izleniyor.

·~rı _.,vı...,_;.,.••
-~-~~n7·,_,r~n~;~-~-
~
V
1
! ' ı l J

ll
_ _...ı---,--,.....ı...·,_
..,__J,/_--"-.
,·vı -r. l
ı...,,~.-•/•--~t..,~~J.,
-V'"'-- -ıt ..,..,

.L..ı-,__.,ı..,.__J.....,,_j_,."'-J.A. _1-- _t.,.__J_,,__c.~J.,.._.J.,A___ L


:ili fol '?a ':111

f =·-,,----~·-t~-~~-,~!
1
--t-- 1r✓--- l ~--r~ t~t-~-jf
Myokard enfarktüsü sebebiyle asetilsalisilik asit, bisoprolol, kap-
topril kullandığı ve yakın zamanda da kendisine digoksin reçete
edildiği öğreniliyor. Bu hasta için en doğru yorum hangisidir?
A. Digoksine bağlı 1.derece AV blok gelişmiştir
B. Digoksine bağlı mobitz tip 1 gelişmiştir - uj
C. Klinik ve EKG Adams-Stokes sendromuyla ilişkilidir
D. Atriyal gibrilasyon gelişmiş olup acil kardiyoversiyon yapıl ­
malı
E. AV tam blok olup kalp pili takılmalıdır

39- 56 yaşındaki erkek hastada, yarım saat önce başlayan göğüs


ağrısı nedeniyle ambulansla getirilirken acil servise ulaşmadan
3 dakika önce kardiyak arrest gelişiyor ve kardiyak resusitasyon
yapılmaya başlanıyor. Acil serviste hemen değerlendirilen
hastanın karotis nabzı alınmıyor ve spontan solunum aktivitesi
bulunmuyor. Bu hastada monitörde aşağıdaki kardiyak ritim-
lerden hangisinin görülmesi beklenmez?

A.

B.

C.

D.

E.
l g tı~d ..
I.!!!!! ~ aKa · ·emısı
40- Daha önce madde kötüye kullanım öyküsü nedeniyle me-
tadan tedavisi alan 32 yaşındaki kadın hasta yakın zamanda bir
üst solunum yolu enfeksiyonu geçirmesi üzerine arkadaşı ona
evden eritromisin etken maddeli ilaç veriyor. İlacı kullanımın­
dan 6 saat sorna acil servise ciddi çarpıntı şikayetiyle getiriliyor.
EKG'si şekideki gibi izleniyor.

Hastaya bu EKG üzerine 2 mg iV magnezyum sülfat veriliyor


ancak klinik iyileşme izlenmiyor. Laboratuvarı olağan izlenen
hasta için bu aşamadan sonra en uygun adım aşağıdakiler­
den hangis"dir?
A.Metoprolol 5 mg intravenöz uygulama
B. Amiodaron 150 mg iV
C. İmplante edilebilir defbrilatör için acil sevk
D. iV izoproterenol infüzyonu
E. Sedasyon ve defibrilasyon
GÖG-ÜS HASTALIKLARI
1- Penetran göğüs yara lanması nedneitle acile getirilen 45
yaşındaki erkak hastanın yapılan fizik muayenesinde vibrasyon
torasikde ve matitede azalma izleniyor.Çekilen akciğer grafisin-
de trakeanin lezyonun karşı tarafına deviye olduğu görülüyor.
Bu hasta için en olası tanı aşağıdakilerden hangis"dir?
A. Bronkopulmoner pnömoni
B. Kardiyak tamponad
C.Amfizem
D. Atelektazi
E. Pnömotoraks

2· Yüksek ateş ve üşüme-titeme şikayeriyle gelen 34 y aş ındaki


erkek hastanın solunum muayenesinde matitede ve vibrasyon
torasikte artış izleniyor. Ayrı ca hekim trakea üze rind e duyulan
sesin akciğerlerde-duyulan sesle benzer olduğunu farkediyor.
Bu hasta için en olası tanı aşağıdakilerden hangis"dir?
A. Pnömoni
B. Plevra! efüzyon
C. Astım
D. Atel ektazi
E. Amfizem

3. 10

7,5

,: i
2,5

İn spirasyon
7,5

Haclm (LJ

Şe kild ek i akım-volüm eğrisi hangisiyle uyumludur?


A. Restriktif a kc i ğer h asta lı ğ ı
B. Ekstrato rasik obstrüksiyon
C. İn tratoras ik obstrüksiyon
D. Obstrüktif akciğer hastalığı
E. Sabit hava yolu obstrüksiyonu

Hemoptizi
4- 51 ya ş ındaki kadın hasta öksürmekle ağ ı zdan yakl aş ık 800 cc
kan ge lmesi nedeniyle acil servise getiriliyor. Bu hasta için ilk
Orofa rinksveGİS Öykü,Anamnez,
yapılması gereken aşağ ıd a kil e rden hang isidir? kayn aklıolmadığından - fizikmuayene,
eminol göğüsgraflsi
A. Bron ş iya l arter embolizasyonu
B. E ndobronşiya l arg on tedavisi
C. Antitusif ilaç tedavisi Masifhemoptizi

D. Çift lümen li tüp ile entübasyon Nan-masif hemoptizi


• • Embolizasyon
E. Havayolu aç ıklğınn sağ l a nm as ı Havayolu açıklıgın ı sagla -+ Devam eden kanama -+ k.

..
v e yareze s ıyon

Göğüsgrafisi
ıDı ~~ademisı
5- 10 t;

MIF
%50 FVC
inspirasyon
7,5

3 2 1 O
Hacim/L)

Şekildeki akım-volüm eğrisi hangisiyle uyumludur?


A. Restriktif akciğer hastalığı
B. Ekstratorasik obstrüksiyon
C. İntratorasik obstrüksiyon
D. Obstrüktif akciğer hastalığı
E. Sabit hava yolu obstrüksiyonu

6-
75
10 1 •

c 2~5
~
"'-
E O - -
i
2,5

ln splra syon
7,5

4 3 2 1 O
Hacim (L}

Şeki ld eki akım volüm eğrsine sahip bir hastada bu aşamadan


sonra ilk yapılması gereken aşağıdaki l erden hangisidir?
A. Servikotorasik tomografi
B. DLCO (diffüz lung karbonmonoksit testi)
C. İnhaler bronkodilatatör uygulama
D. Kardiyopulmoner egzersiz test
E. Ekokardiyografi

7- Hipoksemi hakkında yapılan yorumlardan hangisi yanlış


bir yorumdur?
A. Hepatopulmoner sendromda hipoksinin temel sebebi şanttır
B. İnterstisyel akci ğer hastalığında kan gaz değişimi bozu lur
C. ARDS de hipoksi nötrofil infiltrasyonuna bağlıdır
1

D. Karbonmonoksit intoksunda Fi0 2 artar


E. Pulmoner embo lide ventilasyon/ perfüzyon uyumsuzluğu
olur

8- Alveolar-arterya l oksijen farkı (PA0 2 - PaOJ aşağıdaki hasta-


lıkların hangisinde artmaz?
A. Pulmoner tromboemboli
B. İ diyopatik pu lmon er fibrozis
C. Alveolar hipoventilasyon sendrom
D. Hepatopulmoner sendrom
E. Pnömoni
B.

C.

D.

E.
IIJ gflademisi
10· Düşük doz inhaler kortikosteroid kullanan 34 yaşındaki 11· İki yıldır astım tanısıyla düzen li budesonid inhaler 400
astım hastası Hatice teyze son bir aydır neredeyse hermen her mcg/gün ku llanmakta olan erkek hasta, son 15 gündür gittikçe
gün öksürük ve nefes darlığı ve ayda 3 kez gece semptomu artan nefes darlığı şikayetiyle başvuruyor. Son bir ayda hemen
olduğunu ifade ediyor. Hastaya yapılan solunum fonksiyon her gece nefes darlığıy l a uyandığını, gündüzleri her gün k ı sa
testinde FEV 1 %67 izleniyor. Bu hasta için en doğru yorum etkili beta 2 agonist kullanımına ihtiyaç duyduğunu ve günlük
hangisidir? aktivitelerinin de nefes darl ı ğı nedeniyle kısıtlandığını söylüyor.
A. Hasta orta persistan astıma progrese olmuştur Hastan ı n daha önce acil servise başvurmadığı öğreniliyor. Bu
B. Ağır persistan astım gelişmiştir hasta için en doğru yorum aşağıdakilerden hangisidir?
C. Vaka intermittan astımla uyumludur A. Hasta orta persistan astıma progrese olmuştur
D. Hasta hafif persistan astımın ılımlı halidir B. Ağır persistan astım gelişmiştir
E. Kontrol altında astım olarak sınıflandırılır C. Vaka intermittan astımla uyumludur
D. Hasta hafif persistan astımın ılımlı halidir
E. Kontrol altında astım olarak sınıflandırılır

Astımın Şiddetine Göre Sınıflandırılması ve Tedavisi


Para metreler İntermittan Hafif persistan Orta persistan Ağır persistan

Semptom < 2 gün/hafta > 2 gün/hafta Hergün Gün boyu


Gece semptomu < 2 defa/ay 3-4 defa/ay > 1 defa/hafta Sık sık

Kısa etkili beta-2 agonist kullanımı < 2 gün/ hafta > 2 gün/ hafta Hergün Günde birkaç kez
Günlük aktivite Normal Hafif kısıtlı Orta kısıtlı Ciddi kısıtlı
Akciğer Fonksiyonu FEV1 > %80 FEV 1 > %80 FEV 1 %60-80 FEV 1 < %60

12- Bu hasta için en uygun tedavi aşağıdakilerden hangi- 13- Uzun süredir astım tanıs ıyla izlenen ve ilaçlarını düzen-
s"dir? siz kullanan 34 yaşındaki erkek hasta acile akut astım atağı
A. Kısa etkili beta 2 agonist şikayetiyle başvuruyor. Bakı l an kan gazı hipoksik ve hiperkabik
B. Düşük doz inhaler steroid + Uzun etkili beta 2 agonist izleniyor. Bu hastanın tedavisinde ilk tercih ilaç aşağ ıd aki­
C. Yüksek doz inh. steroid + Uzun etkili beta 2 agonist + Oral lerden hangisidir?
steroid A. Albuterol B. İpratropium
D. Düşük doz inhaler steroide devam C. Lebrikizumab D. Oral teofilin
E. Orta doz inhaler steroid + Teofilin E. Montelukast

Step 5
Step 3 LAMA eklenrresi
Orta doz ida rre ± anti lg E
Düşük doz ida rre
<ÖJrrı Ci:@IJii~lfü
İKS - Forrroterol
Gereğind e düşük doz İKS - Forrroterol İKS - Forrroterol ± a nti IL-5
lfe€1@ı'ıhii
RAHATLATICI: Gerektiğinde düşük doz ICS-forrroterol

St~p 5
Step4 LAMA eklenrresi
St€p 3
Orta/yüksek doz
~teJP 11 Step 2 Düşük doz ida rre ±anti lgE/ ILS
6-\\litıeımıait~f q SABA alındığında iKS Düşük doz ida rre İKS İKS - LABA
ida rre İKS - LABA Yüksek doz İKS+ LABA
lrecıfaıı'hn
RAHATLATICI: Gerektiğinde kısa etkili ~2-agonist

LABA: Uzun etkili ~2-a gonist SABA: Kısa etkili ~2-agonist iKS: İnha ler kortikosteroid

CD
14a 67 yaşında kadın hasta polikliniğe ilerleyici nefes darlığı
ve öksürük şikayetiyle başvuruyor. Yapılan fiizk muayenede ins-
piryum sonunda ral duyulan hasta hastaya çekilen tomografide
şekildeki gibi retiküler opasiteler ve hava kistler izleniyor.

Solunum fonskiyon testinde restriktif patern saptanan bu hasta-


da tanıya yönelik en iyi non-invaziv tetkik aşağıdakilerden
hangisidir?
A. Telekardiyografi
B. Kardiyopulmoner egzersiz testi
C. Ventilasyon perfüzyon sintigrafisi
D. Yüksek rezolüsyonlu bilgisayarlı tomografi
E. Solunum fonksiyon testi

15- 34 yaşındaki kadın hasta; yaklaşık 5 aydır nefes darlığı,


burunda, yanaklarda ve ciltte morumsu renk değişikliği, her
iki şekildeki gibi gözde kızarıklık ve eklem ağrısı şikayetleriyle
başvuruyor.

Fizik muayenede başka patoloji tespit ed ilmeyen h astan ın la-


boratuvar testleri serum kalsiyum düzeyinin 12,5 mg/dl olma s ı
dışında normal bulunuyor. Çekilen akc i ğer grafisinde bilateral
hiler lenfadenopati ve akciğer parankim tutu lumu izleniyor. Sar-
koidoz tanısı konul an hasta hastalığın hangi evresindedir?
A. Evre O B. Evre 1
C. Evre 2 D. Evre 3
E. Evre 4

16- Efor dispnesi tanımlayan 45 yaşındaki erkek h astan ın öykü-


sü nden 40 paket/yıl sigara içti ği öğreniliyor. Fizik muayenede
her iki akciğerde baza il erde inspiryum sonu raller duyuluyor ve
kalp sesleri olağan dinleniyor; ek ses ya da üfürüm duyulmu-
yor. Hastaya yapıl an solunum fonksiyon testinde FEV1 ve FVC
aza lmı ş ; FEV1 /FVC oranı ise artm ı ş izleniyor. Bu hasta için en
olası tanı hangis"dir?
A. Deskuamatif pnömoni
B. Pulmoner lenfangiomiyomatozis
C. Pulmoner alveoler proteinozis
D. Hipersensitivite pnömonisi
E. Kronik obstrükstif a kc i ğer hastalığı
tllı ~~ademisi
17- Bir yıl öncesine kadar 20 yıldır kot taşlama işinde çalışan 45
yaşındaki erkek hasta ilerleyici nefes darlığı şikayetiyle geliyor.
Fizik muayenede normotansif ve kalp atım sayısı olağan olan
hastaya çekilen bilgisayarlı tomografide şekildeki gibi yaygın
nodüler opasitler izleniyor.

Nodüllerden yapılan biyopside egg-shell kalsifikasyon izleniyor.


Bu vakada hangisinin görülme riski artmıştır?
A.Astım
B. Pulmoner alveoler proteinöz
C. Panasiner amfizem
D. Lenfangiomiyomatozis
E. Yuvarlak atelektazi

18- HLA DPB1 pozitifliği'nin izlendiği meslek hastalığı ~


aşağıdakilerden hangisidir?
A. Berilyozis B. Silikozis
C. Asbestozis D. Bissinozis
E. Kömür işçisi pnömokonyozu

19- 24 ya ş ında multiple kemik kırıkları olan hasta yoğun bakım Hasarlanmış alveol
operasyon sonrası izlem amaçlı takip ediliyor. Takibin 2.günün-
de hastanın oksijen ihtiyac ında artış izleniyore ve hem nazal
kanül hem de maske yoluyla %100 oksijen verilmesine rağmen
s-parmak ucunda ve eş zamanl ı kan gazından bakılan saturas-
yonu yükselmiyor. Hastaya çekilen akc i ğer grafisinde diffüz
infiltrasyon izlenen hastada ARDS düşünülüyor.

Bu hasta hakkında hangi yorum doğru deği dir?


A. Hyalen membranlar görülebilir ve eksudatif evrenin göster-
gesidir
B. Hastada patogenetik süreci başlatan yaygın alveolokapi ller
hasardır
C. Tedavinin temel basamağı altta yatan hastalığın tedavisidir
D. Aşırı sitokin deşarjını baskılamak için düşük doz glukokortiko-
id başlanabilir
E. Mekanik ventilatör ihtiyac ı gelişirse düşük tidal volümde
yüksek frekanslı ventil asyon yapılmalıdır

CD
·~-~mii::v:;.n:a.ı• -~r.ıu, 1~;,;11111111

L:2..E22~~52S22--------:l2_2_--------2J~~~-R~~Gmffl- D ~flademisi
20-Aşağıdaki hastalıkların hangisinde permisif hiperkapni
görülmesi beklenir?
A. Pnömotoraks
B. Pulmoner tromboemboli
C. Tansiyon pnömotoraks
D. Sağlık bakımı ilişkili pnömoni
E. Akut respiratuvar distress sendromu tedavisi

PE şüphesi

21- Dört saat önce ani başlayan nefes darlığı nedeniyle aci l
servise geltirilen 45 yaşındaki erkek hastanın fizik muayenesin-
Düşük risk Yüksek risk
de kalp atım hızı 130 atım/dk, tansiyonu 70/60 mmHg izleniyor.
Akciğer grafisi normal izlenen hastanın çekilen toraks ang iogra-
fisi çekildeki gibi izleniyor. D-dimer

Normal Yüksek
t:
PTE dışlanır Toraks BT angiografi

Hastada bakıla NT-proBNP 1800 pg/ml izleniyor ve ekokar-


DVT şüphesi
diyografide geniş bir sağ atriyum ve ventrikül, orta derecede
triküspit yetersizliği ve hiperkontraktil sağ ventrikül apeksiyle
akinetik orta-sağ ventrikül serbest duvarı gösterdi. Ayrı'ca bakı­ Düşük risk Yüksek risk
lan pulmoner arter basıncı 70 mmHg izleniyor. Bu hastada en
olası tanı hangisidir?
A. Astım bronşiole D-dimer
B. Pulmoner hipertansiyon
C. Pulmoner tromboemboli
Normal Yüksek
D. Obezite hipoventilasyon sendromu
E. Tansiyon pnömotoraks
DVT dışlanır Doppler USG

22· Vakadaki hastada bu aşamadan sonra tedaviye yönelik


ilk seçilmesi gereken aşağıdak il erden hangisidir?
A. Trombolitik tedavi
B. Antikoagülan tedavi
C. Vena cava inferior fil tresi
D. Embolektomi
E. Asetil sa lisilik asit

23- Eğer bu hastada trombolitik tedavi açısından konten-


dike bir durum varsa hangi tedavi tercih edilmelidir?
A. Vena cava inferior filtresi
B. Antikoagülan tedavi
C. Trombolitik tedavi
D. Embolektomi
E. Asetil sa lisi lik asit
la!!!!!I OKad'
l.!!!! tı~ emısı
. "
24· 51 yaşındaki kadın hasta, son aylarda şiddetlenen ne-
fes darlığı ve ayak bileklerinde şişlik şikayetiyle başvuruyor.
Öyküsünden daha önce iki defa egzersiz esnasında senkop
geçirdiği öğreniliyor. Fizik muayenesinde boyun ven dolgunlu-
ğu, hepatomegali ve periferik ödem saptanması üzerine y ap ılan
ekokardiyografide sağ kalp boşluklarında genişleme ve hafif
perikardiyal efüzyon görülüyor. Pulmoner arter sistolik basıncı
70 mmHg olarak ölçülüyor. En olası tanı hangisidir?
A. Astım bronşiole
B. Pulmoner hipertansiyon
C. Pulmoner tromboemboli
D. Obezite hipoventilasyon sendromu
E. Tansiyon pnömotoraks

25· Bu hastada tanıyı kesinleştirmek için hangisi yapılma­


lıdır?
A. Spiral toraks bilgisayarlı tomografi
B. Eforlu elektrokardiyogram
C. Venöz Doppler ultrasonografisi
D. Sağ kalp kataterizasyonu
E. Kardiyopulmoner egzersiz testi

26- 65 yaşındaki kadın hasta titremeyle yükselen ateş, öksü-


rük ve göğsünün sol tarafında derin nefes almakla artan ağrı
yakınmalarıyla başvuruyor. Beş gün önce başlayan nezle, boğaz
ağrısı ve öksürük ile bir gün önce aniden üşüme ve titremey-
le yükselen ve terlemeyle düşen ateş tanımlıyor. Çıkardığı
balgamın miktarının az, koyu ve pürü lan bir görünümde olduğu
saptan ıyor. Toraks muayenesinde sol hemitoraks alt zanda
matite al ınıyor ve inspiryum sonu ince raller duyuluyor. Çekilen
akciğer grafisi şekildeki gibi izleniyor.

Klinik olarak toplumdan edinilmiş pnömoni tanısı ko-


nulmuş hastanın balgam kültüründe üremesi en olası
mikroorganizma aşağıdakilerden
hangisidir?
A. Streptococcus pneumoniae
B. Hemofilus influenza
C. Staphylococcus aureus
D. Mycoplasma pneumoniae
E. Legionella pneumophila
27- İnfluenza salgınına bağlı pnömoni gelişmesi üzerine nede-
niyle hastaneye yatırılan 81 yaşındaki bir kadın hasta taburcu
edildikten üç gün sonra ateş, üşüme, titreme ve yan ağrısı ya-
kınmaları ile yeniden başvuruyor. Hastanın yapılan laboratuvar
tetkiklerinde lökosit 21.000/mm 3, eritrosit sedimantasyon hızı
98 mm/h, CRP 156 mg/dl izleniyor. Yan ağrısına yönelik yapılan
ultrasonografide sağ böbrek çevresinde şekildeki gibi apse ile
uyumlu görünüm izleniyor.

Kan kültüründe ise beta hemolitik, katalaz ve koagülaz pozitif,


gram pozitif koklar izole ediliyor. Bu hastada en olası etken
aşağıdakilerden hangisidir?
A. Streptococcus pneumoniae
B. Hemofilus influenza
C. Staphylococcus aureus
D. Legionella pneumophila
E. Pseudomonas aeruginosa

28- Son 1 haftadır devam eden yüksek ateş ve şiddetli sağ


yan ağrısıyla getirilen 76 yaşında bakım evinde kalan bilinen
demans ve geçirilmiş serebrovasküler olay öyküsü olan erkek
hastada yapılan fizik muayenede pis kokulu balgam, di şet lerine
basıldığında pürü lan akıntı ve fizik muayenede sağ akc iğerde
yaygın raller dıu yuluyor. Çekilen akciğer grafisi şeki l deki gibi
sağ akciğer orta zanda infiltrasyon izleniyor.

Bu hasta için en olası tanı aşağıdakilerden _hangisidir?


A. İnterstisyel pnömoni
B. Ati pik pnömoni
C. Bronkopnömoni
D. Aspirasyon pnömonisi
E. Lober pnömoni
g ~~ademisi
29- Trafik kazası nedeniyle üç haftadan beridir yoğun bakım ser-
visinde düşük tidal volümde, yüksek frekansta mekanik ventila-
tör uygulanan 56 yaşındaki hastada son 24 saattir devam eden
ateş yüksekliği nedeniyle çekilen akciğer grafisinde şekideki
gibi sağ üst lobda konsolidasyon ve kaviteleşme gözleniyor.

Öyküsünden sosyal hayatında yoğun alkol içicici olduğu


öğreniliyor. Hastaya yapılan bronkoalveolar lavajın mikroskopik
incelemesinde şekildeki gibi gram negatif, laktoz pozitif, hare- U)
ketli basiller saptanıyor. A

Bu hastada en olası etken aşağıdakilerden hangisidir?


A. Escherichia coli
B. Serratia marcescens
C. Proteus mirabilis
D. Acinetobacter baumannii
E. Klebsiella pneumoniae

30· 56 yaşında biline SLEta nısı olan ve bu sebeple de hidrok-


siklorokin ve steroid tedavisi alan kadın hasta iki haftadır devam
eden ateş, öksürük, kanlı balgam çıkarm a , baş ağrı s ı ve sağ
kolunda güçsüzlük yakınmaları ile başvuruyor. Çekilen toraks
tomografisinde sağda daha fa zla olmak üzere her iki akciğerd e
apse ile uyumlu lezyon beyin tomografisinde ise beyin apsesi
ile uyumlu, tek, halka şeklinde lezyon görülüyor.

Bronkoalveoler lavaj örneğinde gram pozitif, dallanmış


filamantöz bakteriler görülüyor, bu bakterilerin de ARB pozitif
olduğu saptanıyor. Bu hastada en olası etken aşağıdakilerden
hangisidir?
A. Nocardia asteroides B. Streptococcus pneumoniae
C. Moraxella catarrhalis D. Klebsiella pneumoniae
E. Actinomyces israelli
31· 21 yıldır tip 1 diabetes mellitus tanısıyla izlenmekte olan ve
antidiyabetik ilaçalarını düzgün kullanmayan 34 yaşında kadın
hasta ateş, baş ağrısı, burun tıkanıklığı yakınmaları ile başvuru­
yor. Muayenesinde postnazal sarı-turuncu renkli pürülan akıntı
saptanan hastanın paranazal sinüs grafisinde sağ maksillar
sinüste havalanma azlığı ve hava-sıvı seviyesi varlığı görülüyor.

Sağ maksillar sin üsten ponksiyon la alınan örneğin incelenme-


sinde şekildeki gibi 45° açı ile dallanan septalı hifler içeren küf
mantarı varlığı saptanıyor.

,._·
Bu hastadaki en olası etken aşağıdakilerden hangis"dir?
A. Aspergillus flavus B. Rhizopus sp. ,
C. Candida albicans D. Histoplasma capsulatum
E. Aspergillus fumigatus

32· Nefes darlığı ve sağ yan ağrısı yakınm a ları ile başvuran
hastanın akciğer grafisinde şekildeki
gibi sağ h emitoraksın
nerdeyse tamamını kaplayan masif plevral efüzyon saptanıyor.

Tüp torakostomi ile yarım saat içinde toplam 5 litre sıvı drenajı
sağlanan hastada işlem sonrasında ilerleyici solunum yetmezli-
ği gelişiyor ve hasta 4 saat sonra ölüyor. Bu hastadaki en olası
ölüm nedeni aşağıdakilerden hangis"dir?
A. Basınçlı pnömotoraks
B. Reekspansiyon akciğer ödemi
C. Masif bronkopulmoner obstrüksiyon
D. Masif pulmoner emboli
E. Bronkoplevral fistül
g ~~ademisi
33- 24 yaşında erkek hastanın antibiyotik tedavisine rağmen
devam eden plöretik ağrı nedeniyle değerlendiriliyor. Öyküsün-
den son 15 gündür ateşler ve gece terlemeleri olduğu ve son 3
ayda 12 kilo kaybettiği öğreniliyor. Çekilen grafide bilateral hiler
lenfadenopatili ve fibrotik gölgelerle birlikte sağ üst ve orta zon
akciğerinde düzensiz opasifikasyonu gösteren alanlar izleniyor.

Bu hastada tanıya yönelik ilk yapılması gereken hangis"dir?


A. Kompleman fiksasyon testi
B. İdrarda antijen testi
C. Kan kültürü
D. Balgam yaymasında asidmezistan bakteri
E. Balgamda tüberküloz basilinin üretilmesi

34- 44 yaşındaki erkek hasta; öksürük, balgam ve gece terleme-


si yakınmalarıyla başvuruyor. Posteroanterior akciğer grafisinde
sağ üst zonda kaviter infiltrasyon görülüyor.

Balgam ARB pozitif olan hastaya "yeni tanı akciğer tüberkülozu"


teşhisiyle standart izoniazid, rifampisin, pirazinamid, etam-
butol tedavisi başlanıyor. Bir ay sonraki kontrolünde bulantı,
kusma ve idrar renginde koyulaşma yakınması olan hastanın
serum transaminaz düzeylerinin normalin üst sınırının 5 katını
aştığı ve total bilirübin düzeyinin 1,6 mg/dl olduğu saptanıyor.
Bu hastada tedavi sürecindeki bir sonraki adım için en
uygun yaklaşım hangisidir?
A. Tüm ilaçların kesilip transaminaz değerlerinin izlenmesi
B. Tüm değerler kesilmeli ve 1 hafta sonra yeniden başlanmalı­
dır
C. Serum bilirübin değeri 5 mg/dl'yi geçince ilaç kesilmeli
O. Asemptomatikse izlem yeterlidir
E. Tüm ilaçların dozlarını yarıya düşürerek tedaviye devam
edilmesi
ROMATOLO:Jİ
1- Halsizlik, iştahsızlık, kilo kaybı ve ateş şikayetleriyle doktora
başvuran ~4 yaşındaki kadın hastada bakılan ANA granüer Homojen Granüler Periferik Sitoplazmik Nükleoler Sentromer

1/320 (+)izleniyor. Bu hasta için aşağıdaki hastalıklardan


Anti-Ro/La
hangisi ayırıcı tanıya girmez? Anti-histon
Anti-Smith
Anti-Jo Pm-scl
dsDNA AMA Anti-Scl70 Sentromer
Anti-Scl70
A. Sistemik lupus eritematosus UIRNP
Riboıoma l P RNAP

B. Sjögren sendromu
C. Skleroderma
D. Miks konnektif bağ doku hastalığı
E. Primer biliyer kolanjit 1
2- 67 yaşında kadın hasta 5 aydır her iki elinin proksimal
interfalangeal, metakarpofalangeal eklemlerinde ağrı, kızarıklık
ve şişlik şikayetiyle başvuruyor. Öyküsünden ağrısının sabahları ~ş-;",lt.."ffJ

uyandıktan sonra bir satten fazla sürdüğü ve egzersizle ağrı­ So•.ıe

sının azaldığını ifade ediyor. Laboratuvarında sedimentasyon Cııı:11 '-r-t-=•- - - - - ~ ~-,, ·-._~,.:=--y::--~-- M~~ı:,,
Jı:ır J

78 mm/h, CRP 45,7 mg/dL ve romatoid faktör düzeyi 21OU '1"""

u=
.~.ıE:11.

bulunuyor. Bu hastada en olası tanıda yer alan hastalığın


t""iını !c

ır.1m1tx, ,'!'
eklemin hangi komponentini tutması beklenir? ~ .
A. Eklem kıkırdağı B. Sinovyum
C. Vasküler yapılar D. Kemik doku
E. Kıkırdak

3-Aşağ ıdaki doku grupların dan hangisinin var lığınd a roma-


toid artrit riski belirgin şeki lde artar?
A. HLA DR 1 B. HLA B5 (51)
C. HLA B27 D. HLA DR 2
E. HLA DR 3

4- 56 yaşında romatoid artritli kadın hasta günde 5 mg predni- fffmatolojilı:Tutulttm: fos ık bulgu normokrom-normositer anemi Kronik ha&tafık arLemisi
zolonla birlikte anti-TNF-a tedavisi almakta ve hastalık aktivitesi ~neminf11 derinliği fı~5.falığm ıiddeti ı.ea~ut faıı1anıtı1faı koref edir
Akııt faı '(c;m!L olar'"ktromfıosit j
yüksek seyretmektedir. Bu tedaviye rağmen el, el bileği, ayak, 1 Roınat@id artrit -ı- Sple11omegalt-E- N
.ötropeni ~tystndrom
ayak bileği eklem lerinde şiddet li ağrıları vardır. Ellerinde kuğu • İleri evre. sempoefüf ağrı naıstalrğı afarı!ardaı giirü!iir
boynu deformitesi, metatars başlarında subluksasyon, ön kol • Aıef, he~*tomegaı!i,, lenıfadenı:ıpali, frombıısrtoııenıi
• Diffuı biij5k Bh,ücreli lerıfomaı rrski t
ekstansör yüzlerinde subcutan nodüller izleniyor. Fizik muaye-
• Büyuk gı.-ınüler lenfositik lösemi:
nede ateşi 39.1 °C, nabız 112/dk, tansiyon 110/85 mmHg ve
Fe~r send'rom11rraı lıenier
batın muayenesinde splenomegal i izleniyor. Hemogramında: Thücreli fösemrcl'ir
Lökosit: 2300/mm 3 Nötrofil: %15 R'orırto·d ar!citı.emirnir.desıiflğ ı t
Lenfosit: %75 Monosit: %8 R'A' ıru erk ~ evreferintle goriilelıilir
N'öt m~eıı i Feffy'send'mrırmna· g üre:d'aJaı derindir
Eozinofil : %2 Hemoglobin: 9,2 mg/dL
Hematokrit: %28,5 Trombosit: 130.000/mm 3 R'ffe.'.'d; lökopeni varlrğıııdaı ,Jfa·fösemi ııfenfoın~vıe ilaç toksisitesi gelmelidir
Periferik yaymada an izositoz ve poikilositozun eş lik etmediği
normokrom anemi izleniyor. Bu sonuçlarla birlikte hastaya
kemik iliği asipirasyon biyopsisi yapılıyor ve kemik iliği hiper-
sellüler izleniyor. Bu hastanın en olası tanısı aşağ ıd aki l erden
hangisidir?
A. Akut miyeloblastik lösemi
B. Diffüz büyük Bhücreli lenfoma
C. Glukokortikoid toksistesi
D. Felty sendromu
E. Anti-TNF-atedavisine bağl ı idiyosenkrazik reaksiyon

CD
·2023 MARTTUS SORU KAMPI
la
I.!! !!!!I tı~d ► «
aı<a · 'emısı Dahili e
5- Romatoid ilaç kullanımında hangi ilacın kullanımı subkutan
nodül gelişme riskini atrırır?
A. Koryoretinit
B. Leflunomid
C. Anakinra
D. Metotreksat
E. Abetacept

6- Romatoid artrit tanılı 75 yaşında kadın hasta ağız içinde Metotreksat toksisitesi için risk faktörleri:
iltihaplı yaralar, yemek yerken yaralar nedeniyle zorlanma, İleri yaş
iştahsızlık ve idrar yaparken idrarının kırmızı geldiğini görmesi Böbrek yetmezliği
üzerine başvuruyor. Bakılan hemogramında lökosit: 2000/mm 3, Dehidratasyon
nötrofil 100/mm 3, hemoglobin: 6,6 g/dL, trombosit: 75.000/ Düşük serum folat düzeyi
mm 3 izleniyor. Öyküsünden sadece Perşembe günü sabah ve Hipoalbuminemi
akşam metotreksat kullanması önerilen hastanın metotreksatı Beraberinde NSAİİ kullanımı
hergün aldığı öğreniliyor. Hastanın fizik muayenesinde; ağız
içinde iltihaplı yaralar dışında ek bulgu izlenmiyor. Metotreksata ,.
bağlı pansitopeni düşünülen hastanın diğer laboratuvar ince-
lemesi normal geliyor ve USG'de hepatomegali, splenomegali
ve lenfadenopati izlenmiyor. Bu hastada hangisi metotreksat
intoksikasyon riski artıran durumlardan biri değildir?
A. Hastanın ileri yaşta olması
B. Eşlik eden böbrek yetm ez liği
C. Serum folat düzeyinin düşük olması
D. Glukoz 6 fosfat dehidrogenaz eksikliği
E. Non-steroid anti-inflamatuvar ilaç kullanımı

7- 24 yaşında kadın hasta yaygın eklem ve kas ağrısı şikayetiyle


geliyor. Öyküsünden güneşe maruz kalan yerlerde kabarıklık,
kızarıklık, kaşıntı olduğunu ayrıca 2 defa gebelik kaybı yaşadı­
ğ ını, ağzında sık sık aftlar çıktığını ifade eden hastanın bakı­
lan laboratuvarından coombs (+) hemolitik anemi, lenfopeni,
trombositopeni ve C3 kompleman düşüklüğü izleniyor. Fizik
muayenesinden hepatomegali ve sp lenomega li tespit ed il en
hastamız gebe kaldığında sağlık lı bir bebeği olması adına han-
gi iki antikora da bakılmalıdır?
A. Anti-S m antikor --Anti-ds DNA
B. Anti nükleer antikor -- Anti -ds DNA
C. Anti Ro antikoru --Anti La antikoru
D. Anti La antikoru -- Anti nörona! antikor
E. Anti ribonükleoprotein --Anti ribozomal P

8- Veee hastamız gebe kalıyor. Bakt ığımı z anti- Ro ve La anti- 60r90J atıı:ııı#dk: Aın:tıii Re.,# lla
korları
pozitif gelen hasta için bu aşamadan sonra izlenmesi
gereken yolla ilgili hangisi yanlış bir yorumdur?
A. Hasta 16.haftadan itibaren feta l EKO'yl atakip ed ilmelidir
B. Takipte 1.derece AV blok geliş irse maternal deksametazon
verilir
l e l ı B,mdl e:

C. Takipte 2.derece AV blok ge lişirse fetüse kalıcı kalp pili


tak ılır
D. Takipte 2.derece AV blok ge li şirse fetüse kalıcı kalp pili
takılır
E. Doğumda raş ge liş ebil ir ama bu geçicidir
9· İki yıl önce sistemik lupus eritematosus tanısıyla takipli
hasta takipne ve taşikardi şikayetiyle acile getiriliyor. Acil
serviste bakılan vitallerinde ateşi 40°(, tansiyonu 110/60
mmH, kalp tepe atımı 108 atım/dk, saturasyon %81 izleniyor.
Hastaya çekilen bilgisayarlı tomografide iki akciğerde yoğun
interstisyum tutulumu, yaygın alveoler hasar, alveoler ödem
ve hiyalen membran oluşumu rapor ediliyor. Solunum açlığı
içinde izlenen hastaya% 100 oksijen sol utuluyor ancak yine de
saturasyonunda artış izlenmiyor. Bu hasta için en olası tanı
aşağıdakilerden hangisidir?
A. lupus pnömonisi
B. Diffüz alveoler hemoraji
C. Mikofenolat mofetil toksisitesi
D. Pneumocysticjiroveji pnömonisi
E. Renal tüberküloz

10· Bu hastada aşağıdaki antikor titrelerinden hangisinde artış


beklenir?
A. Anti-Sm antikor
B. Anti nükleer antikor
C. Anti Ro antikoru
D. Anti-ds ONA antikoru
E. Anti fosfolipid antikor

11· 29 yaşında bir gebe hasta çocuğunu miadında doğurduk­


tan sonra sağ hemiparezi ve sağ elde siyanoz gelişmesi üzerine
yoğun bakım ünitesine yatırılıyor. Fizik muayenede livedo
retikülaris dikkati çekiyor. Hemogramda lökosit 10200/mm3,
hematokrit %35' ve trombosit 17.000/mm3 geliyor. Ardından
periferik yayma yapılıyor ve hemogram sonucuyla uyumlu
geliyor ve şistosit gözlenmediği rapor ed iliyor. BUN 36 mg / dl
ve kreatinini 2.3 mg / dl' izlenen h astan ın öyküsünden daha
önce 3 defa gebelik kaybı olduğu öğren iliyor. Bu hastada en
olası tanıyı en iyi şekilde doğrulayan test aşağıdakiler­
den hangisidir?
A. Anti-kardiyolipin antikor paneli
B. Antinükleer antikor
C. Ekokardiyografi
D. Beyninin manyetik rezonans görüntüleme (MRI)
E. Karotis doppler USG

12- Vakada aşağıdakilerden hangisinin gelişmesinin nedeni


tromboz geliş sine b ğlı değildir?
1. Düşük öyküsü 11. Kreatinin yüks e kliği
111. Sağ elde siyanoz gelişmesi iV. Sağ hem iparezi varlığı
V. Livedo retikülaris
A.lvell B. 1- 11 -V
C. iV ve V D. 11 - 111 -V
E. 1- 11 - 111 -V
g g~ademisi
13- 50 yaşında kadın hasta el eklemlerinde ağrı, gözde yanma
ve ağızda kuruluk şikayetiyle başvuruyor. Hastaya tükrük bezi
biyopsisi yapılıyor ve fokal lenfositik infiltrasyon olarak yorumla-
nıyor. Yapılan Schirme(s testi 5 dakikada 4 mm olarak yorumla-
nıyor. Laboratuvar incelemede ANA (+)izleniyor. Aşağıdakiler­
den hangisi bu hastada lenfoproliferatif hastalık gelişme
riskini artırmaz?
A. Lökopeni
B. Persistan parotis büyümesi
C. Kriyoglobulinemi
D. Poliartrit
E. Kompleman düşüklüğü

14- 56 yaşındaki erkek hasta halsizlik, güçsüzlük ve iştahsızlık


şikayetiyle başvuruyor. Öyküsünden sandalyeden kalkarken,
mediven çıkarken çok zorlandığını özellikle de sabahları yatak-
tan kalkmak için çok çaba sarfettiğini ifade ediyor. Fizik mua-
yenesinde göz çevresinde mavi-mor lekelenme ve elin dorsal
yüzünde mor plaklar dikkati çekiyor. Bu hasta için en olası tanı r.

aşağıdakilerden hangisidir?
A. Dermatomiyozit
B. Polimiyozit
C. İmmün aracılı nekrotizan miyopati
D. İnklüzyon cisimcikli miyozit
E. Anti -sentetaz sendromu

15- Yukarıdaki vakada hangi antikorun pozitif gelmesi hastalı­


ğın malignite zemininde geliştiğini gösterir?
A. Anti-MDA 5
B. Anti TIF 1
C. Anti Mi 2
D. Anti HMGCR
E. Anti SRP

16- On yıldan beridir Raynaud fenomeni nedeniyle takipli 45


yaşındaki kadın hasta son 2 ayd ır giderek artan nefes darlığı
neden iyle başvuruyor. Fizik muayenede parmaklarda gergin lik,
ciltte sertlik izleniyor. Bakılan laboratuvar incelemede ANA
1/320 (+)ve anti-sentromer antikor (+)geliyor. Parmakları aşa­
ğıdaki gibi digital ülser ve iskemi nedenli parmakları şekildeki
gibi izlenen hasta için en olası tanı hangisidir?

A. Sınırlı sistemik skleroderma


B. Diffüz sistemik skleroderma
C. Sistemik lupus eritematosus
D. Romatoid artrit
E. İmmün aracı lı nekrotizan miyopati
17· Skleroderma nedeniyle takipli 53 yaşındaki kadın hasta
idrar miktarında azalma, bacaklarda şişlik şikayetiyle başvuru­
yor. Fizik muayenesinde bacaklarda +4 gode bırakan ödem
izleniyor ve tansiyonu 220/11 OmmHg ölçülüyor. Hastaya
takılan sondadan ilk 1 saatte 5 cc idrar çıkışı olduğu görülüyor.
Biyokimyada sodyum:124 mEq/L, potasyum:6,4 mmol/L, kalsi-
yum:?, 1 mg/dl, üre:150 mg/dL, LDH:7800 U/L ve kreatinin:4,8
mg/dl izleniyor. Hemogramda hemoglobin:8, 1 g/dl, trombosit
100.000/mm 3 geliyor. Hemogramda anemi ve trombositopeni
olması üzerine periferik yayma yapılıyor ve şekildeki gibi şistosit
görülüyor. İdrar sedimentinde ise yine şekildeki gibi bol miktar-
da granüler silendirler gözleniyor.

i -~

Bu hastanın en doğu tamsı aşağıdakilerden hangisidir?


A. Prerenal böbrek yetmezliği
B. Renal distal asidoz
C. Skleroderma renal kriz
D. Üreteryal obstrüksiyon
E. Kontrast nefropatisi

18- 64 yaşında kadın hasta halsizlik, baş ağrısı, saçlı deride


hassasiyet ve çene hareketlerinde kısıtlılık olması üzerine
başvuruyor. Bakılan laboratuvarda sedimentasyon 106 mm/h
izleniyor. Fizik muayenede temporal arter trasesinde şişlik ve
hassasiyet tespit ediliyor. Arterden yapılan biyopside şekildeki
gibi lamina elastika internada hiperplazi ve mononükleer dev
hücre infiltrasyonu rapor ediliyor.

Bu hasta için en olası tanı aşağıdakilerden hangisidir?


A. Dev hücreli arterit
B. Kawasaki hastalığı
C. Kutanöz lökositoklastik ang itis
D. Kriyoglobulinemik vaskülit
E. Takayasu arterit
2023 MARTTUS SORU KAM.Pi .·
l a· ı.Q , ~ .
I!!!! !!!!I aKademısı öatııu ·e · . · ·. \ -~. ·. · ··
~ . ·. . .· .
19- Vakadaki hastaya dev hücreli arterit tanısı konuluyor ve
ardından steroid tedavisiyle takibe alınıyor. Takipte steroid dozu
azaltılarak kesiliyor. Steroid kesildikten 5 ay sonra omuz ve
kalça eklemlerinde sabah tutukluğu şikayeti gelişmesi üzerine
yeniden hekime başvuruyor. Yapılan muayenede kas gücünde
kayıp izlenmiyor. Laboratuvarda bakılan kreatin kinaz, aldolaz,
miyoglobin seviyeleri normal aralıkta izleniyor. Yapılan USG de
1

omuz ekleminde bursitle uyumlu görünüm izleniyor. Hastaya


bu aşamadan sonra başlanması gereken tedavi aşağıdakiler­
den hangisidir? .
A. Pulse steroid B. Düşük doz oral steroid
C. Aspirin D. Tocilizumab
E. Abatacept

20· 45 yaşındaki erkek hasta iki gündür devam eden sağ


elde güçsüzlük şikayetiyle başvuruyor. Öyküsünden 2 ay önce
hipertansiyon tanısı a ldığı ve bu nedenle kaptopril ve furose-
mid kullandığı öğreniliyor. Ayrıca 2 ay içinde istemsiz şekilde
8 kilo kaybettiğini ve yemeklerden sonra gelişen karın ağrıları QJ
olduğunu söylüyor. Fizik muayenede tansiyonu 160/105 mmHg
ve sağ el bileğini ekstansiyona getirmede güçlük dışında bir ~
bulgu izlenmiyor. Laboratuvarında sedimentasyon 98 mm/h, ~ l!\I

AST:125 IU/ml, ALT:165 IU/ml olarak izleniyor. Bu hastada ·


tanıya yönelik aşağıdaki laboratuvar testlerinden hangisi ilk
olarak yapılmalıdır?
A. HBs antijeni
B. HCV RNA
C. Antisitoplazmik nötrofil antikor
D. Mezenter arter angiografisi
E. Radial arter biyopsisi

21- 55 yaşında erkek hasta 6 aydır devam eden halsizlik,


ateş, kilo kaybı şikayetiyle başvuruyor. Öyküsünden 2 ay önce
hemoptizi, öksürük, idrar renginde koyulaşma şikayetiyle tetkik
edild iği, kronik sinüzit tanısı konulduğu ve bu sebeple antibiyo-
tik kullandığı ayrıca zaman zaman kanlı pürülan burun akıntısı
ve paranazal sinüs trasesinde ağrı olduğu öğreniliyor. Yapılan
laboratuvar incelemede sedimetasyon 105 mm/h, CRP 21 mg/
dl izleniyor. Hastaya çekilen Brde şekildeki gibi kaviter lezyon-
lar ve pulmoner nodüller dikkati çekiyor.
~

- ......ı!l
Vakadaki hasta için en uygun başlangıç tedavisi hangisidir?
A. Glukokortikoid + Mikofenolat mofetil
B. Metotreksat
C. Rituksimab
D. Glukokortikoid + Siklofosfamid
E. Plazmaferez
22· 45 yaşında kadın hasta halsizlik, kaşıntı, geniz akıntısı ve
nefes darlığı şikayetleriyle başvuruyor. Öyküsünden 5 yıl önce
astım tanısı aldığı ve inhaler bronkodilatör kullandığı öğrenili­
yor. 3 yıl önce gastrointestinal sistem kanaması geçirdiği ve bir
yıl önce de geçirdiği baygınlık sonrası yapılan incelemede A-V
blok tespit edildiği öğreniliyor. Fizik muayenede ciltte gözlemle-
nen nodüllerden alınan biyopside eozinofilik vaskülit saptanı­
yor. Yapılan tam kan sayımında mutlak eozinofil sayısı 2000/
mm 3 ve posteroanterior akciğer grafisinde paran ki mal opasite-
ler tespit ediliyor. Bu hasta için en uygun tedavi hangisidir?
A. Glukokortikoid + Mikofenolat mofetil
B. Metotreksat
C. Rituksimab
D. Glukokortikoid + Siklofosfamid
E. Plazmaferez

23- Aşağıdaki hangisi yukarıda verilen hastanın en olası tanısıy­


la ayırıcı tanıya girmez?
A. Bronkosentrik granülomasyon
B. Hipereozinofilik sendrom
C. Hipersensitivite pnömonisi
D. Alerjik bronkopulmoner aspergillozis
E. Eozinofilik pnömoni

24· 44 yaşında hastanede çalışan bir hemşire her iki ayağında


döküntü ve diz eklemlerinde hareketle ağrı ve hassasiyet olması
üzerine başvuruyor. Öyküsünden 3 ay önce eline iğne battığı
ancak tetkik ettiremediği öğreniliyor. Hepatit serolojisinde
anti-HVC (+)izlenen hastanın biyokimyasında üre 78 mg/dL,
kreatinin: 2,7 mg/dl izleniyor. Tam idrar ana lizind e protein
(+++) ve idrar mikroskopisinde her büyük büyütme alanında
50 eritrosit izleniyor.

Fizik muayenede döküntüleri yukarıdaki gibi izlenen hasta için


en olası tanı hangisidir?
A. Cogan hastalığı
B. Miks kriyoglobulinemi
C. İdiyopatik kütanöz vaskül it
D. Hipokomplemanemik ürtikeryal vaskülit
E. Mikroskopik poliangitis

25· Yukarıda veri len hasta için aşağıdaki l erden hangisi kötü
prognoz göstergesidir?
A. Palpable purpura varlığı B. Eklemlerde ağrı ve hassasiyet
C. Anti-HCV pozitifliği D. Üre/ kreatinin yüksekliği
E. Kompleman düşüklüğü
2023 MARTTUS SORU KAMPI
Dahili e
26- 23 yaşındaki erkek hasta derin ven trombozu nedeniyle
acile başvuruyor. Öyküsünden 6 ay önce de benzer şekilde acil
başvurusu olduğu, yılda en az 1Odefa oral aft çıkardığı, daha
önce 2 defa genital bölgesinde ülser çıktığı ve bu ülserlerin iyi-
leştikten sonra iz bıraktığı öğreniliyor. Fizik muayenesinde her
iki bacakta deriden kabarık, kızarık ve ağrılı döküntülerin varlığı
dikkati çekiyor. Bu hasta için en olası tanı hangisidir?
A. Cogan hastalığı
B. Miks kriyoglobulinemi
C. Behçet hastalığı
D. Hipokomplemanemik ürtikeryal vaskülit
E. Mikroskopik poliangitis

27- Aşağıdakilerden hangisi Behçet hastalığı için olmazsa


olmaz tanı kriteridir?
A. Tekrarlayan genital ülser
B. Oral aftlar
C. Göz tutulumu
D. Deri lezyonları
E. Paterji testi pozitifliği

28- 45 yaşında kadın hasta 12 yıl önce başlayan hareket ve


egzersizle azalan bel ve topuk ağrısıyla başvuruyor. Öyküsünden
ailesinde de benzer şikayetlerin olan bireyler olduğu, ağrılarına
yönelik birçok farklı NSAİİ ve sulfsalazin kullandığı ve bu teda-
viyle ağrılarının kısmen geçtiği ancak tam rahatlama sağlama­
dığı, 2 hafta önce görmede azalma şikayeti gelişmesi üzerine
anterior üveit tanısı aldığı öğreniliyor. Çekilen grafide sakroiliak
eklemde bilateral sakroileit ve akciğer grafisinde akciğer apek-
sinde fibrotik değişiklikler izleniyor. Bakıl a n laboratuvarınd a
HLA B27 (+)ve sedimentasyon 124 mm/h izleniyor. Vakadaki
hasta için bu aşamadan sonraverilmesi gereken en uygun
tedavi hangisidir? ·
A. Metotreksat
B. infli ksimab
C. Hidroksiklorokin
D. Leflunomid
E. Sistemik steroid

29- Periferik eklem tutulumu olan, aksiyel eklem tutulumu


olmayan yeni tanı ankilozan spondilit hastasında ilk tercih en
uygun tedavi kombinasyonu hangisidir?
A. Golimumab
B. NSAİİ + Sulfasalazin
C. Sulfasalazin
D. Metotreksat + NSAİİ
E. NSAİİ + İnfliksimab
30· Hangi anti-TNF ilaç paradoksal üveiteyol açabilir?
A. İnfliksimab
B. Etanercept
C. Adalimumab
D. Sertolizumab
E. Golimumab

31 · 41 yaşında erkek hasta 6 yıl önce başlayan hareket ve


egzersizle azalan bel ve topuk ağrısıyla başvuruyor. Öyküsünden
ailesinde de benzer şikayetlerin olan bireyler olduğu, ağrılarına
yönelik birçok farklı NSAİİ ve sulfasalazin kullandığı ve bu teda-
viyle ağrılarının kısmen geçtiği ancak tam rahatlama sağlama­
dığı, 1 hafta önce görmede azalma şikayeti gelişmesi üzerine
anterior üveit tanısı aldığı öğreniliyor. Çekilen grafide sakroiliak
eklemde bilateral sakroileit ve akciğer grafisinde akciğer apek-
sinde fibrotik değişiklikler izleniyor. Yapılan fizik muayenede
şekilde gibi pigmente oluşumlar dikkati çekiyor.

Bu hasta için en olası tanı aşağıdakilerden hangisidir?


A. Ankilozan spondi lit
B. Diffüz idiyopatik skeleta l hiperostozis
C. SAPHO sendromu
D. Okronozis
E. Reaktif artrit

32· Ülsetarif kolit nedeniyle takipli 57 yaşındaki erkek hasta


2 gün önce başlayan sağ metakarpofalangeal ve sağ 3.dista l
interfalangea l eklem lerinde ağrı ve kanlı dışkılama şikayetiyl e
başvuruyor. Fizik muayenede bacakta şekildeki gibi yarası
olan hastada bakılan laboratuvar incelemede HLA B27 (+ )'liği
izleniyor.

Bu hastaya anti-TNF tedavi başlanıyor ve takipte kanlı dışk ıl ama­


sı geriliyor ve eklem lerindeki ağrı kayboluyor ancak dirsek ön
yüzde eritemli zeminde gümüş, beyaz skuam lı, iyi sınırlı papül/
plak gelişiyor. Bu hastaya başlanan hangi anti-TNF ilaç buna
yol açmış olabilir?
A. infliksimab B. Etanercept
C. Adalimumab D. Sertolizumab
E. Golimumab
33- Bir yıldır hipertansiyon ve ateroskleroz nedeniyle tiazid ve
aspirin (300 mg/g•ün) kullanan 50 yaşında bir erkek hasta sol
ayak bileğinde ve metatarsofalangeal eklemde 24 saat önce
başlayan ağrı, şişlik ve kızarıklık yakınmalarıyla başvuruyor.
Hastanın öyküsünden 6 ay önce sol dizinde, 3 ay önce de sağ
ayak bileğinde ağrı ve şişme olduğu; yaklaşık 4-5 günde iyileş­
tiği öğreniliyor. Yapılan laboratuvar incelemelerinde eritrosit
sedimentasyon hızı 58 mm/saat, lökosit sayısı 17 .000 mm 3 ve
C-reaktif protein düzeyi 45 g/L olarak bulunuyor. Bu hasta için
en olası gut sebebi aşağıdakilerden hangisidir?
A. Ateroskleroz varlığı B. Aspirin kullanımı
C. Tiyazid kullanımı D. Hipertansiyon varlığı
E. Yüksek akut faz yanıtı

34- 24 yaşındaki kadın hasta sağ ayak bileğinde ağrı ve kıza­


rıklık n-edeniyle başvuruyor. Öyküsünden son 3 yılda 4-5 defa
ortaya çıkan diz ve ayak bileklerini tutan1 hafta içinde kendi-
liğinden geçen eklem ağrıları olduğu öğreniliyor. Anam nez (1)
derinleştirildiğinde son 5 yılda zaman zaman nefes almakla
batıcı tarzda göğüs ağrısı atakları olduğu, son 2 yıldır da adet
öncesi ve sonrasında şiddetli karın ağrısı atakları ve teyzesinde
de benzer şikayetlerin olduğu öğreniliyor. Fizik muayenede
şek ildeki ayak iç yüzünde kızarıklık izleniyor.

Bu hastada en olası tanı hangisidir?


A.Ailevi akdeniz ateşi B. Relapsin polikondrit
C. lg G4 ilişkili hastalık D. Fibromiya lji
E. Erişkin başlangıçl ı stili hastalığı

35- Vakadaki hastanın tan ı sında yer alan en önemli proinflama-


tuvar sitokin hangisidir?
A. İnterlökin 1a B. İnterlökin 1b
C. İnterlökin 4 D. İnterlökin 8
E. interlöki n 1O

36- Ailevi Akdeniz ateşin in tedavisinde kullanılan kolşisin ile


ilgili olarak aşağıdaki ifadelerden hangisi doğrudur?
A. Tüm hastalarda düzenli kullanımı önerilir
B. Seyrek atak geçiren hastalarda yalnızca atak sırasında kullanı ­
mı önerilir
C. Kolşisin tedavisine yanıt vermeyen hastalarda tedavinin
son l and ırılm ası önerilir
D. Gebelikte kontrendikedir
E. Dozu hastanın vücut ağırl ı ğ ın a göre ayarlanma lıdı r
37- 41 yaşındaki kadın hasta, 4 hafta önce başlayan boğaz
ağrısı, boyunda ele gelen şişlikler ve yüksek ateş şikayetleriyle
başvuruyor. El bileklerinde ağrı ve şişlik olduğunu ifade eden
hastanın öyküsünden, ateş nedeniyle verilen seftriakson tedavi-
si sonrasında kol ve bacaklarında kırmızı renkte döküntüler or-
taya çıktığı öğreniliyor. İlaç erüpsiyonu düşünülerek antibiyotik
tedavisi kesilen hastanın son 15 gündür gün içerisinde 40 °Cyi
bulan ve kendiliğinden düşen ateşi ve el bilek eklemlerinde
şişlik yakınmaları devam ediyor. Laboratuvar incelemelerinde
lökosit: 21.000/mm3, hemoglobin: 10,2 g/dl,ALT: 60 U/L,
eritrosit sedimentasyon hı z ı: 98 mm/saat ve CRP: 56 mg/L bu-
lunuyor. ANA ve romatoid faktör negatif tespit ediliyor. Hastada
takipte halsizlik ve iştahsızlık şikayetinin artması üzerine acile
başvuruyor. Acilde bakılan hemogramında lökosit:4000/mm 3,
nötrofil :850 /mm 3, lenfosit: 1000/mm 3, trombosit: 110.000/
mm 3, hemoglobin izleniyor.Biyokimyada INR 2, 9 geliyor ve
fizik muayanede cilt altı purpuralar izleniyor. Bu hastada hangi
komplikasyon gelişmişt ir?
A. Hemofagositik sendrom B. Akut lenfoblastik sendrom
C. Seftriakson intoksikasyon D. Felty sendromu
E. Diffüz büyük B hücreli lenfoma

38- 45 yaşında erkek hasta sağ kulakta şişlik ve kızarıklık sebe-


biyle başvuruyor. Öyküsünden son 4 aydır işitmesinde azalma
olduğunu, artık televizyon seyrederken sesi yükseltmek zorunda
kaldığı, bu yüzden apartman sakinleri tarafından sık sık şikayet
edildiği sol 1.metakarpofalangeal eklemde de ağrısının olduğu
öğreniliyor. Fizik muayende semer burun deformitesi olduğu
görülen hastanın en olası tanısı hang isidir?
A.Ailevi akdeniz ateşi B. Relapsing polikondrit
C. lg G4 ilişkili hastalık D. Fibromiyalji
E. Erişkin başlangıçlı stil! hastalığı

39- Vakad aki hastan ın patogenezinden hangisi rol oynar?


A. Tip 1 kollajene karşı aş ırı duyarlılık
B. Tip 2 kollajene karşı aşırı duyarlılık
C. Tip 3 kollajene karş ı aş ırı du yarlılık
D. Tip 4 kollaj ene karşı aşırı duyarlılık
E. Tip 3 / 4 kollajene karşı aş ırı duyarlılık

40- 71 yaşında bir kadın hasta distal interfa langea l eklemlerin-


de kullanm akl a artan ağ rı ve kemik büyümesi yakınm a l a rıyla
başvuruyor. Fizik muayenede distal interfalangea l eklem lerd e
hassasiyet ve eklem hattında kemik lehi ne büyüme saptan ıyor.
Direkt grafide ekl em a ra lığında daralma ve osteofit görülüyor.
Bu hastanın ağrı palyasyonunda en iyi ilaç hangisid ir?
A. Diklofenak B. Parasetamol
C. Metilprednizolon D. Kolşisin
E. Klorokin
la
l.!!!!!!!!!I tı~
2023 MARTTUS SORU KAMPI
► " Dahili e
OKad emısı
GERİATRİ
1· 85 yaşında kadın hasta sinirlilik, zaman zaman olan çarpıntı
hissi ve ishal şikayetiyle başvuruyor. Bakılan laboratuvarda TSH
düşük; T3 ve T4 ise yüksek izleniyor. Hastaya çekilen sintig-
rafide çekildeki gibi toksik multinodüler guatr ile uyumlu
görünüm izleniyor.

Bu hastanın tedavisi için en uygun seçenek hangisidir?


A. Tiroidektomi
B. Radyoaktif iyot tedavisi
C. Metimazol
D. Glukokortikoid
E. İyot lugol solusyonu

2-Aşağıdakilerden hangisi geriatrik yaş grubunda koruyucu


hekimlik uygulamaları kapsamında yer almaz?
A. l<ud uz aşısı
B. Pnömokok aşısı
C. İnfluenza aşısı
D. l<adın hastalar için smear taraması
E. Yıllık gaitada gizli kan tetkiki

• İnterst isyel fibrozise bağlı tübüle r atrofi


Tübül ve • Kreatinin kliren si azalır
İnte rstisyel * Na + geri emilimi ve K• sekresyo nu aza lır
ai * İdrarın dilüsyon ve konsantrasyon yeteneği aza lır
;g
3 -Aşağ ıdakilerden hangisi yaşla birlikte görülen renal deği­ 32 * pH tamponizasyon yeteneği aza lır
·;;;. * RAAS sistemi yavaşlar
şikliklerden biri değildir? io'.b
cıı • ADH duyarlılığı aza lır
o
cıı

A. Nefrotik düzeyde proteinüri -o


E • Bazal ANP düzeyi artar
cıı
-:;: * Pl azma renin aktivitesi azalır
B. Sklerotik glomerül sayısı artar i.i'i
-. Sıvı - elektrolit .....
• Bazal aldosteron salgısı azalır
* En s ık sıvı-elektrolit bozukluğu hiponatremidir
C. Renal plazma akımı azalır * Hiperkalseminin en sık sebebi malignitelerdir
* Hipokalsem inin en s ık sebeb i hipoalbumin em idir
D. Arteriolar hiyalinozis artar
* Renal arterde kalınl aş ma ve renal kan akımı azalır
E. Sodyum dengesi bozulur Vask üler
* Küçük arterlerde arteriosklero z
• Afferent / efferent arteriollerde intimal kalınlaşma
* Hiyal e n ve kollajen sube ndote lyal aland a birikir
* Renal vasküler direnç artar

* D vitamin emilimi aza lır


İnc e barsa ki ar -+ "' Çinko ve kalsiyum e rnilimi aza lır
* 812 ve folat em ilimi aza lır
* D e mir e milimi bozulmaz
4-Aşağ ıdakilerden
hangisi yaşla birlikte görülen gastroin- * Villüslerd e atrofi izlenir
* ince barsak boşalm a zamanı normaldir

testinal sisteme ait değişikliklerden biri değildir? *


*
Laktoz intoleransı s ıklığı artar
Yağ~ protein ve k.hidrat e milirni değişmez
* Mezenter iske mik riski a rtar
A. l<onstipasyon sıklığı artmıştır * Mukozal İmmünite ve lg A üretimi azalır

B. C.difficili kolononizasyonu sık görülür


* Peristaltizm yavaşlar
C. Folat, B12 eksikliği ve demir eksikliği anemisi izlenebilir *
*
l<onst ipasyon ve fekal imp aks iyonu ri sk i artar
Divertikül görülıne s ıklı ğ ı artar

D. Barsak mukozasında lg Asalgısı ve immünite azalır *


*
Alt GİS kanama riski arta r
Mukozal proliferasyon artmış iz le nir
Kalın barsaklar -.. * Rektal genişleme azalır
E. Mukoza ! kan akımı azalır ve NSAİİ'lara bağlı yan etkiler sıktır *
*
Beklenmedik a n a l gevşe m e sonu c u fekal urgency
intern a l a nal sfinkter basıncı aza lnıı şt ır
* İ ske mik kolite yatkınlık arta r
5- Aşağıdakilerden hangisi yaşla birlikte görülen endokrin * İyot eks ikli ğ i olan bölgelerde TSH
* tT3 ve·sT 3 düzeyi aza lır
düzeyi artar

_ * tT4 ve sT4 düzeyi genellikl e değişmez


sistemde tiroid bezine ait değişikliklerden biri değildir? Tiroid
* Subklinik hipotlrodi sıklığ ı arta r
• T4 ➔ T3 dönüşümü azalır
A. T4'ün T3'e dönüşümü azalır * Tiroid Otoantlkor sık lığı artar

B. Otoimmünite azaldığı için otoimmün hastalıklarda otoanti-


kor (+ )'liği daha nadir izlenir
C. Total ve serbest\ düzeyi azalır
D. Total ve serbest T4 düzeyi genellikle değişmez
E. Subklinik hipotiroidi sıklığı artar

* GHRH azalır ➔ GH / IGF 1 sa l g ı s ı azalır


• GnRH düzeyi artar ➔ FSH ve LH artar
6- Hangisi geritatrik yaş grubunda endokrin sistemde Hipotalamus - * Prolaktin düzeyi a zalır
* ACTH düzeyi v e CR H'ya ACTH yanıtı değişmez

hipotalamusa ait değişikliklerden biri değildir? *


*
Cerrahi strese karşı kortizol yanıtı aza lır
Kortizol üretimi artm ı ş/ aza lmı ş/ normal
• Hipotalamo-hipofizer aksın korti zol duyarlılığı aza lır
A. ACTH düzeyinde değişiklik beklenmez
B. Prolaktin düzeyi azalır
C. GHRH salgısı ve buna bağlı GH / IGF-1 sa lgısı aza lır
D. Cerrahi strese karşı kortizol yanıtı artar
E. Hipotalamo hipofizer aksın kortizol duyarlılığı azalır

7- Aşağıdakilerden hangisi yaşla birlikte görülen kardiyovas- * Aortta kalsifikasyon ve buna bağlı damar seti iği
* İl eti sisteminde fibrozis görülür
küler sisteme ait değişiklikle rd en biri değildir? * Kalp debisi, atım hacmi, kalp hızı ve EF normaldir

A. İleti sisteminde değişiklik izlenmez Kardiyovasküler


* EKG'de sol aks deviasyonu izlenebilir
-+ · • EKG'de PR, QRS ve QT uzaması görü lebilir
B. Diyastolik kan basıncı azalır sistem * Sistolik kan basıncı artar
* Diyastolik kan basıncı aza lı r
C. EKG'de sol aks deviasyonu izlenebilir * Nabız basıncı artar
* Yaşla sol ventrikül kitlesi artar -+ Diyastolik disfonk.
D. İstirahatte kalp debisi, atım hacmi, kalp hızı ve EF normaldir * S4 daha belirgin duyulabilir

E. Yaşla sol ventrikül kitle~i artar

8- Aşağıdakilerden hangisi geritatrik yaş grubund a hemato- * Hematopoez normaldir


poetik sisteme ait değişikliklerden biri değildir? Hematopoetik ➔ * Eritrositlerin yaşam ömrü gençlerd eki gibidir
sistemi * Lökosit ve trombosit sayıları değişmez
A. Eritrositlerin yaşam ömrü gençlerdeki gibidir * Eritropoetin düzeyi artmıştır

B. Yaşlılarda aneminin sıklığı artmıştır


C. Hematopoez normaldir
D. Lökosit ve trombosit sayıları değişmez
E. Eritropoetin düzeyi azalmıştır

9- Aşağıdakilerden
hangisi bir yaş lıda değerlendirilen Temel gün lük yaşam aktiviteleri ölçeği :
günlük temel yaşam aktivitelerinden biridir? Banyo
A. Giyinme Tuvalet
Beslenme
B. Telefon kullanabilme
Dışkılama
C. İlaçlarını kullanabilme
Giyinme
D. Para idaresi
Yürüyeb ilme
E. Yem ek yapabilme
tllı ~flademisi
10- Aşağıdakilerden hangisi geriatrik yaş grubunda boyu
ve ki losu ölçülemeyen hastalarda vücut kitle indeksi yerine
kullanılan en duyarlı yöntemdir?
A. Üst ko l çevresi ölçü mü B. El kavrama gücü
C. Biyoelektrik impedans analizi D. Baldır çevresi ölçümü
E. Triseps cilt kalınlığı ölçümü

11- Nutrisyonu değerlendirmede en çok kullanılan para-


metreyle en az kullanılan parametre ikilisi hangisidir?
A. Prealbumin -- Retinol bağlayıcı globulin
B.Albumin --Transferrin
C. Prealbumin -- Albumin
D. Retinol bağlayıcı globulin -- Transferrin
E.Albumin -- Transferrin

12- Yaşlılarda üriner inkonsitnansın en sık sebebi


hangisidir?
A. Urge (sıkışma) inkontinansı B. Taşma inkontinans
C. Miks inkınsitnans D. Fonksiyonel inkon Strfts lnl\ontlnaınsı
Östrojc n eksik li ğ i,
Urge
A ş ın
lnkontlna,nsı
aktif nı c s ;:ınc
T.;ışma Jnkontln.ıınsı
Dctrüsör kasın yetersiz N~~:~o1;~:t;1':~;1: ı::~t
1 1

multlparitc, travmatik veya d ctrOsör dlsfonkslyonu so nucu

E. Stres inkontinansı doeu ıTI

so nucu
nedeniy l e
s tlnktc r tonus aza l ma s ı
ge li şen
hip c rrc.:ı ktivi tc
son u c u ecllşcn
lnk ont in .::ı n s t ip i
ge li şe n
taşma
lnkontln a n s
lnl<ontln a n s tipi
gelişen lnkontin a n s
tipi

lnl<o ntlnans tipi

13- Deliryum tan ı sı konulan hastada aşağıdaki tetkiklerden


hangisinin yapılması öncelikli deği dir?
A. Üre/ kreatinin düzeyi
B. Plazma glukoz düzeyi
C. Serum elektrolit seviyesi
D. Gaita mikroskopisi
E. Bilgisayarlı beyin tomografisi

14- Bası yaraları hakkında hangisi doğru bir yorum


değildir?
A. Bası yarası oluşmasında en önemli risk faktörü basınç
Deri -
artışıdır Yumuşak J
B. Yatar pozisyonda en sık oksiput, saktum ve topuk etkile- doku 1
nirken; otur pozisyonda ise bası yaraları en sık iskiyum ve
trokanter bölgesinde izlenir Evre 1 Evre 2 Evre3 Evre4
Basmakla solmayan Dermisi içeren deri Deride tam kat Tam kat deri
C. İdrar ve gayta inkontinansı riski artıran durumlardandır eritemli cilt. Deri kaybı. Deride nekroze şekilde tutulumu ve kas,
D. Bası ya ras ını değerlendirmek için Norton ve Braden risk bütünlüğü ayrılma ve bül tutulum. Derin ligaman tendon,
bozulmamıştır. görülebi lir. çukur oluşumu olur. kık ırdak tutulumu
değerlendirme skalası kullanılır
E. Bası yaralarının çoğu cerrahi tedavi gerektirir

15- Aşağıdakilerden hangisi sarkopeni tanısında hem kas


kitlesini anlamlı şekilde gösterir hem de sarkopeni progno-
zunu öngörmekde faydahdır?
A. Biyoempedans analiz
B. Ultrason
C. Lomber 3.vertebranın BT ile incelenmesi
D. Kas gücü muayenesi
E. DEXA çekilmesi
ENFEKSİYON HASTALIKLARI
1. Kırk yedi yaşındaki erkek hasta; baş ağrısı, hatırlamada
güçlük, yeni başlangıçlı alzheimer, zaman zaman hafıza kaybı ve
kişilik değişikliği tablosuyla getiriliyor. Öyküsünden sol dizinde
ağrı, karın ağrısı, kronik diyare ve istemsiz kilo kaybı olduğu
öğreniliyor. Fizik muayenesinde oftalmopleji, nistagmus ve
bazen sıklaşan miyoklonik ataklar geçirdiği gözleniyor. Kraniyal
görüntülemede şekildeki gibi parameningeal bölgede kontrast
tutulumunda artış saptanan hastanın beyin omurilik sıvısında
protein artışı ve 510/mm 3 boyalı hücre görülüyor.

Hastanın BOS sitosantrifüjünde periodik asit-Schiff (PAS) pozitif


boyanan makrofajlar tespit ediliyor. Duodenal biyopside de
lamina propriada PAS pozitif makrofajlar saptanıyor. Alınan bi-
yopsi materyalinden yapılan PCR incelemede gram pozitif basil
izleniyor. Hastada bu klinik tabloya neden olan en olası etken
aşağıdakilerden hangisidir?
A. Borrelia burgdorferi
B. Stafi lokokkus aureus
C. Entamoeba histolytica
D. Tropheryma whipplei
E. Toxoplasma gondii

2. 62 yaşında kadın hasta 1Ogün önce öksürük ve ateş ya-


kınmaları nedeniyle acile başvuruyor. Çekilen a kciğer grafisi
pnömoni lehine değer l endiri l en hastanın hesaplanan CURB-65
skoru 3 geliyor ve hasta yoğun bakıma yatırılarak intravenöz
antibiyotik tedavisi başlan ıyo r. Taburcluğunun 2.gününde hi-
pokondriyumda ağrı şikayetiyle acil servise başvuruyor. Çekilen
karın ultrasonografisinde şeki l deki gibi safra çamuruyla uyumlu
görünüm izleniyor.

Bu hastada bu duruma yol açması en muhtemel antibiyotik


aşağıdak il erden hangisidir?
A. Moksifloksasin
B. Klaritromisin
C. Seftriakson
D. Sefuroksim
E. Linezolid
3. Kreatinin klirensi 5 mi/dakik altında olan bir
hastanın tedavisinde, aşağıdaki antibiyotiklerden hangisinin
dozunun ayarlanmas ı na gerek yoktur?
A. Sefepim
B. Ampisilin-s ulbaktam
C. Meropenem
D. Klaritromisin
E. Moksifloksasin

4. Dört gündür devam eden bilinç ka y b ı, yüksek ateş ve


ş iddetli sol yan ağr ısıyla hasta nede takip edilen 65 yaşınd a bir
erkek hastada yapılan fizik muayenede pis kokulu balgam, piyo- Ul
re ve sol akciğer tabanınd a yaygın raller ve matite izleniyor. Bu
hasta için en olası tanı ve buna yönelik en uygun edilen hasta
tedavi aşağıdakilerden hangisidir?
A. HSV pnömonisi -Asiklovir
B. İnterstisyel pnömoni -Ampisilin
C. Aspirasyon pnömonisi - Klaritromisin
D. CMV pnömonisi - Gansiklovir
E. Aspirasyon pnömonisi - Piperasilin-tazobaktam

5. Sjögren sebebiyl e 3 yıldır immünosupresif tedavi alan 54


yaşındaki kadın hasta, öksürük ve ateş şikayetiy l e başvuruyor.
yapılan fizik muayenede sol alt ve orta zanda matite ve vi bras-
yon torasikte artış izlenmesi üzerine pnömoni teşhisi konuluyor.
Ba lgam incelemesinde şe kil deki gibi gram pozitif, dallanan
ve filamentleri olan, aside dirençli mikroorganizmalar tespit
ed iliyor.

Bu hastada ampirik tedavi için ilk tercih antimikrobiyal ilaç


aşağ ıd ak il erden
hangisidir?
A. Trimetoprim-su lfametoksazol
B.Amikasin
C. İmipenem
D. Linezolid
E. l(olistin
ıDJ ~~ademisi
6. Otuz iki yaşındaki kadın hasta dört günden beridir ateş, baş
ağrısı ve şuur bulanıklığı nedeniyle getiliriyor. Hastanın öykü-
sünden anti-HIV (+)olduğu ve il aç larını düzenli kullanmadığı
öğreniliyor. Laboratuvarda CD4+ lenfosit 75 hücre/µL bulunu-
yor. Beyin manyetik rezonans görüntülemede ödemle çevrili ve
halka tarzında kontrast tutulumu gösteren multiple lezyonlar
görülüyor.

Bu hasta için en olası tanı aşağıdakilerden hangisidir?


A. Sitomegalovirüs ensefaliti
B. Progresif multifokal lökoensefalopati
C. Primer santral sinir sistemi lenfoması
D. Toksoplazma ensefa liti
E. Epstein - Barr virüs ensefaliti

7. Ateş, baş ağrısı, ense sertliği, bilinç bulanıklığı şikayetleriyle


hastaneye başvuran hastanın yapılan lomber
ponksiyon bulguları şöyledir:
BOS basıncı 210 mmHg
Hücre sayıs ı > 1500/mm3 (Nötrofil hakimiyeti)
Protein: 120 mg/dl
Glukoz: 15 mg/dl
Bu hastada ampirik olarak en uygun antimikrobiyal
ajan aşağıdakilerden hangisinde verilmiştir?
A. Vankomisin + Seftriakson
B. Yalnızca seftriakson
C. Gansiklovir
D. Penisilin G
E. Amikasin

8. Tarım işiyle uğraşan 67 yaşındaki kad ı n hasta ani başlayan


ateş, baş ağrısı
ve bulantı - kusma yakınmal a rıyla acil servise
başvuruyor. Fizik muayenede ense sertliği saptanan hastada göz
dibi incelemesinde şekildeki gibi papil ödem gözleniyor.

Bu hastada bakteriyal menenjite yönelik yaklaşımın basamakla-


rı aşağ ıd akilerin hangisinde doğru sırayla verilmiştir?
A. Lomber ponksiyon - BOS incelemesi - kan kültürü
B. B il g i sayar lı BT - lomber ponksiyon - ampirik tedavi
C. Kan kültürü - BT - lomber ponk. - ampirik tedavi
D. Kan kültürü - ampirik tedavi - BT - lomber pansiyon
E. Ampirik tedavi - bilgisayarlı BT - lomber ponksiyon
ıDı ~ftademisı
9. Üroloji servisinde varikosel operasyonu sonrası yatan 56
yaşındaki erkek hastaya aldığı-çıkardığı sıvı takibinin yapılması
için idrar sondası takılıyor. Takipte ateşi olan hastadan alınan
idrar kültüründe genişlemiş spektrumlu ~ laktamaz (ESBL)
üreten bir bakteri kolonizasyonu rapor ediliyor. Ayrıca üreyen
bakterinin şekildeki gibi iri mukoid koloniler oluşturduğu, lak-
toz pozitif, oksidaz negatif, hareketsiz, gram negatif basil olduğu
öğreniliyor.

Bu olguda aşağıdaki antibiyotiklerden hangisinin tedavide


faydası yoktur?
A. Sefotaksim ıı.
en
B. Levofloksasin
C. Fosfomisin
E
D. Meropenem
E. Amikasin

10. İzmir'in torbalı ilçesinde bir köyde yaşayan 42 yaşındaki


kadın hasta yaklaşık 3 haftadır var olan akşama doğru yükselip
saba ha karşı terlemeyle düşen ateş, halsizlik, iştahsızlık, eklem-
lerinde ağrı, özellikle de eğilip kalkarken ciddi zorlanma ve kilo
kaybı şikayetiyle başvuruyor. Fizik muayanede ateş 39. 0(, nabız
112/dakika, soluk sayısı 15 soluk/dk, tansiyon 120/90 mmHg
ölçülüyor. Ayrıca karaciğerin kot altından 4 cm aşağıda ele gel-
diği, dalağın da inguinal bölgede palpe ed ildiği tespit ediliyor.
Sa kroiliac eklem germe testi pozitif izleniyor. Laboratuvarda
%80'i mononükleer hücrelerden oluşa n lökositoz, sedimentas-
yon ve CRP art ı şı izleniyor. Kan kültüründe şeki ld ek i gibi gram
negatif, oksidaz ve katalaz pozitif, hareketsiz kokobasil üremesi
rapor ediliyor.

Bu hastada ilgili klinikten sorumlu en olası etken aşağ ıd aki l er­


den hangisidir?
A. Brucella melitensis
B. Camplybacter jejuni
C. Burkho lderia pseudomallei
D. Salmonella typhi
E. Shigella flexneri
11. 81 iki yaşında erkek hasta ateş, üşüme, titreme ve bel ağrısı
şikayetiyle acile getiriliyor. Bakılan laboratuvar tetkiklerinde
lökosit sayısı 26.000/mm3 (%90 nötrofil), eritrosit sedimen-
tasyon hızı 81 mm/saat olarak saptanıyor. İdrarın mikroskopik
incelemesinde şekildeki gibi her sahada 30 lökosit ve bol
bakteri görülüyor.

İdrar kültüründe ise gram negatif, laktoz pozitif, hareketli bak-


teriler izole ediliyor. Bu hastada düşünülmesi gereken en olası
klinik tablo aşağıdakilerden hangisidir?
A. Akut nonkomplike sistit
B. Akut komplike piyelonefrit
C. Akut nonkomplike piyelonefrit
D. Asemptomatik bakteriüri ffl
w
E. Perinefritik apse

12. Bilinen bir hastalık öyküsü olmayan 54 yaşındaki obez bir


kadın karın ağrısı, bulantı, kusma, üşüme ve titreme yakınma ­
larıyla acile başvuruyor. Yapılan fizik muayenede ateş 39.8°(
olarak ölçülüyor ve karın sağ üst kadranda derin palpasyon ile
ağrı saptanıyor ve bu ağrı neticesinden hasta kendini muayene
ettirmekten vazgeçiyor. Yapılan laboratuvar tetkiklerinde lökosit
23.000/mm3,AST126 IU/L,ALT176 IU/L,ALP 213 IU/L, GGT345
IU/L, tota l bilirubin 12 mg/dl, direkt bilirubin 8 mg/dL geliyor.
Yapılan ultrasonografik incelemede şekildeki gibi safra kesesi
duvarının ödemli olduğu ve kese içinde çok sayıda taş bulundu-
ğu görü lüyor ve intra/ekstrahepatik safra yolları geniş, koledok
çapı 8 mm ölçülüyor.

Bu hastada en olası tanı buna yol açması en muhtemel mikroor-


ganizma aşağıdakilerden hangisidir?
A. Akut kol esistit - Escherichia coli
B. Akut pankreatit - Escherichia coli
C. Kolanjit - Escherichia coli
D. Kolanjit- Enterococcus faecium
E. Salmonella typhi
2023 MARTTUS SORU KAMPI
ıa t•~d
!!!!.I aKa , ,emısı
... Dahili e
13. 35 yaşında kadın hasta sırt ağrısı idar yaparken idrar
renginin kırmızı olması şikayetiyle geliyor. Çekilen direkt üriner
sistem grafilerinde her iki böbrek kaliksleri ni ve pelvisi doldu-
ran taşla uyumlu opasiteler görülüyor.

~~
Yapılan idrar kültüründe kanlı agar besiyeri yüzeyini buğu-dal-
gavari şekilde sarark üreyen, laktoz negatif, indol negatif, üreaz
pozitif, H2Süreten, çok hareketli, gram negatif basiller izole
ediliyor. Bu hastada grafideki görünüme yol açabilecek en olası
etken aşağıdakilerden hangisidir?
A. Escherichia coli
8. Enteroaderen Escherichia coli (EAEC) .
C. Proteus mirabilis
D. Haemophilus ducreyi
E. Shigella sonnei

14. 32 yaşında hemokromatozis sebebiyle takipli erkek hasta


beş gün önce göle girerken ayağını bir istiridyenin kestiğini
ifade ediyor. Ayağındaki yaranın iyileşmemesi, kendisini halsiz
hissemesi ve ağrısının artması nedeniyle acile başvuran has-
tanın yapılan muayenesinde yara çevresinde ödem ve eritem
izleniyor. Vital bulgularında tansiyonu 90/60 mmHg, ateşi 39°(,
nabzı 112 atım/dk izleniyor. Hastanın yarasından alınan örnek-
ten yapılan gram boyamada, kıvrık şekilli gram negatif basi ller
görülüyor. TCBS agarda şekildeki gibi bakterinin oksidaz pozitif,
laktoz pozitif, gram negatif, hareketli olduğu görü lüyor.

Yukarıdaki klinik ve laboratuvar bulguları dikkate alındığında


düşünülmesi gereken en olası etken aşağ ıdakilerd en hangisi-
dir?
A. Serratia marcescens
B. Vibrio vulnificus
C. Plesiomonas shigelloides
D. Calymmatobacterium granulomatis
E. Proteus mirabilis
15. Hayvanat bahçesinde hayvan bakıcısı olarak çalışan 35
yaşındaki bir erkek hasta sağ el başparmağında şe kildeki gibi
ülsere yara izleniyor.

Fizik muayenede sağ aksillada ağrılı lenf nodları palpe ediliyor.


Vitallerinde ateşinin olduğu dikkati çekiyor ve öyküsünden
herhangi bir hayvan tarafından ısırılmadığını ancak; hayvanla-
rın yünleri ve dışkılarıyla çok sık temas ettiğini öğreniliyor. Lenf
nodundan alınan materyalin Gram boyamada şekilideki gibi bol
lökosit ve gram negatif kokobasill er görülüyor.

Bu hastada klinikten sorumlu en olası etken aşağıdakilerden


hangisidir?
A. Francisella tularensis
B. Eikenella corrodens
C. Listeria monocytogenes
D. Bartonella henselae
E. Pasteurella multocida

16. Kompleman son ürünü olan membran atak kompleksi (CS-


9) eksikliğ in de hangi mikroorganizmanın enfeksiyon riskinde
artı ş izlenir?
A. Salmonella typhi
B. Staphylococcus aureus
C. Streptococcus pneumonia
D. Pseudomonas aeurigonsa
E. Neisseria meningitis

17. Aşağıdaki enfeksiyöz mikroorganizmalardan hangisinde


ateş o lm as ın a rağmen rölatif bradikardi gözlenir?
A. Salmonella typhi
B. Staphylococcus aureus
C. Streptococcus pneumonia
D. Pseudomonas aeurigonsa
E. Neisseria meningitis
la
l.!!.I! !!!!!I
tıgd
2023 MARTTUS SORU KAMPI
· . " Dahili e
aKa :emısı
18. Kronik diyaliz, evde infüzyon tedavisi almak ve evde yara
bakımı yapılması aşağıdaki etkenlerden hangisinin görülme
riskini belirgin şekilde artırır?
A. Metisiline rezistan s.aureus
B. Acinetobacter baummania
C. Streptococcus pneumonia
D. Enterobacteriaceae
E. Metisiline sensitif s.aureus

19. 43 yaşında on yıl önce travmaya bağlı splenektomi yapılmış


kadın hasta ateş şikayetiyle acil servise getiriliyor. Öyküsünden
splenektomi sonrasında aşılarının yapıldığını ancak 10 yıldır
takipsiz olduğu öğreniliyor. Bakılan vitallerinde ateşi 39°(, tan-
siyonu 60/50 mmHg, kalp atım hızı 140 atım/dk, soluk sayısı 28 =
soluk/dakika izleniyor. Bakılan vitallerinde lökosit 17.000/mm3, '
CRP 139 mg/dL, sedimentasyon 76 mm/h izleniyor. Bakılan
parmak ucu saturasyonu %96 gelen hastada bu şikayetlere yol
açması en muhtemel mikroorganizma aşağıdakilerden hangi-
sidir?
A. Acitenobacter spp
B. Streptococcus pneumonia
C. Neisseria meningitis
D. Listeria monocytogenes
E. Escherichia cali

20. Vakadaki hasta için en uygun antibiyoterapi aşağıdakiler­


den hangisidir?
A. Seftriakson + Ampisilin
B. Seftriakson + Vankomisin + Ampisilin
C. Seftriakson + Vankomisin
D. Vankomisin + Amfoterisin B
E. Klindamisin + Vankomisin

21. Temiz minör yaralanma s ı olan 34 yaşındaki erkek hastadan


tetanoz aşısı hakkında güvenilir bilgi alınamıyorsa en uygun
yaklaşım hangisi gibi olmalıdır?
A. Tetanoz aşısı
B. Tetanoz immünglobulini
C. Tetanoz aşısı+ İmmünglobulin
D. Profilaksiye gerek yoktur
E. 1 hafta Klavulanat tedavisi

You might also like